为了正常的体验网站,请在浏览器设置里面开启Javascript功能!

英语四级培训教师用书

2017-09-02 50页 doc 598KB 31阅读

用户头像

is_882336

暂无简介

举报
英语四级培训教师用书英语四级培训教师用书 听力 Zhu Yongfeng Xie Qin 第一部分 专项技巧详解 1、对话应试技巧 大学英语四级听力的对话部分由10个简短对话构成,每一段对话有一个问题。有的是从正面提问,有的是从侧面或反面提问;有的问题着眼于某个词或词组,有的则着眼于整段内容;有的直接能从对话中找到答案,有的则需要考生进行推理、判断或进行数字运算才能得到。综合起来,四级听力对话部分的题型大致可以分为四类:简单推断及归纳类型,否定类型,地点、人物、事件类型,数字、时间类型。 1、1 简单推断及归纳类型 一、应试技巧 ...
英语四级培训教师用书
英语四级培训教师用书 听力 Zhu Yongfeng Xie Qin 第一部分 专项技巧详解 1、对话应试技巧 大学英语四级听力的对话部分由10个简短对话构成,每一段对话有一个问题。有的是从正面提问,有的是从侧面或反面提问;有的问题着眼于某个词或词组,有的则着眼于整段内容;有的直接能从对话中找到答案,有的则需要考生进行推理、判断或进行数字运算才能得到。综合起来,四级听力对话部分的题型大致可以分为四类:简单推断及归纳类型,否定类型,地点、人物、事件类型,数字、时间类型。 1、1 简单推断及归纳类型 一、应试技巧 众所周知,像四级考试这样的大型全国性考试,选题都是相当严谨的。听力试题中的材料都有紧密的内在联系性和很强的逻辑性。由于要对考生有较高的要求,因此,大部分的对话题目都不是直接得到答案,必须经过相应的推断与归纳才能得到。在四级听力考试中,这种推断归纳题就有相当比重,而在六级听力试题中,这类题的比重又有进一步的提高。 1、对信息进行预测和判断 在人们的日常会话中,听众对所听信息的理解程度通常取决于其知识、经历以及对信息主题的熟悉程度。听者往往会依此就对话的内容进行一些超前的预测,以确定自己的作答和应对之策。四级听力对话题的材料都是日常生活中场景式的对话,上述的交谈规律同样存在并发挥作用。如我们可以从对话者的语气、语调、委婉表达,推测出说话者的态度、倾向性及话外音。甚至由试题给出的各选项也能推测出此对话材料的内容范围及问题的类型。如有一对话题的四个选项如下: A) Satisfied B) Tired C) Interested D) Discouraged 四个选项都是关于感受和态度的形容词,很容易猜测出,这段对话是关于对某事的态度或者感受的交谈,提问自然也是问某人的感受。在听材料之前考生如能浏览一下选项,并做相应预测,听的时候就能更容易抓住要点。 2、推断观点 英语会话有个十分重要的特点,那就是人们往往会用含蓄的方式来表达某种观点、要求或态度。例如: On entering the sitting room the mother says to her children ―The newspaper is all over the floor.‖ 母亲的弦外之音相当明确,那就是希望孩子们收拾一下地板上糟糕的报纸。而孩子只要稍加推敲,就会明白母亲的意图,自动收拾地板了。 在四、六级听力测试中,上述这种情况极为常见。说话的一方用含蓄的方式来表达某个意思,考生必须通过上下文、语气语调等信息,才能理解出其真实意图。 M: I‘m in big trouble, Jane. I need some cash badly but I haven‘t the slightest idea where I‘m going to get it. W: I‘m glad to hear that——I was afraid for a minute you might have an idea that you could borrow it from me. 用请下载www.docin.com/yongqing Q: What does the woman mean? A) She doesn‘t want to lend the man any money. B) She is glad that the man is in big trouble. C) She can show her favor by having an idea to get the cash. D) She hopes the man can only borrow the money from her. 男方遇到大麻烦——急需一笔钱,却不知道上哪儿去弄。在这种情况下,按照一般的理解,女方会慷慨解囊,资助男方,但这一题却反其道而行之。从女方开场白的glad就能看出苗头,哪有他人遇难还表示乐意的,听到后面我们就能明白,原来女方是怕男方会向她伸手借钱,所以才惴惴不安,当男方说不知上哪儿弄钱时,心里的石头才落地。glad是理解女方真实意图的关键,由glad再结合后面的话可以看出,女方十分不愿意借钱给男方,因此A项为正确答案。 要正确理解说话者的真实意图,就不能只片面理解字面意思,而应该结合具体语境、语气、和语调,仔细揣摩。 3、归纳概括结论 在谈话过程中,除了作一些推断之外,听者常依据对方谈话信息作出一些结论和概括。例如: W: Jack must have been joking when he said that he was going to quit his job. M: Don‘t be too sure. He told me that he was trying to sell his house. Q: What conclusion does the man want us to make? A) He doesn‘t believe that Jack will sell the house. B) He believes that Jack was joking. C) He agrees to what the woman said. D) He believes that Jack will quit his job. 女方用must have been joking 来表示自己的较为肯定的推断,即她认为Jack是在开玩笑,他不会真去辞掉工作。男方除了否定女方观点之外,又用另一个例子:Jack说他将卖掉房子,来论证女方所说事情的可能性,因为既然想卖房子,那么想辞掉工作很正常,也许Jack是打算出国或去别的地方工作。因此,男方的实际意思是,他相信女方说的事不是开玩笑,而是认真的。选D项。 1、2 否定类型 一、 应试技巧 否定类型的对话题,在四级听力测试中也是重点和难点。这类题常在对话中对某 段内容的全部或部分内容进行否定,要求考生能够听懂否定的部分,以正确理解意思。 此类题难点在于,英语中否定形式和否定表达相当多样化,各自的否定点和否定方式 都有差别,并且有的否定是用含蓄的方式表示出来的,这些都需要考生进行快速而准 确的判断。 1、 直接否定句 直接否定指句子中有明显的否定词或否定结构,如not, neither, nor等。包括一般否定、特指否定、全部否定、双重否定、部分否定等几类。 M: I won‘t ever go there again. W: I can‘t promise I won‘t return, but I‘ll try not to. Q: Which is true? A) They both will go back. B) The man will not return. C) The man promises to go back. 用请下载www.docin.com/yongqing D) The man won‘t try to avoid the place. 我们先看男方的话,意思比较明白,“再也不会回去了”。女方的话较为费解,其正确意思是:“我不能保证不回去,但我会尽量不回去”。意思是还是有可能会回去。弄懂两人各自真实意思之后,就容易选出B项为正确答案。 2、间接否定类型 即含蓄否定类型,在四、六级听力考题中大量出现的就是这种类型,也是较难把握的一类题,这种题从表面上看并不是否定,但意义上却包含否定的意思;或者使用了包含有否定意义的词,如miss, little, hardly, without, seldom等词,以及一些由否定前后缀构成的衍生词等,听的时候必须抓住这样的关键词,否则理解就会正好相反。 M: Let‘s go to the movies after dinner. W: Well, I‘ll go if you really want me to, but I‘m a little bit tired. Q: What do we learn from the conversation? A) The man is too tired to go to the movie. B) The woman wants to go to the movie. C) The woman wants to go out for dinner. D) The woman does not want to go to the movie. 表面上看两人说话都是肯定式,但分析女方的话可以知道,她的回答十分勉强,这表明实际上是不想去看电影的,but一词是关键词,它将语气转向了否定。正确答案是D。在这种类型题中,but经常会出现,起转折作用,因此考生应特别注意but之后的内容。 M: John is a smart boy. W: Yes, if he had studied harder, he could have got better grades. Q: What does the woman mean? A) John got better grades when he studied harder. B) John is getting better grades. C) John will get better grades. D) John did not get better grades because he did not study hard enough. 女方话中包含一个十分典型的对过去事实的虚拟语气句,如果能听出来,就知道实际情况与她说的完全相反,John不好好学习,也没有评上更好的等级,前者是后者的原因。因此正确答案是D。如果听不出来虚拟语气,就可能理解为肯定意思,而去选择A了。 1、3 地点、人物、事件类型 一、应试技巧 六级听力中,有关地点、人物、事件等方面的对话题占有一定的比例。 地点题主要涉及谈话者所处的场所或谈话内容涉及的地点。这些地点有些是谈话中直接提及的,有些则需要考生根据谈话内容作合理的想象和推断。遇到此类题,可以先阅读选项,然后集中精神听材料,抓住文中出现的地点名称,或者能体现特定地点的信息,再根据要求作出选择。 W: When do you think that I can go home, Dr. Smith? M: Well, you came in on Monday and today is Friday, I‘d say that you could probably leave tomorrow, but I don‘t want you to go back to work for several days. Q: Where is the woman in the conversation? A) At home B) In the doctor‘s office C) In a hospital D) At a hotel 此例涉及的地点虽然在谈话中通过Dr.一词有所暗示,但不明显,也无法通过它直接判断。Dr.有“医生”和“博士”两个意思,究竟是哪一个呢,后面男方说女方是周一来的, 用请下载www.docin.com/yongqing 周六就可以回家了,但不要工作还要休息几天。这很显然是在医院里住院病人与医生的交谈。所以选C。 人物题主要涉及谈话双方或谈话内容涉及的人的职业、身份及人物关系等内容。做这类题,先看一下材料比较好,因为这样就能基本知道会问什么,然后再听材料做题就比较主动。 M: Could you please tell me which stop I should get off for the MetropTitan Museum? Also, how much is the fare? W: Of course, I‘ll tell you when we get there. The fare is sixty cents. Just put it in the fare box. Q: What is the probable relationship between these two persons? A) Doctor and patient B) Passenger and driver C) Citizen and stranger D) Customer and merchant 这道题是十分典型的人物型对话题,要求从人物对话中分析出二者的身份关系。此题关键词在于stop, get off 和 fare。女方最后一句话Just put it in the fare box是双方关系的进一步暗示,只有乘客才会向公交车司机既问哪一站下车,又问车票价多少。因此选B。此题C项迷惑性也相当大,如果考生只听出了男方向女方问哪一站下车的信息,就极有可能选C,所以一定要听全信息。 事件题是指就谈话双方之间发生的事情或谈话内容涉及的事件提问。对话题中,二者的谈话往往是围绕某个事件展开的。事件题主要是测试考生的归纳概括和推理能力。因此,解答这类题,应该在听材料的过程中,把握好主题思想,再进行一定的归纳和概括。 W: Why do you look so happy this morning? You haven‘t seemed so happy these days. M: I just came from my adviser‘s office and found out that the college board had done away with the foreign language requirement for graduation. Q: What can we learn from the conversation? A) The man doesn‘t have to study the foreign language. B) The man just received an ―A‖ on his language exam. C) The man‘s advisor gave him some good advice on exam. D) The man doesn‘t have to take final language exam. 这段对话讲的是:女方发觉男方一反近来的低落情绪而突然兴高采烈,感到奇怪。男方告诉她,他高兴的是刚得知校方已取消了毕业时对外语的要求。显然,他是为自己不必再为毕业外语考试担忧而感到高兴。应该选择D项。 1、4 数字类型 一、应试技巧 四、六级考题中都会涉及到一定量的有关数字的题,这些题目有些是要求对文中所说的时间、价格、号码、年龄、数量等数字信息听清楚并直接作答——直接型;有些则要求根据所给数字进行比较或进行简单的加、减、乘、除运算——间接型。 直接型题的答案就在材料当中,所以,解题要点是要听清材料,排除干扰信息,设法从多个数字中找到正确的答案。实际上,这也是做好间接型题的必要前提。 M: Does the film start at 8:00 or 8:15? W: It starts at 8:30 and ends at 10:18. Q: When does the film start? A) At 8:15 B) At 10:18 C) At 8:00 D) At 8:30 用请下载www.docin.com/yongqing 此题共有四个不同的时间,因此听时最好做一下笔记,然后根据提问要求很容易选出正确时间。正确答案是D。如果没做笔记,即使都听懂了,但由于涉及时间很多,当问题出来后,却不知该选那一个了。 间接型题目的特点是听力材料中至少出现两个数字,其中一个数字在中间起参照作用。这类题关键是要听清各数字以及参照数字同其他数字之间的关系。体现数字之间关系的词很多,如:more, less, earlier, later, faster, slower, before, behind, double, triple, two-third (分数) 及有关动词如delay等。由这些关系对原数字进行简单运算,才能得出正确结果。 W: I thought that a typewriter ribbon cost three dollars. M: It used to, but the price increased by ninety-five cents last month and it is now twice as much as the original price. Q: How much more does the woman have to pay for three typewriter ribbons now than last month? A) ,18 B) ,11.85 C) ,6.15 D) ,2.05 这道题的运算较为复杂。对话中谈到,原来每条色带3美元,上月涨了95美分,即上月价格为每条3.95美元。现在价格又涨了,价格上涨一倍,即每条6美元。因此现在买3条需3×6=18美元。但题目并不是问现在买3条要多少钱,而是问现在买比上月买要多花多少钱。因此还要算出上月的价钱,即3×3.95=11.85美元,18,11.85=6.15美元,即为要多花的钱数。做这种题,听录音时精力要集中,计算要快捷,还要注意听清提问要求。 2、短文理解应试技巧 2、1 题型特点 短文部分一般包括三篇短文材料,每一篇材料后面,提出三、四个问题,要求考生做单项选择。六级听力短文理解部分,主要有以下两个特点: (1) 篇幅长,信息量大。每篇短文材料介于180—300字之间,基本信息的辨别、隐含信息的推理、综合信息的归纳都可能出现在同一篇材料中。另外,放音时间长,并且只读一遍,很容易出现听到后面就忘了前面的情况。记忆起来相对困难。 (2) 干扰信息强。由于短文材料信息量大,造成信息间的干扰也大,确定重点比较困难,不容易分清主次信息。 短文理解部分共有10道题,根据题型大致可分为归纳题、辨别题、情节认定题、推断题四大类。 一、归纳题 顾名思义,归纳题材要求在听完短文的基础上归纳总结出短文的主题思想;有的要求为短文选择最合适的题目;也有的要求对作者的结论或意图进行判断归纳。这类题目主要着眼于对全篇总体上的理解。对于大多考生来讲,听懂一篇材料的大意应该不是一件难事。此类题常见的提问方式有: What is the main topic of the passage? What do you think is the best title for the passage? What is the passage mainly about? What does the passage finally advise us to do? 要做好这类题目,首先要能从总体上把握材料内容。要求在主题上不失偏颇,特别是对于说明性和议论性的材料更要注意准确把握主题。如能注意准确把握主题,对某些细节就可进行合理的推断。另外,还应该有一定的归纳和总结能力,要能够从较为杂散的行文中归 用请下载www.docin.com/yongqing 纳出主要脉络和线索,如对故事性文章,就要能够对故事发展的过程有较清楚的认识。 二、辨别题 在听力测试中,常采用变换之后的信息或材料中没有出现过的信息作为干扰信息。辨别题要求考生能够在真实信息和干扰信息间进行正确判别,其目的在于考查考生对于材料中具体信息的把握程度。这类题常用以下方式提问: According to the passage, which of the following is (NOT) true? Which of the following is (NOT) mentioned in the passage? Which of the following statement is (NOT) true? 这类题几乎可以对文章涉及的所有内容进行考查,因此干扰性很强,很难把握。表面上看,只要在文章中听到相关信息,就很容易判断,但由于材料篇幅长,信息量大,在听的过程很多信息可能一带而过,没有注意,或者根本就没听懂。因此,对于考生来说,很难全面把握。这就要求考生在听的过程中,集中精神,不漏要点。另外,不要进行看似合理实际上没有根据的推理。 三、 情节认定题 情节认定题主要用于对具体细节信息进行考查。因为每篇短文中必然会包含许多细节来说明和论证中心。这类题提问方式不拘一格,依要考查的内容而异,如: What is the author‘s attitude towards…? Why does the man feel that…? What did the man do…? 可以看出,由于是对细节的考查,这类题的难度也是较大的。答题时除了要把握准有关细节信息,还要注意听清提问的角度和具体要求。 四、推断题 这类题要求考生根据所听内容推断出材料没有说明又有所暗示的内容,例如情节的发展趋势,作者的态度、倾向性等。推断题要求思维的逻辑性,要求能够以语篇为基础进行合理的推测。常用以下方式提问: What will be dealt with in the following paragraph? What does the passage probably imply at the end? What can we learn from the passage? What does the author imply by saying…? What does the story imply? What can we infer from the passage? 这类题有以下特点: 1. 难度高 推断题不仅要求考生把握原文信息,还要能够在此基础上合理地进行推断。关键在于这个“合理”的尺度较难把握。因为大部分推断题只要求在材料论述的内容范围内进行推测,如果过多加入自己的理解,可能会超出材料内容,选出错误的结果。所以,回答这类推理题,一定要谨慎推断。 2. 逻辑性强 由于要进行推断,当然要求有严密的逻辑性,否则,极容易被误导。 2、2 不同体裁短文应试技巧 一、叙述型 叙述型短篇是短篇听力部分的主要类型。这种体裁往往是既有叙述又有描写。题材十分广泛,如地理风貌、气候景色、名胜古迹、人文风格、人物性格、行为思想、心理活动、情感等。幽默类小故事以及简单的传记或短文显然也可以归入此类,只是此类文章难度较小, 用请下载www.docin.com/yongqing 在六级听力中已较少采用。叙述型短文往往都具有条理清晰,层次分明的特点,比较容易把握。 二、议论说明型 议论说明型短文能够很好地检验考生的理解能力、信息把握能力和归纳推理能力。议论文往往在开篇即将论点摆出来,如以设问句开头,引出论点等,而后围绕论点进行论述,最后达到确立论点的目的。这类文章常用推理的办法,如归纳、演绎、类比等方法;说明文则一般会选择一个事物或事物的某个方面,从多个角度进行说明,以使被说明的事物特征鲜明而清楚,常用定义、例证、分类、比较等多种方法。有时,在一篇短文中,议论说明是混合使用的,往往既有议论又有说明。 三、科普类文章 科普类文章在六级听力中一直是重点和难点,它的行文方式、论述方法都与议论说明型文章相似。不过因为这类文章选题都是一些较为专业的学科领域,往往有一些专业词汇。如果是考生不熟悉的学科领域,理解起来会有相当的难度。对于这类文章,考生要充分利用逻辑思维,结合上下文信息来理解文章内容;对于比较熟的学科领域,也可结合自己的知识来理解文章内容甚至答案。 3、听写应试技巧 听写是国家对英语等级考试改革的一大举措。增加听写题型的目的是为了提高听力测试的可信度和难度,更真实地反映学生的听力水平。它包括两方面的能力考查——对听力理解能力的考查和对书面表达能力的考查。因此要想做好听写题,就不能忽视其中任何一种能力的培养。 在大学英语六级听力测试中,听写测试有两种供选择的题型:短文听写和复合式听写。 3、1 短文听写 要做好短文听写题,考生可以遵循以下技巧: (1) 听前通读,从总体上把握大意。 (2) 听第一遍时,重点在于把握文章意思,不要急于下笔。在听音的时候,看卷面上的文字,加深理解,并特别留意卷面空白部分,因为那是要听写的部分,对卷面上已有的部分只要稍加注意,能跟得上节奏就成。 (3)听第二遍时,利用读到空格后的停顿快速书写空缺处内容。因为第一遍已基本听懂要填写的信息,因此这一遍要及时写下来。另外,一定要注意,要等到空格内容读完之后再动笔,以免遗漏掉后面内容。 (4)听第三遍时,检查修改,仔细判断刚才所写的内容是否与原文材料相符。 (5) 一定要注意用单词的适当形式填写。英文中单词有单复数、时态、语态形式的不同,拼写、读音可能都有变化。但在听的时候,材料对于这些细节往往读得并不明显,或者考生没有听清楚,这时最好的办法就是从上下文推断此单词应有的形式。 3、2 复合式听写 一、应试技巧 复合式听写是很有潜力的考试题型,因为它要求考生有较为全面的英文知识。同时这也意味着,考生必须有较好的英文功底,才能做好这类题。 这种题型具有以下特点: (1) 篇幅较长,信息量大,辨认处理困难,对捕捉信息要求高。 (2) 放音时间长,连贯性强,信息记忆困难,对强化记忆要求高。 (3) 前后关联紧,整体理解困难,对融会贯通要求高。 用请下载www.docin.com/yongqing (4) 要求主观参与,听懂并写出,还要对听到的信息加以归纳整理,对书面表达能力要求高。 第二部分 通用技巧详解 一、 各种听力考项通用办法 ,(听音前抓紧时间阅读选择项 在考生拿到试题册、正式开始做听力理解考试题之前,考生都应该尽量挤出时间,快速浏览选择项。如:每节听力理解题开始之前,磁带里都有一段考试要求(Directions),Section A还有一段对话实例。考生务必在考前熟悉考题的要求,考试时,则可以利用这段时间来阅读选择项,提高听力理解力。 (1)听音前抓紧时间阅读选择项,可以达到以下三点目的: a. 选择项里出现的文字,有助于提高听力理解力。如:He had his foot caught between two posts in the river这句话,如果没有任何文字资料,就不易听懂,因为听者很难想象、相信某人的脚被河里的两根柱子夹着。但在读了文字资料后,再听,就很容易听懂。 b. 根据选择项里的重要信息,预测听力内容。如: A) She doesn‘t have any cigarettes with her. B) She doesn‘t want him to smoke. C) She has no objection. D) She doesn‘t smoke. 从以上四个选择项里,我们就完全可以正确地预测这段对话的大致内容:她自己是否吸烟;她是否反对他人吸烟。 c. 根据选择项里的重要信息,推断题目的问题。如: A) Nobody came to talk to the speaker. B) People didn‘t listen to the speaker. C) People kept interrupting the speaker. D) People make fun o f the speaker. 从以上四个选择项里,我们可以对该题的问题作出预测:听众对演讲者的反应或态度。 (2)读选择项也应该注意技巧。如果一字一句地读,不仅耽误时间,而且有可能事倍功半,因为短时间里接受的信息太多,抓不住重点。所以一定要注意选择项里的重要信息。 下面,对阅读选择项的有关技巧,从四个方面进行介绍: a. 对选择项出现的语言现象要敏感。如: A) Librarian and student B) Customer and repairman C) Boss and secretary D) Operator and caller 以上选择项均为有一定关系的人,我们完全可以预测该题的问题是关于对话人的身份。所以,听音时对这方面的问题要特别敏感。 A) Look for the key B) Repair the car C) Paint a shelf D) Fix a shelf 以上选择项均为动词,我们完全可以预测该题的问题是关于某人将干什么事。 b. 注意选择项里重复出现的单词,一般都为关键词。如: A) He didn‘t pass the physics exam. B) He didn‘t work as hard as he was supposed to. C) He did better in an earlier exam. D) He found sth. wrong with the exam. exam 一词三次出现在选择项里,椐此我们可以推断听力材料可能与exam 有关。听音时,就可以着重听有关exam 的信息。 但有时,选择项文字材料比较复杂,重复词不止一个,我们就应该更加注意快速浏览, 用请下载www.docin.com/yongqing 准确把握重要信息。如:从以下四项选择项里,我们就发现,Mike及party 都出现两次,应该为重要信息。 A) They don‘t know how to get to Mike‘s home. B) They went to the same party some time ago. C) They are discussing when to meet again. D) They will go to Mike‘s birthday party. c. 如果选择项需要阅读文字太多,可以读两项选择,对听力材料的大致内容及问题的种类,作出预测。如: A) To attract more students. B) To make the summer school more like a holiday. C) To let the students have a good rest. D) To make the courses suitable for students of all levels. A) Because they all work very hard. B) Because their teacher are all native speakers of English C) Because they are all advanced students. D) Because they learn not only in but also out of class. 以上两题的选择项都比较长,我们都只要分别读两项,就能对听力材料及问题的大致内容,作出预测。第一题的问题是有关做某事的目的,第二题的问题是有关做某事的原因。而且,这两题都与学生的学习有关。 d. 如果选择项需要阅读文字太多,还可以采取竖着阅读选择项的办法。专门挑读主语、谓语或宾语,抓句子的重要信息。如: A) He fell into the river but couldn‘t swim. B) He fell into the river together with his bike. C) He had his foot caught between two posts in the river. D) He dived into the river but couldn‘t reach the surface. 又如: A) He asked what the young man‘s name was. B) He asked the young man to take hem home. C) He gave his name and then ran away. D) He thanked the young man and then ran away. 以上两题的四个选择项,只要阅读谓语及宾语部分,就能抓住重要信息,对有关听力材料及问题作出正确判断。 2. 在选择项后勤做记号,与选择项无关的不用记笔记。 考生在听音时,应该学会边听边筛选选择项。有时,四个选择项,有三项选择项都先后提到。这种情况下,只要提到的选择项,就在旁边做一记号,并分别记下与各选择项内容有关的信息。在做短文听力时,有时也有这样的问题:Which is not the case according to the passage you have heard? What is not mentioned in the passage? 这时,三个选择项都应该在听力材料中出现。所以,只要听到某个选择项被提到,都应该在旁边注上记号,并记下有关信息。有时,选择项都为年月,而且听力材料里可能四个年月都提到,象这样的题,肯定是问具体某件事在什么年月发生。如果听音过程不做笔记,最后就很难猜出正确答案。注意,与选择项无关的不用记笔记,这样,考生可以集中精力收听与答案有关的句子。请看下面一个简单的例子: W: The blue bag is $15.00 and the red one is $ 10.00. 用请下载www.docin.com/yongqing M: The black one is only $8.00. Q: How much is the red one? A) $ 8.00 B) $10.00 C) $25.00 D) $15.00 这种题是对话部分的一个简单计算题 。但如果考生只听,不做任何笔记,加上考场上的紧张气氛,恐怕也不易立即选出正确的答案。相反,只要简单地做点笔记,在$15.00后写上一个字母 ―b‖, 表示blue, 在$10.00后写上 ―r‖, 在 $ 8.00后下上 ―bla‖, 做这种题就能稳操胜券。 3. 控制答题时间 题与题之间约有 15秒间隔,也就是说,可以用于回答时间只有 15 秒钟。但是,如果回答足足用了 15 秒,就有可能影响做下一个题目。因此,回答最好控制在 13 秒以内。若对前一个问题不甚清楚,最好先猜选一个答案,准备好仔细听下一题,否则就可能影响下一题的得分。 4. 平时,要注意提高语言基本功 (1) 语音:如:The professor asked his assistant to collect the papers for him与The professor asked his assistant to correct the papers for him。又如:It‘s a great pleasure 与It‘s a great pressure 。这两组句子只有一音之差。 (2)音变现象:如:学生对连读、弱读、句子重音、语调等掌握好坏,直接影响听力的效果。同样一句 He came to visit me, 如果采用不同的语调,句子含义就会有差异。降调表示肯定,升调表示怀疑。 (3) 语法: 如下面的对话题: W: I really can‘t stand the way David controls the conversation all the time. If he‘s going to be at the Christmas party, I just won‘t come. M: I‘m sorry you feel that way, but my mother insists that he come. 如果考生对insist这个需要接虚拟语气的词掌握较好,听懂男士的话,应该说问题不大。 (4) 单词、短语及惯用法: 这方面的基本功是否扎实,也直接影响听力的成绩。如以下是一些口语中常用的惯用语: He is the last person to undertake illegal dealings. 他最不可能干违法的交易。 Let‘s go Dutch. 我们各付各的。 Let‘s call it a day. 今天就到这儿吧。 I can‘t agree with you more. 我十分同意你的意见。 You can‘t be too careful on this matter. 在这件事情上,你再仔细也不为过。 Useful expressions: not in the mood; take your time; book up; on reserve; in season; cut a class; cut school; for anything; on the right track; make it; 5. 积累英语国家文化背景知识 听力材料都与英语国家的文化背景有着或多或少的关系。考生如果对某段听力材料文化背景熟悉,借助背景知识,考生就能更好地把握听力材料。所以,同学们在平时的英语学习中,也应注意积累有关英语国家文化背景知识。 二. 对话听力应试技巧 1. 正确理解委婉表达方法。如: W: John, do you want to go swimming with me today? M: Sure. But I can‘t. I have got an appointment with my professor at three o‘clock.. 如果这题的问题是:他是否去游泳了,同学们不能因为 Sure 就推断他去游泳了,而应根据后面But的内容作出推断,判断出他必须与他的老师会面。又如: I hope these apples are as good as they look. 这句话委婉表达的意思是:恐怕这些苹果不 用请下载www.docin.com/yongqing 如他们看上去那么好。 2. 正确推测言外之意。如: W: Is your boss going to raise your wages? M: Does the sun come up from the East? 中国人常用“太阳从西边出来”比喻某事没有希望。而这里英语用的是反问:难道太阳不是从东边出来吗,言外之意是:老板给我加薪,就象太阳从东边出来一样自然。 M: Did you watch the game last night? W: I wouldn‘t have missed it for anything. 如果这题的问题是:他是否看了球赛, 同学们不能因为 原文出现miss (错过) 就推断他错过了球赛,而应根据整句话的内容作出推断:他说什么也不会错过球赛。 3. 正确理解否定的含义,注意双重否定等于肯定。如: M: John is a smart boy. W: Yes, if he had studied more, he could have gotten better grades. Q: What does the second speaker mean? A) John got better grades when he studied. B) John is getting better grades by studying. C) John will get better grades because he did not study hard enough. D) John did not get better grades because he did not study hard enough. 答案: D) 双重否定也经常出现在听力考试中。如: W: Did you attend his lecture yesterday? M: Yes. There is nothing short of brilliance. Q: What can we infer about the lecture? 答案应该是:The lecture is marvelous. 4. 归纳总结,抓住中心思想,不能只根据只言片语来作出选择。如: W: Did you want a day course or an evening course? M: Well, it would have to be an evening course since I work during the day. 如果这题的问题是:他们在谈论什么,既不能选 a day course , 又不能选 an evening course, 而应该选 the choices of courses. 又如: W: If it hadn‘t been snowing so badly, I might have been at home by 9 o‘clock. M: It‘s so bad that you didn‘t make it. Jane is here. She wanted to see you. 如果这题的问题是:What happened to the woman? 答案应该是:She was delayed. 5. 考生对会话部分的出题方式了如指掌,有利于推断问题、进行积极而有目的的收听。对话部分的问题比较容易推测,从题下的四个选择项里就能推测出来。这部分的问题大致分为以下五种: (1)问数字 (2) 问地点 考试中常见的地点有:at a bookstore, at a travel agency, at a post office, on the street, in a garden, in a hotel, at home, in a department store, at the restaurant, in a plane, in a grocery store, at a drugstore, in a tailor‘s, etc.。如: (3)问身份(参照问地点的题) (4)细节辨识 对话的一方或双方围绕一个话题提出与之相关的多个细节,要求考生辨识其中细节之一。如: M: We already have a sofa, a very good one, I think. 用请下载www.docin.com/yongqing W: I never like it. It‘s brown and our other furniture is light-blue. it doesn‘t match. The yellow one in the window will look good with our other things. Q: Which of the follow sofa doesn‘t the woman like? A) A yellow one B) A gray one C) A brown one D) A light-blue one 答案是:C)。 (5)问由谈话内容推出的结论 这类问题比较难,考生必须在把握对话的完整内容的基础上才能作出选择。这类问题最常见的形式是:What do we learn from this conversation? 或 What do we learn about A from this passage? ...如: M: Bob doesn‘t stand a chance of winning the first prize in the World Ice Skating Championship. W: I think so, too; because he was away from the skating rink for three months, but he is doing his best. Q: What do we learn from this conversation? A) Bob is standing in line for the first prize. B) Bob was the best, so he got the first prize. C) Bob probably won‘t win the first prize. D) Bob wanted to get the first prize and got it. 答案是: C)。 6. 对话中常涉及的地点及各种用语: (1)银行用语 open an account 开银行帐户 a savings account 存款帐户 a checking account 支票帐户 „percent interest „的利息 withdraw 提款 deposit 存款 balance at a bank 银行存款余额 a teller 出纳员 cash a check 支票兑换现金 a penny = one cent a nickel = five cents a dime = 10 cents a quarter = 25 cents a half dollar = 50 cents (2)邮局用语 mail that package 邮寄包裹 parcel post 邮小包 a letter by air mail 航空信 a registered letter 挂号信 postage 邮资 remittance 汇款 (3) 理发店用语 long on top 上面长 trim one‘s hair 修理头发 take off the sides 修侧面的头发 beard 胡子 (4)诊所用语 prescription 处方 lose one‘s appetite 没有食欲 a sour throat 喉咙痛 come down with a cold 感冒病倒 an injection 注射 take one‘s temperature 量体温 (5)旅行社用语 a round-trip ticket 来回车票 one-way fare 单程费用 traveler‘s check 旅行支票 first class 头等座 用请下载www.docin.com/yongqing (6)机场用语 a window seat 靠窗座位 an aisle seat 靠走道座位 boarding card 登机证 depart from Gate 6 由6号门登机 (7)电话用语 make a local call 打市内电话 make a long distance call 打长途电话 dial your number 拨号 make a collect call 打对方付款的电话 The line is busy. 占线 hang up 挂断电话 (8) 学校用语 credit system 学分制 required course 必修课 term paper 学期论文 diploma 毕业证 tuition 学费 scholarship 奖学金 fellowship 研究金 teaching assistantship 助教金 7.易混淆短语 下面是常用的、且在意义上容易弄错的一些短语、习语和习惯用法 absence of mind 心不在焉 cannot help doing sth 不禁要(作某事) back number 过期刊物(美) check in 登记;报道 back up 支持;援助 check out 结帐;付款退租旅馆房间 beyond control 无法控制 come down in the world 落魄 beyond description 无法形容 come forth 出现;涌现;被公布 beyond doubt 无疑的;不容怀疑 come into being 形成;产生 beyond question 毫无争议;无可争辩 come into effect 开始生效 burn one‘s boats 破釜沉舟 come into power 执政;掌权 burn the midnight oil 开夜车;工作到深夜 come off with flying colors大功告成;凯旋 call for 要求;需要;提倡 come to 达到;苏醒 call it a day 今天到此为止;收工 come to an agreement 达成 call off 取消 come to an end 结束;终止 call on 号召;呼吁;拜访 come to light 暴露;被发现 call one‘s attention to 叫某人注意 come to one‘s senses 苏醒过来;醒悟 call the roll 点名 come to oneself 苏醒;醒悟;恢复知觉 come to terms with 达成协议;妥协;让步 give way (to) 屈服于;让步于 come to the point 到达要点 give in 投降;屈服;让步 come up with 想出(等);提出;赶上 give off 发出,放出(蒸汽、气味、光) count for little 无足轻重;不足取 give out 分发;用尽;发出(声音、热) count for much 非常重要;关系重 give one‘s hand to sb 答应和某人结婚 count for nothing 毫无价值 give sb a hand 帮助某人 count in 把„计算在内 give sb a lift 让某人搭车,帮某人忙 count on/upon 依靠;指望 give up 放弃(念头、希望等) do away with 废除 hardly/scarcely„when„刚„就„;一„就„ do good to 有益于 leave „to sb 有某人决定„ do one‘s bit 尽自己的一份力量 lose heart 灰心;泄气;丧失勇气 do sb a favor 帮(某人)一个忙 lose one‘s heart to sb 爱上某人;非常喜欢 do sb wrong 冤枉好人;委屈某人 make a point of doing sth 决心做某事 do well in 在„方面做的好;成绩好 man of iron 意志坚强的人 用请下载www.docin.com/yongqing eat one‘s words 收回前言;认错道歉 miss the boat/bus 错过机会;坐失良机 break one‘s promise/word 食言 out of question 毫无疑问;一定 fall asleep 入睡 out of the question 不能考虑的;不可能的 fall back on sb 求助于(某人) out of tune (with„) (与„)不一致;不协调 fall behind 落后;根不上 play one‘s best card 使出绝招 fall ill/sick 生病 put all one‘s eggs in one basket 孤注一掷 fall into a sleep 睡了一觉 ring a bell 使想起„;听起来觉得耳熟 fill in/out 填写 sleep like a log/top 酣睡;睡的很熟 for the time being 暂时;眼下 take a short cut 走捷径 get along (with) 相处;进展;设法生活下去 take French leave 不告而别 get down to one‘s work 静下心来工作 throw cold water on 对„泼冷水;不赞成 get in one‘s way 挡路;妨碍 turn a blind eye to 熟视无睹;假装未见 get in trouble 惹上麻烦 turn a deaf ear to 对„充耳不闻;置若罔闻 get into hot water 陷入困境 turn down 减弱,降低(声音等);拒绝 get into the habit of 养成„的习惯 give rise to 引起,使发生 get rid of 摆脱,除去 give birth to 产生 get the wrong number 号码错了 三. 短文听力应试技巧 1. 听音前,抓紧时间阅读选择项尤其重要( Anticipate the topics and the questions) 听音前阅读选择项,有利于把握短文中将出现的关键词,并猜测短文的大意。每篇短文的选择项都是一个整体,意义往往互相关联。如有一篇听力短文是比较难懂的科普性文章。如果听音前不读选择项,听力理解的难度很大,但阅读选择项后,在猜准了关键词、短文的内容大意后,听力理解的难度会大为降低。如: 1. A) Gas B) Vapor C) Water D) Chemicals 2. A) By heating dry ice B) By passing steam over dry ice C) Vapor D) By mixing dry ice with ordinary ice 3. A) It takes a longer time to melt. B) It is not so cold as ordinary ice. C) It is lighter to carry. D) It is cleaner to use than ordinary ice. 4. A) In the 1920‘s B) In the 1930‘s C) In the 1940‘s D) In the 1950‘s 从上面第二题、第四题中重复出现的关键词:ordinary ice 和dry ice,我们完全可以对文章的大意作出猜测:这是一篇有关dry ice and ordinary ice 的文章。 2. 边听边找答案,重点收听与选择项有关的信息( Exception: Just listen to the passage and ignore the answers) 就上一篇文章的选择项来说,要做好第一题,就必须非常注意与Gas、 Vapor、 Water、Chemicals有关的信息。原文只要提到某个词,就应该在相应的选择项后做个记号。听音中,我们发现只有一句与上述选择项有关:Dry ice is made by freezing a gas. 因此,答案即是 D)。 另外,有时问题的顺序与原文顺序不一致。所以,考生在做题时,应该尽量兼顾下面的题。如上一篇文章的第四题,在做完第一题后就出现,如果考生作完第一题后,只盯着第二题,很可能错过很容易拿分的第四题。 最后,特别提出的是,以上一篇短文为例,第三题最难,但如果考生做题时能够通过选择项对问题作出正确预测,听力理解难度就会大为降低。四项选择项均为比较级,而且可以作进一步推测:dry ice 与ordinary ice 比,有什么不同。 用请下载www.docin.com/yongqing 又如:[ Anticipate topics and questions] 1. A) Find work on campus 4. A) Every morning B) Work in the employment office B) Afternoons and weekends C) Help students find jobs C) When he is in class D) Ask the woman questions D) Weekdays 2. A) In the library 5. A) Fill out a form B) In a classroom B) Give her some additional information C) In a campus office C) Tell her some news D) In an apartment D) Phone her 3. A) No more than ten B) At least twenty C) Not more than twenty D) Up to ten 3. 通过主题句(topic sentence, usually at the beginning of the passage), 把握短文的完 整脉络 尽管短文中的有些题目是针对具体事实的,但考生如果能抓住全文的主题思想或大意, 将有利于猜测、了解细节问题。听音时,听清第一句尤为重要,因为80% 以上的主题句出 现在句首。听清了主题句,就容易领会作者的观点和态度,诸如肯定、否定、怀疑、询问、 幽默、讽刺等。 When you hear: The major earthquake that occurred east of Los Angeles in 1971 is still affecting the economy of the area today. You will think: The topic of the passage is the effect of the 1971 earthquake on Los Angeles today. 4. 注意过渡词语、不要因为个别难词、难句而影响做题 听语篇时,听者没有必要也不可能把每个单词或单句都听得清楚。 然而,考生在听语 篇时往往跳不出单词、句子的圈子,常常把注意力集中到词、句或具体的细节上。如果一个 单词听不懂,便会停下来琢磨一阵,等反应过来时,录音已过了好几句了,结果常常是顾此 失彼,只记得支离破碎的几个词或几个句子,抓不住语篇的主要信息,对短文缺乏整体概念。 所以,考生在听音时,除了要抓主题句外,还应注意过渡词语,注意表示短文思路、结构的 词,如有些结构词表达短文空间、时间、举例、例证、对照、比较、引申、转折、推论、或 总结等逻辑关系。 5. 把握短文听力的题材,积极预测短文内容及听力理解问题 (1) 社会问题 这类文章的范围包括:妇女解放、民族问题、劳动就业、城市污染、能源、住房、交通、 卫生等。大多数文章形式是作者先通过揭示、分析、研究某一社会问题,然后再阐明作者的 观点和态度。这类文章的主观问题多于客观问题。回答这些问题,常常需要对短文进行分析 及推理。 (2) 一般知识 这类文章在考试中比较常见,包括:文化教育、风土人情、地区特征、节假日、历史事 件、法律、宗教、文学、艺术等。这类文章的特点是富有趣味性,短小精悍、题材丰富,通 俗易懂。短文的提问以客观事实为主。 (3) 科普文章 有关自然科学各方面的文章,其目的是把自然科学领域新观察、新发明、新技术等作一 用请下载www.docin.com/yongqing 介绍。文章结构比较严谨,常使用被动语态,论点及论据都很严密。在提问形式上,主观性的逻辑推理、综合归纳于客观性的细节问题各占一半。 (4) 传记与故事 文章题材多为记叙文,具有时间、地点、人物及情节四大要素,情节一般不太复杂。短文的问题一般以when、 where、 who、what、why、how等词开头。 Key to “ Anticipate topics and questions”: Topic: Finding a job on Campus. Clues for the guess: A) Work on campus; B) Employment office; C) Students‘ job Questions: 1. What will he or she try to do? does the conversation probably take place? 2. Where 3. How many numbers do you want to finish? 4. When can you do your work? 5. What are the students going to do? 如没有听懂,可用下列skills:(通过选项猜测答案) Skill 1 听到的不要选 Example 1 M: The place I‟ve heard so much about is Los Angeles. The climate is pretty good. Year-round swimming. How do you like it? W: Well, the beaches are beautiful. But the people there are terribly annoyed by the dirty air. I mean, the combination of fog, smoke, and automobile exhaust. There is not enough wind to blow it away. Q: What does the man think of Los Angeles? A) The air is polluted. B)The people there are terrible. C)It‟s too windy. D)The beaches are dirty. Skill 2 BUT句型 答案均在BUT后。 Skill 3 含义不肯定的可能是答案 Possible words include: can; could; probably; may; might; be likely to Skill 4 “SUGGEST”的技巧 选择项中包括suggest一词的可能是答案。 (其实也是不肯定的一种表现。) Skill 5 选择项中包括some的可能是答案 Skill 6 选择项表示抽象概念可能是答案 Skill 7 择项意思复杂的、包括转折词的可能是答案 Skill 8 择项中包括比较结构的可能是答案 Skill 9注意选择项与录音原文的词意替换。 用请下载www.docin.com/yongqing 第三部分 Listening Exercises 听力原文及练习答案 一、 分类训练 1、 时间题 1. W: The plane leaves at 6:15. Do we have time to eat first? M: No. We only have 40 minutes left. Q: What time is it now? ( C ) 2. M: Will Dr. Black be able to see me at nine fifteen tomorrow? W: Sorry, but he‘s fully booked till eleven unless there‘s a cancellation. Would ten to one be convenient? Q: When will Dr. Black be free? ( C ) 3. W: Excuse me. Do you have the time? M: Yes ma‘am. I have 1:15, but my watch is a little bit fast. Q: What time is it? ( D ) 4. M: Hello. This is John Smith. I have an appointment with Mrs. Anderson at five o‘clock this afternoon, but I‘m afraid I‘ll have to be about fifteen minutes late. W: That‘s OK, Mr. Smith. She doesn‘t have another appointment scheduled until six o‘clock. Q: When will Mr. Smith most probably meet with Mrs. Anderson? ( C ) 5. W: Why don‘t you try to leave your office at 6:15 and I‘ll try to pick you up at 6:30? M: That may be a little late if we want to be sure of getting good seats. I think I‘d rather leave here at 6:00. Q: When will the man probably meet the woman? ( B ) 2、 地点和方向题 1. M: Well, let me look at this one. Yes, I think we can get off here and wee what is on. W: It‘s an air-conditioned one, let‘s have a good time here instead of the theatre. Q: Where are they probably going? ( B ) 2. W: Are you glad that you came to Washington? M: Yes, indeed. I‘d considered going to New York or Boston, but I‘ve never regretted my decision. Q: Where does the man live? ( D ) 3. M: Shall I get off at the corner of this street? W: No. According to the map, it‘s the next one after the traffic lights. Q: Where is the man? ( B ) 4. W: You are leaving for Los Angeles. I hope you have a good flight. M: Thanks. The weather is said to be clear, so it should be pretty smooth. Q: Where did this conversation most probably take place? ( A ) 5. W: What can I do for you, sir? M: Well, I‘d like to get this gook renewed. You know, it‘s so helpful that I can‘t finish my research paper without it. Q: Where are the man and woman? ( D ) 3、 数字计算题 1. W: How many students tried out for the basketball team this year? M: About 40, but only half of them have real talent for the sport. Q: How many students are good at basketball? ( D ) 2. M: Hello! Is this triple eight double zero four two? W: Yes. Who is speaking? Q: What is the telephone number? ( D ) 3. M: Is this 415 Fifth Street? W: No, it‘s 514 Fourth Street. Q: What address is the man looking for? ( D ) 4. M: How much is the rent? W: It‘s $150 a month unfurnished or $200 a month furnished. Utilities are $25 extra. Q: How much will it cost the man to rent an unfurnished apartment including utilities? (B) 5. M: How many dozens of hamburgers do you need today? 用请下载www.docin.com/yongqing W: Usually I take three dozens, but now I cut it down to half. Q: How many hamburgers does she need now? ( C ) 4、 身份职业题 1. W: That‘s a long distance call. You‘ll have to call Los Angeles Information. M: Could you please give me the area code? I don‘t have a directory. Q: What is the woman? ( B ) 2. M: May I help you, madam? W: Yes, I‘ll have two hamburgers and a cake. M: Would you care for anything else? W: No, that will be all. Q: What is the man‘s profession? ( B ) 3. W: Jack sent you a present yesterday. M: I got one from John but not from Jack. Q: Who did the man get a present from? ( C ) 4. M: Good evening, madam. Would you like to sit here? I‘m afraid that there are no other seats free at the moment. W: I‘d prefer to sit alone but I suppose this will do. Have you a menu, please? Q: What is the relationship between the two speakers? (A ) 5. M: Make thirty copies for me and twenty copies for Mr. Brown. W: Sure, sir. As soon as I make the final correction on the original. Q: What is the woman‘s job? ( A ) 5、 建议、请求及提议题 1. W: I‘m interested in the advertising job that you have offered. M: Oh, yes. First, fill out this form, and then someone will be with you in a few minutes. Q: What does the man mean? ( B ) 2. W: Can I help you? M: Yes, Madame. I saw a note outside that says ―Rooms For Rent‖. Can I rent an apartment with a bathroom and a kitchen? Q: What does the man want? ( C ) 3. M: I‘m afraid that you can‘t get into the cinema because you do not have a ticket. W: I‘ve lost it, but I can find a person to prove that I did have one ticket. Is that all right? Q: What did the woman offer to do? ( C ) 4. M: I‘d like to drive to the concert, but my brother is using the car tonight. W: No problem. How about the subway? And the station isn‘t far away. Q: What‘s the woman‘s suggestion? ( B ) 5. W: If I were you, I would take a plane instead of a bus. It will take you forever to get there. M: But flying makes me so nervous. Q: What does the man prefer to do? (C ) 6、 条件关系题 1. W: I wish I could , but I‘m late already. M: What a shame! Q: What does the man mean? ( B ) 2. M: Do you think Ed will get here on time? W: If Ed doesn‘t, nobody will. Q: What does the woman mean? (C ) 3. W: If you are promoted to manager, will you give us a raise? M: No problem. What else would I do with all the money that will come pouring in? Q: What will happen to the woman if he is promoted? ( A ) 4. W: What was the weather like during Spring Festival? M: Not bad, but it would have been better had it been a little warmer. Q: What was the weather like during the man‘s holiday? ( C ) 5. M: I wonder if I could borrow your bicycle. W: You certainly could if I had one. Q: What does the woman mean? ( C ) 用请下载www.docin.com/yongqing 7、 因果关系题 1. W: What do you think of his experiment? M: He has done well considering he has no experience. Q: Why does he think he has done well? ( A ) 2. W: Are you going to the Johnson‘s party tomorrow night? M: I don‘t think so. I have to work and my wife will be out of town. Q: Why isn‘t the man going to the party? ( B ) 3. M: Mary, why isn‘t Jane teaching here this term? W: She can‘t. She was fired. Q: What reason was given for Jane‘s not teaching? ( B ) 4. M: The taxi driver must have been driving too fast. W: I don‘t think so. He crashed into the tree because he was trying not to hit a box that had fallen off the truck just ahead of him. Q: What happened to the taxi driver? ( A ) 5. W: Mr. Hampton‘s explanation was too complicated to understand. M: He didn‘t speak slowly enough for us to take notes, either. Q: What does the second speaker complain about? ( A ) 8、 比较关系题 1. M: I read a poem by E.E. Cummings last night. It is very beautiful. Do you like modern poetry? W: I like modern poetry, but not as much as traditional poetry. Q: What kind of poetry does the woman like better? ( C ) 2. W: Your red and green shirt looks nice. M: Thank you but I like blue best. Q: What color does the man like? ( C ) 3. W: I like science more than maths, but history is my favorite subject. M: But I think computer programming is the best. Q: Which subject does the woman like the most? ( C ) 4. M: What would you like to have for dessert, apples or oranges? W: She makes a better wife than mother. Q: What does the woman mean? ( B ) 5. M: What would you like to have for dessert, apples or oranges? W: As for dessert, I‘d like to have oranges rather than apples. But usually I prefer apricots to anything else. Q: What did the woman dislike to have this time for dessert? ( A ) 9、 否定关系题 1. M: I paid fifteen dollars for three books, I think they‘re too expensive. W: Expensive? You shouldn‘t have said so. Q: How did the woman feel about the book‘s price? ( B ) 2. W: Help me with this box of books, will you, Jack? M: Help you! Do you think I work here? Q: What does Jack mean? ( A ) 3. W: Should we get Ed a record for his birthday? M: Just because he‘s a composer doesn‘t mean he only likes to listen to music. Q: What did the man say? ( B ) 4. W: Will you go to attend Prof. Green‘s lectures next term? He‘s a very learned scholar. M: Well, I‘d rather not. It‘s said that his lectures are always boring and dull. Q: What‘s the man‘s response to the woman‘s remark? ( B ) 5. W: Has your group decided on a research project yet? M: No one‘s come up with anything yet. Q: What does the man say about the project? ( B ) 10、推理判断题 1. W: I think I ought to buy a bigger cabinet. M: All you really need to do is to put away the items you rarely use. Q: What does the man imply about the cabinet? ( D ) 用请下载www.docin.com/yongqing 2. W: There was such a heavy fog yesterday and today it‘s such a fine day. Why don‘t we climb the mountain? M: Oh, I would love to, Susan, but I have been sick for a few days and all that exercise may not be too good. Q: How did the man feel about Susan‘s invitation? ( A ) 3. M: David and I argued about a new building at the meeting. W: Oh? How did he give up his plan? Q: What is the conclusion from this conversation? ( A ) 4. W: Would you be kind enough to offer me a hand? M: Sure. Why not? Q: What did you learn from this conversation? ( C ) 5. W: Do you know if Nick has got enough money for his study? M: Yes. If Nick hadn‘t worked in the steel plant last summer, he wouldn‘t have earned enough money for his living expenses during his senior year. Q: What does the man mean about Nick? ( B ) 二、实战训练 Unit 1 Section A 1. W: That salesman wouldn‘t take on for an answer. I finally bought something just so he would leave. M: A lot of things wouldn‘t get sold if salesman accepted the first refusal. Q: What does the man say about salesman? ( A ) 2. M: Let‘s go to the dance at the Student Centre on Friday. W: I‘d like to, but I‘m going to a lecture. Thanks for asking me, though. Q: What is the woman going to do on Friday? ( C ) 3. M: Have you filled out your tax forms yet? W: Don‘t remind me of them! They‘re so confusing that I‘m discouraged before I start! Q: What emotion is the woman feeling? ( A ) 4. M: That is $3.25 on the meter, and a dollar for the suitcases. W: Okay. Here is five dollars, keep the change. Q: How much was the driver‘s tip? ( A ) 5. M: Would you like to have dinner with me tomorrow? W: Well, I don‘t know. Would you mind if I let you know tomorrow morning? Q: Did she accept the invitation to have dinner with the man? ( C ) 6. W: Why were you late for the meeting this afternoon? M: I just lost track of time. Q: What does the man mean? ( D ) 7. W: I‘ll go to Beijing to visit my uncle during the summer vacation. What about you? M: I‘ll spend some time in the library reading some novels and see some of my old friends. Q: What are they discussing? ( A ) 8. M: When will the train from Boston arrive? W: It was scheduled to be here at 9:15, but now it is expected to be 15 minutes behind schedule. Q: When is the train expected to arrive now? ( B ) 9. M: Yes, madam, what would you like today? W: Give me a pound and a half of pork roast and 3 pounds of hamburger, please. Q: Where is this conversation probably taking place? ( D ) 10. W: Are you ready to order now? M: No, I‘ve just finished. I‘m waiting for my change. Q: What does the man want? ( A ) Section B Passage One M: How are your driving lessons going? W: I‘m not taking lessons any more. M: You‘re not? I thought you wanted to get your driver‘s license this summer. Didn‘t you sign up 用请下载www.docin.com/yongqing for a 6-week course? W: Yes, but after two weeks the driving instructor asked me not to come back. The school refunded my money for the remaining 4 weeks. M: I‘ve never heard of a case like that. Didn‘t you and the driving instructor get along together? W: At first we got along fine, but he got a little unfriendly after I had the second accident. M: The second accident? W: And after that they asked me to stop coming. I really don‘t understand it, he seemed like such a nice man. M: I think I do. Questions 11 to 13 are based on the passage you have just heard. 11. What portion of her driving lessons did the woman complete? ( A ) 12. What kind of a driver can we assume the woman is? ( B ) 13. How many accidents did the woman have? ( B ) Passage Two The United States has long been known as a melting pot because so many people from all over the world make up its population. The first immigrants in American history came from England and the Netherlands. Immigrants from many other countries then began arriving and are still coming. The greatest numbers came in the years 1880 to 1914. Between 1820 and 1973, the United States admitted over 46 million immigrants. Most came from Europe, but many also came from Latin America, Asia, Australia and Canada. Questions 14 to 17 are based on the passage you have just heard. 14. What is the reason for calling the U.S. a ―melting pot‖? ( B ) 15. Where did the first immigrants to the U.S. come from? (D ) 16. When did the greatest numbers of immigrants reach the U.S.? ( D ) 17. What was the number of immigrants permitted to enter the U.S. between 1820 and 1973? (C) Passage Three The first man who cooked his food, instead of eating it raw, lived so long ago that we have no idea who he was or where he lived. We do know, however, that for thousands of years, food was always eaten cold and raw. Perhaps the first cooked food was heated accidentally by a forest fire or by the molten lava from an erupting volcano. No doubt, when people first tasted food that had been cooked, they found it tasted better. Even after this discovery, cooked food must have remained a rarity until man learned how to make and control fire. Early peoples who lived in hot regions could depend on the heat of the sun to cook their food. For example, in the desert areas of the south-western United States, the Indians cooked their food by placing it on a flat stone in the hot sun. They cooked pieces of meat and thin cakes of corn meal in this fashion. Questions 18 to 20 are based on the passage you have just heard. 18. When did the first man who cooked his food live? ( D ) 19. What is the author‘s attitude towards cooked food? ( A ) 20. How did the early Indians in the southwestern United States cook their food? ( A ) Section C 1. skill in using language 2. an American novelist 3. recreate 4. rhythms 5. influence 6. arrival 7. the colorful characters 8. measuring river depth 9. mastery of style 10. form the basis of his novels Unit 2 Section A 1. W: You must have said the right things during your interview. They‘re very selective about 用请下载www.docin.com/yongqing whom they hire. M: I know how to make a good impression. Q: Why was the man being interviewed? ( A ) 2. M: Can you lend me ten dollars until tomorrow? W: I just spent my last dollar for groceries. I wish you had asked me an hour ago. Q: where has the woman just been? ( B ) 3. M: How much are the pencils? W: 35 cents each or three for a dollar. Q: How much does one pencil cost? ( A ) 4. W: I hear you got lost on your way to the meeting at the famous hotel. M: I don‘t know how I did it. I‘ve been there a million times. Q: What does the man mean? ( B ) 5. M: Yesterday, we went to Jane‘s house to listen to records. W: I heard that she has over 100 jazz records. Is that true? Q: What music does Jane like? ( D ) 6. M: Do you think the price of our meals will go up next term? W: There‘s no question about it. Q: What does the woman expect? ( B ) 7. M: Let‘s go to the party at the Johnson‘s on Friday. W: I‘d like to, but I‘m going to the movies. Q: What is the woman going to do on Friday? ( C ) 8. M: Can you tell me how to get to the state highway? W: Keep going for about a mile. The entrance is coming up on your right. Q: What should the man do to get to the state highway? ( D ) 9. W: Bob, are you going straight home after school today? M: No, I have a class until one o‘clock and after that I‘m going to spend a couple of hours at the library before going home. Q: When is Bob going home this afternoon? ( B ) 10. W: It‘s lovely. I‘ll take it. M: Are you paying cash? W: No. I‘ll pay by check if that‘s okay. M: Yes, certainly. That‘s fine. Q: where does the conversation take place? ( B ) Section B Passage One Red Cross is an international organization which cares for people who are in need of help. A man in a Paris hospital who needs blood, and a family in India that lost their home, and a woman in Mexico who was injured in an earthquake may all be aided by Red Cross. Red Cross exists in almost every country around the globe. The world Red Cross organizations are sometimes called the Red Crescent, The Red Mogen David, The Sun, and the Red Lion. All of these agencies share a common goal of trying to help people in need. The idea of forming an organization to help the sick and wounded during a war started with Jean Dunant. In 1859, he observed how people were suffering on a battlefield in Italy. He wanted to help all the wounded people regardless of which side they were fighting for. The most important result of his work was an international treaty called the Geneva Convention. It protects prisoners of war, the sick and wounded and other citizens during a war. The American Red Cross was set up by Clara Barton in 1881, Today Red Cross in the United States provides a number of services for the public, such as helping people in need, teaching first aid, demonstrating water safety, and providing blood. Questions 11 to 13 are based on the passage you have just heard. 11. What is true about Red Cross? ( B ) 12. Who started the idea of Red Cross? ( B ) 13. What was responsible for the idea of forming Red Cross? ( A ) Passage Two In August 1970 the S.A. Vaal sailed from Cape Town to Southampton. William Honeywell was a passenger on the ship. He was twenty-eight years old and very strong. At four o‘clock one 用请下载www.docin.com/yongqing Friday morning Honeywell suddenly fell into the sea. Why did he fall into the sea? He did not know. Not one person saw him. The ship did not stop. It sailed into the distance. Honeywell thought, ―I have fallen into the sea so I must swim.‖ Captain Alan Freer looked for Honeywell. He was not on the ship. The Captain wanted to find his passenger so he went back. He sailed 225 kilometers. Eleven hours later the second officer saw Honeywell in the sea. The ship stopped and rescued him. Honeywell was alive but very tired and weak. Questions 14 to 17 are based on the passage you have just heard. 14. Who was William Honeywell? ( C ) 15. Why didn‘t the ship stop when he fell into the sea? ( B ) 16. How was he rescued? ( A ) 17. How long was he in the sea? ( B ) Passage Three Jean Paul Sartre--possibly the best known and most discussed modern French writer and thinker—was born in Paris in 1905. He was educated in Paris and later taught philosophy. In 1934 he spent a year in the French Institute in Berlin where he became acquainted with modern German philosophy. He played an active role in the Resistance during the war, and afterwards left the teaching profession. Since 1946 he has spent his time writing. Sartre is the founder of French existentialism. He is also known as a great novelist, seeming to weave an intricate tapestry of thoughts, feelings and incidents. Questions 18 to 20 are based on the passage you have just heard. 18. When was Sartre born? ( C ) 19. During World War Two, how did he distinguish himself? ( D ) 20. Sartre‘s philosophy is usually included in which of the following categories? ( B ) Section C Compound Dictation 1. symbolizes 2. capability 3. luxury 4. preoccupied 5. creatures 6. tremendous 7. belief 8. if you are kind enough to help others, especially the poor, money is a good thing for you 9. you can do much more for the benefit of people and your country, and it will add to your own happiness 10. only if you are generous can money be the source of your happiness Unit 3 Section A 1. M: Do you have any reason to believe that Sally is annoyed with you? W: Yes, today when I saw her, she wouldn‘t even stop to talk with me. Q: What does the woman believe? ( C ) 2. W: Do you need a lift home today? M: No, my wife is coming with the car late in the afternoon so we can do some shopping. Q: What does the woman want to know? ( D ) 3. W: Hi, Bob. This is Marcia speaking from the agency. Can you go on an interview tomorrow? I have a law office that‘s very interested in you. M: No, I‘m sorry, Marcia, but I‘ve planned to be at school all day tomorrow. How would Wednesday suit you? Q: What‘s Marcia‘s job? ( C ) 4. M: Hello, Mary, this is Dan Morrison from the office. I‘m calling to see how Tom is doing today. W: Oh, hello Mr. Morrison. The doctor said he‘d be able to go back to work tomorrow. M: Please tell him there‘s no hurry. I‘ve had Stam Johnson take his assignments for the time being. Q: Who is Dam Morrison? ( C ) 5. W: The total area of the earth is 196,940,000 square miles. M: The land area is 54,225,000 square miles. Q: what portion of the earth is land? ( A ) 6. W: I just want a wash and set (做头发), please. 用请下载www.docin.com/yongqing M: Fine. Why don‘t you let me blow it dry this time instead of putting it up in rollers? I think that you would like it that way. Q: Where did this conversation most probably take place? ( A ) 7. M: Would you tell Julia I am sorry I lost her tape? W: Hadn‘t you better tell her yourself? Q: What does the woman advise the man to do? ( B ) 8. W: Will you come to my novel reading next week? M: I‘ll be out of town then. Q: What does the man mean? ( B ) 9. M: Excuse me, can you tell me if this bus goes to Victoria Station? W: No, I‘m afraid I can‘t. I‘ve only been here a few days myself. Q: Why can‘t the woman give the man directions? ( C ) 10. M: Where can I get in touch with you while you are out of town? M: I‘ll stay at the Swan Hotel in California for a couple of days, and then I‘ll fly to Chicago to visit my brother. You can reach me there. Q: Where can the man contact the woman? ( B ) Section B Passage One Money! What do you ―see‖ when you hear this word? Do you picture a round metal coin? The money we see and use is made of paper or metal. But the money of long ago was not at all like the money we use today. Coins were not always made of metal. Soap was once money to the people of Mexico. Lumps of coal were used as coins bay the people of England. Stone money was used by the people on the island of Yap. With stone money a man on Yap could buy a wife! Even food was used as money. In Russia, ―coins‖ of cheese could be used to buy things. Bricks of tea leaves were used as money in Tibet. The tea leaves were first boiled in water. They were then pressed into hard brick shapes. Coins were not always round. The coins of old China were once in the shape of a knife. In another land coins were made in the shape of a fish. Square money is still used by the people of India. Money in the shape of rings and bracelets is also still seen in some parts of the world. Ring money is easy to carry. 11. With what money could a man on Yap buy a wife a long time ago? ( D ) 12. How were tea leaves made into money in Tibet? ( B ) 13. Which country is still using square money? ( C ) 14. What can be the best title for this passage? ( A ) Passage Two Switzerland is a land well known for her beautiful landscape and people generally tend to associate Switzerland with mountains and mountain scenery. But Switzerland is more than just a land of mountains. It is a country of highly intelligent, resourceful and hardworking people. The Swiss manufacture precision instruments such as watches, calculators and other specialized goods like typewriters, sewing machines, turbines and generators which are sold to other countries. They are also able to produce such goods which do not need huge amounts of raw materials. Most of the raw materials she uses in her industries are imported from other countries. As Switzerland has no sea coast, she has to pay heavily for imported raw materials which have to be sent over long distances and rocky land. For this reason the Swiss are interested only in producing precision products like watches, cameras, and other types of goods which make use of their skill and craftsmanship. The well-known Swiss watch industry is a good example of this. Small amounts of raw materials are needed but a great deal of kill and precision is needed to produce a little item with so many minute parts. Foodstuffs, especially milk products, are also processed and these are exported. Swiss cheese, butter, chocolate and condensed milk are popular in many countries. Swiss exports are sent by rail to neighboring countries. But, for her overseas business, Switzerland has special arrangements with foreign ports like Amsterdam, Genoa and Rotterdam through which Swiss goods are 用请下载www.docin.com/yongqing exported. 15. What do Swiss products require? ( D ) 16. Which of the following statements is true of Switzerland? ( C ) 17. How does Switzerland deal with her overseas business? ( A ) Passage Three thThe first postal service in North America Began in New England in the 17 century. All mail arriving in Massachusetts colony was sent to the home of appointed official in Boston. In turn, he would deliver the mail from Boston on horseback to its destination, receiving one penny for each good article of mail. Later in the century postal services were established between Philadelphia and Delaware. In 1691, the British crown appointed the first postmaster general to have charge of the mail for all the colonies in North America. Later Benjamin Franklin served as the postmaster general for the British government and then was made postmaster by the newly formed United States government. Franklin was responsible for establishing the United States postal system on a permanent basis. He increased the number of post offices, introduced the use of stagecoaches to carry mail, and started a package service system. Later, in the nineteenth century, as railroads and steam boats appeared, they were used to carry mail into the towns. Some communities, especially those out west, were far from the services of transportation to serve them, the post office developed a system called ―star routes‖. Private contractors were paid to deliver mail to the communities from railways by horse and wagon. The postal service, which was started over 3 centuries ago, has developed into an extensive government service with post offices in every city, town, and village in the United States. 18. What is the main subject of the lecture? ( B ) 19. For what does Benjamin Franklin deserve credit? ( D ) 20. What eventually happened to the postal service? ( B ) Section C Spot Dictation 1. The most likely place 2. a set of books with articles and pictures 3. Instead of going to the bookshelf 4. switch on the television set 5. with music and sound effects 6. about classical music 7. hear an example of music 8. the sound of its songs 9. As events occur 10. add and change information Unit 4 Section A 1. W: James, your article in the newsletter was excellent. M: I just wish they had published the whole thing. Q: What does James tell the woman? ( A ) 2. M: Could you give me some information about trains for Righton? W: The fast trains leave at ten minutes to and twenty minutes past each hour. You just missed the 11:50 train. It left 5 minutes ago Q: When will the nest train leave? ( D ) 3. W: Tom, did you hear about the house that the Whites bought in the country? M: Yes, and John said that they got a very good deal on it. Q: What do we learn from the conversation? ( A ) 4. M: Miss, could you bring me a rum and a cake, please? W: Certainly, but we have to wait until the captain has turned off the fasten-seatbelt sign. Q: Where does this conversation most likely take place? ( A ) 5. M: The car was total loss. Did you see it? W: Yes, and to think that all the three women and a baby were in that horrible crash without being hurt. 用请下载www.docin.com/yongqing Q: How many people were injured in the accident? ( C ) 6. W: Do you want a day course or an evening course? M: Well, it will have to be an evening course since I work during the day. Q: What are they talking about? ( C ) 7. M: I thought you were going to see your sister last weekend before you left New York? W: I intended to, but at the last minute she called and said this weekend was inconvenient so I stayed home altogether. Q: What did the woman do last weekend? ( A ) 8. M: If you don‘t have an account here, I can‘t cash your check. I‘m sorry, but that‘s the way it is. W: Well, thanks a lot! You‘re a big help! (愤怒的语气) Q: How does the woman feel? ( C ) 9. W: I‘m afraid Dr. Anderson won‘t be able to see you then. His appointment book is filled for the next couple of weeks. M: Oh, but I don‘t have to see him. I‘ll just leave my teeth in his office and he can look at them when he has time. They are false teeth! Q: What is Dr. Anderson? ( A ) 10. W: Jane told me that she was going to quit her job. I‘ll certainly be sorry to see her to. M: Oh, she always says that! I wouldn‘t buy her a going –away present if I were you. Q: What does the man think about Jane? ( C ) Section B Passage One So long as man believes in technological advancement, revolutions are inevitable, as the new concepts continuously replace the old in a never-ending process. In these last few decades, a revolution is under way. We are at the dawn of the ear of the smart machine—an ―information age‖ that will change forever the way people work, play, travel and even think. Just as the industrial revolution dramatically expanded the strength of man‘s muscles and the reach of his hand, so the smart-machine revolution will magnify the power of his brain. But unlike the industrial revolution, which depended on finite resources, such as iron and oil, the new information age will be fixed by a seemingly limitless resource: the inexhaustible supply of knowledge itself. Unfortunately, we know as the smart-machine revolution burst into life, it will eventually face its end, replaced by another revolution waiting in the wings. Questions 11 to 13 are based on the passage you have just heard. 11. What is the present revolution according t this passage? ( C ) 12. How did the industrial revolution help man according to this passage? ( A ) 13. What would happen to this new revolution eventually according to the speaker? ( C ) Passage Two A famous writer who was visiting Japan was invited to give a lecture at a university to a large group of students. As most of them could not understand spoken English, he had to have an interpreter. During his lecture he told an amusing story which went on for rather a long time. At last he stopped to allow the interpreter to translate it into Japanese, and was very surprised when the man did this in a few seconds, after which all the students laughed loudly. After the lecture, the writer thanked the interpreter for his good work and then said to him, ―Now please tell me how you translated that long story of mine into such a short Japanese one.‖ ―I didn‘t tell the story at all,‖ the interpreter answered with a smile. I just said, ―The honorable lecturer has just told a funny story. You will all laugh, please.‖ Questions 14 to 16 are based on the passage you have just heard. 14. What was the writer invited to do? ( D ) 15. What make the writer surprised? ( B ) 16. What did the interpreter do to make the students laugh? ( B ) Passage Three In 1789 the U. S. government passed a law which said that the land of the American Indians could never be taken from them without their agreement. One hundred years later, however, the Indians only had a very small part of the land that originally belonged to them. How did this great injustice occur? 用请下载www.docin.com/yongqing After 1812 white settlers began to move west across North America. At first, the settlers and the Indians lived in peace. However, the number of settlers increased greatly every year, and slowly the Indians began to see the white settlers as a danger to their survival. To feed themselves, the settlers killed more and more wild animals. The Indians, who depended on these animals for food, had to struggle against starvation. The settlers also brought with them many diseases which were common in white society, but which were new for the Indians. Great numbers of Indians became sick and died. Between 1843 and 1854 the Indian population in one are of the country went down from 100,000 to 30,000. More land was needed for the increasing number of white settlers. In Washington, the old respect for the rights of the Indians disappeared. The old promises to the Indians were broken; the federal government began to move groups of Indians from their original homelands to the other poorer parts of the country. Some Indians reacted angrily and violently to this treatment. They began to attack white settlers, and the Indian Wars began. For thirty years, until the late 1880‘s, different groups of Indians fought against the injustices of the white-man. Many Indians were killed, the survivors were moved from their homelands to different areas of the country. It was a terrible chapter in the history of a country that promised freedom and equality to everyone. Questions 17 to 20 are based on the passage you have just heard. 17. When did the Indians begin to see the white settlers as a danger to their survival? ( B ) 18. What happened to the Indians when more and more white men began to settle in the west? ( B ) 19. How did the Indians react to the government‘s decision? ( D ) 20. What is the speaker‘s attitude towards the treatment to the Indians by the government? ( A ) Section C Compound Dictation 1. perform 2. entertaining 3. valuable 4. property 5. hunting 6. messages 7. same 8. smart 9. However, some dogs will always attack the postman who comes to deliver letters. One explanation for this behavior is that, although the postman comes to the house often, he never enters the house. 1. These homing pigeons begin their training when they are about four weeks old. After a few weeks they can begin flying and carrying messages. Unit 9 Section A 1. W: Steve asked if he could get a ride with us to the beach. M: Well, I‘ll speak to him. Whether or not we‘ll have room is still not clear. Q: what does the man mean? ( D ) 2. W: I can‘t decide what color to paint my room. M: What about white? It will make the room brighter. Q: What does the man suggest? ( C ) 3. W: Good afternoon, sir. Have you anything to declare? M: No. I have only been abroad for a few days. That‘s why I came through the door where the notice says ―Nothing to Declare.‖ Q: where are the two speakers? ( C ) 4. M: Ranger, thanks for taking the time to give us those tips. W: Not at all. Watch out for the bears! Don‘t leave any food around, or you‘re liable to have an unwanted visitor. Q: What are they talking about? ( B ) 5. M: I hope that the bank will be open. W: The sign says: nine A. M. to five P. M. weekdays nine A. M. to noon Saturdays close Sundays. 用请下载www.docin.com/yongqing Q: When will the bank be open on Saturday? ( D ) 6. W: It looks like the English course is going to be a lot of work. M: Didn‘t you see the reading list is enormous? Q: What did they think of the English course? ( C ) 7. M: Two gallons of gas please, and the ordinary kind. W: Why don‘t you choose the leadless? It‘s of superior quality. Q: What do we learn from the conversation? ( D ) 8. W: I‘d like to go to the art gallery. Do you know what time it‘s open? M: Yes, it‘s open from 9 until 5. But it has an hour‘s break. Q: For how many hours is the gallery open? ( B ) 9. W: Can‘t we slow down? We ‗re going beyond the speed limit, we‘ll be fined. M: Slow down? Don‘t you realize how late we are going to be? Q: What does the man mean? ( D ) 10. W: I am sorry I am having trouble dreading my notes. Did you say three lunches in that first paragraph? M: No. I said free lunches, the children don‘t have to pay for them. And there are many lunches involved, you know, a lot more than three. Q: what did the man say about the lunches? ( B ) Section B Passage One He usually sat behind me in class but seldom talked. Perhaps he feared to make mistakes. His worry was not reasonable because he always spoke words of wisdom. One day, there he was leaning against a tree alone in front of my dormitory. Could he be waiting for me? I looked directly at him but he didn‘t notice. My heart beat fast. Was he waiting for someone else? It was Wednesday. The professor asked him to distribute our homework. He called out each English name of my classmates and returned the assignment. Then he came to my desk and held out my paper. He looked at me and when he said my name—my Chinese name—it sounded more beautiful than anyone else had ever pronounced it. For a time we took a firm hold of the paper. A force seemed to pass through the paper between us. That day he had a lot to say and so did I. All the world seemed sunny and magnificent. He hurriedly left the classroom when the bell rang. I hastened to follow him and at the doorway I froze. My notebook fell. He took my notebook and put it under his arm as he brushed my shoulder, pushing me slightly toward the outside door: ―I know where you live. I will walk you home.‖ Questions 11 to 13 are based on the passage you have just heard 11. What‘s the probable relationship between the boy and the author? ( B ) 12. From the first paragraph we may find the boy was ______________. ( A ) 13. Whom was the boy waiting for leaning against a tree one day? ( D ) Passage Two After I finished middle school, I went to the countryside. In my spare time, I studied English and some other subjects. I wanted very much to become a university student in the future. In 1976, I went back to my hometown and began to work at a factory. One Monday morning, I got up at six o‘clock. Outside, it was still dark, and there was snow in the street. At half past six, I got on my bicycle and began the long journey to the factory. I arrived at the factory at five minutes to eight. Most of my comrades were already there. As I walked through the long room, they looked up from their machines and smiled at me. I sat down in front of my machine and turned it on. After a few minutes my leader came up to me. ―Xiao Zhang,‖ he said. ―will you help us again? Will you translate the technical manual for us?‖ I smiled. ―Certainly.‖ I replied. Then I began to work. Questions 14 to 16 are based on the passage you have just heard. 14. What was the day in the story? ( B ) 15. How long did Xiao Zhang‘s journey take from her home to the factory? ( D ) 16. When did Xiao Zhang arrive at the factory? ( C ) Passage Three 用请下载www.docin.com/yongqing The African ancestors of today‘s black Americans were brought to the U. S. as slaves in the seventeenth, eighteenth, and nineteenth centuries. They worked on farms, especially the large farms in the southern states. Slowly they became a necessary part of the economic system of the South. Slaves did not have the rights of people; according to the law. They were ―things‖ which belonged to the person who bought them. They had to obey the orders of their owners without question. They were not allowed to learn to read; their owners feared that educated slaves would begin to think about the injustice of the system and would learn to struggle for their freedom. Slaves had to work long hours in extremely unhealthy conditions. Their owners had complete power over them. They could be bought and sold like animals. At the slave markets, black children were separated from their parents and never saw them again. Slave owners had the right to punish severely any slave who broke rules or protested against the system. Slaves were often beaten brutally by their owners or killed. After the Civil War, one free slave reported that his owner killed an older slave who was teaching him to read. There was a law against brutality to slaves, so in theory any owner who treated a slave badly could be punished. In practice, however, the law meant nothing. Another law said that slaves could not give evidence against white people, so very few owners were ever punished for their brutality. Opposition to slavery began very early in the history of the U. S.—in 1671—but little progress was made until the beginning of the nineteenth century. By 1804 slavery was illegal in the northern states. But it continued, it even grew, in the southern states, which depended on cotton for their economic health. Slavery ended in the South only after the Civil War. For blacks, however, the end of slavery was only a beginning, the late beginning of a long and difficult struggle for true justice and equality. Questions 17 to 20 are based on the passage you have just heard. 17. According to the passage, why couldn‘t slaves learn to read? ( B ) 18. What happened to slaves who broke the rules of the system? ( A ) 19. According to the passage, which of the following is true? ( C ) 20. What can we infer from the passage? ( C ) Section C Spot dictation 1. they do this rather slowly 2. that have taken place 3. about 1,500 years ago 4. the sound system has changed 5. A speaker‘s knowledge of a language 6. by the use of grammar 7. Any part in grammar may change 8. what words and forms exist 9. constitute the vocabulary 10. changes of spelling Unit 10 Section A 1. W: Where did you and Sue go on your vacation? M: We spent three days in Scotland, one week in Spain, and five days in Switzerland. Q: Which of the following countries was not mentioned? ( B ) 2. W: Your Hi-Fi is so good that it sounds as if it were playing in a concert hall. M: Yes, but my brother has a model which is superior to mine, and you can hardly imagine how it sounds. Q: What do we learn from the conversation? ( D ) 3. M: The election will be very close, according to the polls. W: Remember what happened in 1948? At first we all thought that Dewey would win, but , when all the votes were counted, it was Truman instead. Q: What is being discussed? ( B ) 4. W: Excuse me. My radio doesn‘t seem to be working. M: Okay, let me have a look at it. Why don‘t you call me in a week? 用请下载www.docin.com/yongqing Q: What does the woman most likely want the man to do? ( C ) 5. W: I can‘t understand why Mary didn‘t at least telephone me. M: She did! Didn‘t I tell you? She said she was sorry she couldn‘t come for dinner, but hoped to be able to next time she came to town. Q: What did the man say about Mary? ( D ) 6. M: Now then, Mrs. Charlton, how long has this tooth been giving you trouble? W: Well, I had it filled last year and it was all right until last Sunday. Then it started hurting again. Q: what do we know about one of Mrs. Charlton‘s teeth? ( B ) 7. M: Do you want airmail or regular? W: Airmail. They‘re for overseas, France. Q: What are they talking about? ( C ) 8. W: Oh, no, it‘s five o‘clock already and I haven‘t finished typing these letters. M: Don‘t worry, that clock is half an hour fast, you still have time to do them. Q: When does this conversation take place? ( A ) 9. M: What do you think of Professor Miller‘s lessons? W: Well, his lessons are interesting enough, but I think he could choose more appropriate questions for the tests. Q: What does she mean? ( A ) 10. W: How do you get to Route 22 from the airport? M: Take the airport exit and immediately get onto Route 1 north. Then right away take the exit for Routes 21 and 22, and follow the signs for 22. On account of the construction, there are several very sharp turns, so be careful. Q: Why should the woman be careful? ( D ) Section B Passage One In England, Christmas is the most important of all the ―Bank Holidays‖ in the year. Two important things, apart from its religious significance, help to set this holiday apart from all other: the custom of giving gifts and the habit of spending it with the family. The custom of giving presents to one‘s family and friends is a very pleasant one so long as one remembers that it is the spirit behind the gift which matters most and not the gift itself. How good it is at Christmas to return to the family and meet parents, grandparents and as many aunts, uncles and cousins as can be accommodated. Without twentieth-century means of transport, many families would be denied the Christmas reunion. Christmas Day is spent quietly at home. The excitement of all the presents is hardly over before it is time for the traditional Christmas dinner: turkey, duck or chicken with rich fruity Christmas pudding afterwards. At tea-time, the crackers are pulled out. The evening is spent in games, merriment and more eating and drinking. There is always Boxing Day, the Bank Holiday after Christmas Day, on which to recover, if all the excitement and food have proved a little too much. Questions 11 to 13 are based on the passage you have just heard. 11. Why does Christmas differ from other holidays? ( D ) 12. Whom do people give presents to on Christmas Day? ( D ) 13. What is Boxing Day for? ( A ) Passage Two At sixteen Henry Vincent was separated from his family as a result of the war. He wandered aimlessly from one country to another before finally settling down in Australia, where he trained as an electronics engineer. He established his own business but it called for so much work that marriage was out of the question. His retirement suddenly made him realize how lonely he was and he decided to take up a hobby. With his interest in electronics, being an amateur radio operator seemed a natural choice. He installed his own equipment and obtained a license and his call sign, which is the set of letters and numbers used to identify oneself when making radio contact with other radio amateur operators all over the world. Soon Henry had a great many contacts in far-off places. One in particular was a man in 用请下载www.docin.com/yongqing California with whom he had much in common. One night the man in California happened to mention the village in Europe he had come from. Suddenly, Henry realized that this man was in fact his younger brother, Peter. At first, the two brothers were at a loss for words but then little by little they filled in the details of their past lives and not long afterwards Henry Vincent flew to California to be reunited with his brother. Questions 14 to 16 are based on the passage you have just heard. 14. What kind of training did he get when he was in Australia? ( A ) 15. What hobby did he take up after his retirement? ( D ) 16. Who is the person he had much in common with? ( C ) Passage Three Plants seem to know which way is up and which way down; furthermore, they seem to know right from left. If a cutting from a Lombartdy poplar is kept alive, new shoots will grow from the end that grew uppermost in the tree. There is no visible difference between the top and the bottom of the living stick, even under a microscope. Even so, the stick will not send out shoots from the end it views as bottom even if this end happens to be on top! Scientists studying this subject further split their cutting lengthwise. To their surprise, they made another interesting discovery. A good many more buds grew on the right-hand side of the split surface than on the left. They split the sticks again and found that the buds again grew on the right side. The results of the entire study showed a 60-40 preference for the right side, proving plants are basically ―right-handed.‖ Questions 17 to 20 are based on the passage you have just heard. 17. What did scientists examining the ends of a fresh cutting find? ( C ) 18. What will the buds do if a cutting is placed upside down in the laboratory? ( A ) 19. What happens to the cutting? ( B ) 20. What is the best title for this passage? ( A ) Section C Compound Dictation 1. agricultural 2. rely 3. vary 4. economic 5. Consequently 6. raise 7. quantities 8. In Florida, income from tourism accounts for more than half the state‘s annual income during the winter months. 9. the U. S. military and space programs also contribute greatly to the economy of the region. 10.In addition, there are more army, navy, air force and marine bases in the south than in any other region of the U. S. 2003年1月大学英语四级听力材料 1-10 BDDBA DBACB Section A: 1. M: Mary, would you join me for dinner tonight? W: You treated me last weekend. Now it‘s my turn. Shall we try something Italian? Q: What do we learn from the conversation? 2. W: Good afternoon. I‘m calling to inquire about the four – bedroom house you advertised in the newspaper. M: I‘m sorry, but it‘s already sold. Q: What do we learn about the house from the conversation? 3. W: john, what are you doing on your computer. Don‘t you remember your promise? M: This is not a game. It‘s only a crossword puzzle. They help increase my vocabulary. Q: What is the probable relationship between the speakers? 4. M: Do you still keep in touch with your parents regularly after all these years? W: Yes, of course. I call them at weekends when the rates are down 50%. Q: What do we learn about the woman from the conversation? 5. M: Hurry, there‘s a bus coming. 用请下载www.docin.com/yongqing W: Why run? There will be another one in two or three minutes. Q: What does the woman mean? 6. M: Wow, that‘s a big assignment we‘ve got for the English class. W: Well, it‘s not as bad as it looks. It isn‘t due until Thursday morning. Q: What does the woman mean? 7. W: Hello, is that Steve? I‘m stuck in a traffic jam. I‘m afraid I can‘t make it before 7 o‘clock. M: Never mind. I‘ll be here waiting for you. Q: What do we learn from the conversation? 8. M: you really seem to enjoy your literature class. W: You are right. It has opened a new world for me. I‘m exposed to the thoughts of some of the world‘s best writers. I‘ve never read so much in my life. Q: What does the woman mean? 9. W: Listen to me, Joe. The exam is already a thing of the past. Just forget about it. M: That‘s easier said than done. Q: What can we infer from the conversation? 10. M: I hear you drive a long way to work every day. W: Oh, yes. It‘s about 60 miles, but it doesn‘t seem that far. The road is not bad, and there‘s not much traffic. Q: how does the woman feel about driving to work? Section B: Compound Dictation It‘s difficult to imagine the sea ever running out of fish. It‘s so vast, so deep, so (S1). mysterious Unfortunately, it‘s not bottomless. Over-fishing, (S2) coupled with destructive fishing practices, is killing off the fish and (S3) ruining their environment. Destroy the fish, and you destroy the fishermen‘s means of living. At least 60 (S4) percent of the world‘s commercially important fish (S5) species are already over-fished, or fished to the limit. As a result, governments have had to close down some areas of sea to commercial fishing. Big, high-tech fleets (S6) ensure that everything in their path is pulled out of water. Anything too small, or the wrong thing, is thrown back either dead or dying. That‘s an (S7) average of more than 20 million metric tons every year. (S8) When you consider that equals a quarter of the world catch, you begin to see the size of the problem. In some parts of the world, for every kilogram of prawns (对虾) caught, up to 15 kilograms of unsuspecting fish and other marine wildlife die, simply for being in the wrong place at the wrong time. True, (S9) Some countries are beginning to deal with this problem, but it's vital we find rational ways of fishing. , before every ocean becomes a dead sea. (S10) It would make sense to give the fish enough time to recover grow to full size and reproduce. , then catch them in a way that doesn‘t kill other innocent sea life. 用请下载www.docin.com/yongqing 阅读技巧与训练 Dong Wei Cai Huijian I 阅读理解测试的能力要求 阅读理解在大学英语四、六级测试中占总分的40%,因此,这部分成绩的好坏在很大 程度上影响到总成绩的高低。无论是四级还是六级,阅读理解在题型、题数、考试时间、 计分等方面都一样,只是要求不同。这部分通常包括4篇短文。四级的总阅读量在工作上 1200字左右。根据大学英语四级考试大纲,阅读理解部分文章的题材、体裁、语言难度以 及所测试的能力要求是: 题材广泛,包括人物传记、社会、文化、日常知识和科普常识等;体裁多样,包括叙 述文、说明文、议论文等;语言难度中等,要求掌握所读材料的主旨和大意;了解说明主 旨和大意的事实及细节;既理解字面的意思,也能根据所读材料进行一定的判断和推论; 既理解个别句子的意义,也能理解上下文的逻辑关系。 从考试大纲的要求可以看出阅读理解部分的目的是测试考生通过阅读获取信息的能 力。 既要求准确,也要求有一定的速度。这个要求并不低。要提高阅读水平,提高阅读理 解测试的成绩,除了进行一些应试技巧训练外,更重要的是扎扎实实地打好语言知识基本 功,训练阅读技能,提高阅读速度。忽视基本功的片面训练,只从“技巧”出发,不但学 不到技巧,阅读水平也难以提高。 具体说来,阅读理解的能力包括三个方面,语言知识、背景知识以及阅读技能。从某 种意义上说,阅读理解是一种综合性考试:它既是对语言因素包括词汇、语法结构、篇章 知识和体裁等方面的考试,也是对非语言因素如考生的知识面的考试,同时,也是对运用 这些知识的熟练程度的考试。 1.语言知识方面的能力 语言知识首先是指词法、句法和篇章结构这些方面的知识。 Example1 Computer technology enabled the researchers to obtain precise measurements of the volume of the front and side sections of the brain, which relate to intellect and emotion, and determine the human character. (The rear section of the brain, which controls functions like eating and breathing, does not contract with age, and one can continue living without intellectual or emotional faculties (功能).) Contraction of front and side parts –– as cells die off –– was observed in some subjects in their thirties, but it was still not evident in some sixty- and seventy-year-olds The word ―subjects‖ in paragraph 2 means _______. A) something to be considered B) branches of knowledge studied C) persons chosen to be studied in an experiment D) any member of a state except the supreme ruler 在阅读理解中,把握篇章的能力是至关重要的。是否具有把握篇章的能力可以从几个 方面来看:(1) 考生对不同文体的特点的了解;(2) 考生全面把握文章的能力(文章的观点、 思路、结构、落脚点等)。 熟悉不同文章的结构特点,有利于把握文章的目的和内容的具体分布。阅读理解的文章 因体裁的不同,结构也有其各自的特点。 用请下载www.docin.com/yongqing 议论文是一种剖析事物、论述事理、发表意见、提出主张的文体。议论文主要包括论点 和论据两大要素。论点是作者提出讨论的问题。文章的论点往往以破题句(topic sentence) 的 形式出现,这个句子表述了段落主旨大意,多位于段首或段尾,因此认真阅读文章中每段中 的第一句和结尾句是十分必要的。阅读理解中有关主题思想的问题在议论文中就是针对论点 而设的题。我们可以把议论文的结构特点归纳如下: 主题段 提出本文的主题(论点) 议论文 论证段 论证文章的主题 „„ 结论段 得出结论或总结中心思想 说明文是用说明的表达方式来解释事物、阐述事理的文章。其目标是通过介绍事物的形 状、构造、用途或情况,来说明事物的性质、特征、成因、结构等,客观地向读者提供知识、 传授技能。说明文的结构特点如下: 开篇段 提出本文将要说明的事物 说明文 展开段 就事物某一方面进行阐述(说明) 展开段 就事物另一方面进行阐述(说明) „„ 文体的不同还体现在谴词造句上。叙述文在用词上含蓄,寓意深刻,词语 感情色彩浓。而在说明文与议论文中,因文章的目的关系,往往会使用不同的信号词,如: for example / because / however / but / consequently / therefore / so / in contrast / on the contrary / compared with / more…than / as…as等等。这些信号词对理解文章信息的布局,思维的发展 和变化有很大的帮助。 三种文体在四级考试中出现的频率从高到低依次为议论文、说明文和叙述文。 Example 2 More than 30,000 drivers and front seat passengers are killed or seriously injured each year. At a speed of only 30 miles per hour it is the same as falling from a third floor window. Wearing a seat belt saves lives; it reduces your chance of death or serious injury by more than half. Therefore drivers or front seat passengers over 14 in most vehicles must wear a seat belt. If you do not, you could be fined up to ,50. It will not be up to the drivers to make sure you wear your belt. But it will be the driver‘s responsibility to make sure that children under 14 do not ride in the front unless they are wearing a seat belt of some kind. However, you do not have to wear a seat belt if you are reversing your vehicle or you are making a local delivery or collection using a special vehicle; or if you have a valid medical certificate which excuses you from wearing it. Make sure these circumstances apply to you before you decide not to wear your seat belt. Remember you may be taken to court for not doing so, and you may be fined if you cannot prove to the court that you have been excused from wearing it. 1. This text is taken from ______. A) a medical magazine B) a police report C) a legal document D) a government information booklet 2. Wearing a seat belt in a vehicle ______. A) reduces road accidents by more than half 用请下载www.docin.com/yongqing B) saves lives while driving at a speed up to 30 miles per hour C) reduces the death rate in traffic accidents D) saves more than 15,000 lives each year 3. It is the driver‘s responsibility to _____. A) make the front seat passenger wear a seat belt B) make the front seat children under 14 wear a seat belt C) stop children riding in the front seat D) wear a seat belt each time he drives 4. According to the text, which of the following people riding in the front does not have to wear a seat belt? A) Someone who is backing into a parking space. B) Someone who is picking up the children from the local school C) Someone whose job is delivering mails or newspapers. D) Someone who is under 14. 5. For some people, it may be better ______. A) to wear a seat belt for health reasons B) not to wear a seat belt for health reasons C) to get a valid medical certificate before wearing a seat belt D) to pay a fine rather than wear a seat belt 2.非语言因素的能力 影响阅读理解效果除了词法、句法和章法这些语言因素以外还有一些非语言因素, 如知识面和智力情况;此外,学生的阅读习惯和心理素质也影响着阅读的质量。 知识面反映在阅读理解上也就是题材问题。考生在平时要注意阅读各种题材的文 章,努力拓宽自己的知识面,消除不熟悉的题材所带来的陌生感。 Example 3 Rudolf, the Red-nosed Reindeer(驯鹿), has canceled his trip to Britain this Christmas. Instead, Father Christmas will have to rely on horses and husky(爱斯基摩) dogs to pull his sleigh this year in case reindeer spread highly infectious foot-and-mouth diseases which has plagued(困绕) Britain this year. ―Reindeer are cloven-hoofed(偶蹄的) and so are capable of catching and carrying foot-and-mouth. For that reason they are subject to the same restrictions as sheep and cattle,‖ said an environment Department spokesman. So top London store Harrods will use a horse and carriage this year for Santa Claus. ―Rudolf has to stay in Lapland. This is the first time in living memory Harrods has not used a reindeer,‖ a spokesman for the store said. In the southern English port of Southampton, Santa is relying on a team of husky dogs to pull him along. It all came as a bitter blow to the Cairngorm Reindeer Center in Scotland which is home to Britain‘s only herd of reindeer, numbering some 150. ―This has hit us extremely hard,‖ a spokesman said. 本利并非来自于CET4考试原题,只作为一例来证实背景知识的重要性。 对文中Rudolf, Santa Claus, Lapland 和foot-and-mouth 的认识程度可能会直接影响到读者对 文章内容的理解深度。如果事先就知道Rudolf是传说中圣诞老人的一头名叫的红鼻子驯鹿, Santa Claus指的是圣诞老人,Lapland是传说中圣诞老人的家乡,而foot-and-mouth指的是 口蹄疫,那么在阅读中就不会有障碍了。 一些考生养成了不良的阅读习惯,如逐词阅读(把句子分割成孤立的单词而不是意 用请下载www.docin.com/yongqing 群)、回读、声读、,译读、进行不必要的语法分析等,或有生词恐惧症,再加上平时缺乏限时阅读训练,阅读的质量和速度就难以保证。 总地说来,正确的阅读方法是快读、查读和细读相结合。快读(Skimming) 的目的是用浏览全文的方法了解文章的大意和主题思想,并对文章的结构有个总的概念。顾名思义,快读时,速度要快,精力必须十分集中,不必去记忆细节;遇到个别生词及难懂的词句,均应略过,以求对全文总体意思的了解。为了更好地抓住全文的中心思想,在快读时要留心某些反复出现的词语,这些词语往往与文章的主题有关。还应注意文章的开始段和结束段及文章中每一段的首尾句,因为它们往往是对文章内容的最好概括。 查读(Scanning)时,目光要自上而下,一目数行地搜寻与答题内容有关的词句,与此无关的内容要很快地掠过。细读指在找到文章中的有关范围以后,在此范围内逐句阅读,进行较深刻、较准确的理解和掌握。在正式考试中,阅读理解部分可参照下列步骤来进行,即可先用“快读”的方法浏览全文,以了解中心思想及大意。在此基础上,可根据要回答的问题用“查读”的方法,查到文章中与答题有关的范围后, 用“细读”的方法来确定答案。 II阅读理解的题型与解题技巧 阅读理解测试的范围主要包括以下几个方面: 1.主旨和大意,在试卷上出现的形式便是主旨题; 2.说明主旨和大意的事实和细节,即细节题; 3.对关键词语或单句语义的把握,及其在上下文中的特别含义即词汇与语意题; 4.根据所读材料进行一定的判断和推论,即推断题; 各类型题目在历年真题中出现的频率的从高到低依次为细节题、推理判断题、主旨题、词汇与语意题: 阅读技巧,换句话说,就是使考生更好地将掌握的语言知识水平体现出来的一种本领。在学生中有时出现两个极端。有的考生语言知识掌握甚好,可是阅读理解的能力却很差。究其原因,大凡平时只注意语言知识的学习而忽略了阅读技巧的训练。另一种学生急功近利,带着投机的心理学习,以为阅读理解就是阅读技巧的考试,于是,对技巧死钻牛角尖,而在语言知识方面下苦功。如果要在阅读理解上真正体现出自己的语言水平,那么,两者缺一不可。 PART ONE HOW TO GRASP THE MAIN IDEA I主旨题的目的: 检验学生是否抓住了篇章所表达的中心思想,考察学生的归纳能力,从整体上理解文章的能力,或把握某个段落的主旨的能力。总体型问题的答案往往隐藏于文章的主体构架里。 在针对主旨大意的阅读理解题中,大多数针对段落(或短文)的主题(subject),主题思想(main idea),题目(title)或目的(purpose)拟题。题型颇多,用词不尽相同,但所针对的都是段落(或短文)的主旨大意。 II用于检查对主题思想的掌握的问题(The comprehension questions directed towards the main idea(the question patterns): 1. The main idea(central thought ) of the passage is 2. What is the author‘s main point? 3. Which of the following sentences best expresses the main idea? 4. The topic sentence of this passage is 5. What the passage tells us would be summarized by the sentence 6. The best summary of the passage is that 7. The main purpose of the passage is 8. What is the author‘s main purpose in the passage? 9. The primary objective of the passage is 用请下载www.docin.com/yongqing 10. The title of this passage would be 11. The title that best expresses the main idea of the passage? 12. Which of the following is the best title for the passage? 13. The (main) subject of the passage is 14. Which of the following best the theme of the passage? 15. What topic is treated in this passage? 16. The passage states (illustrates, discusses, deals with, is concerned about, describes…) 主旨题可归纳为两类:总结文章中心大意类(Key words: main idea, purpose, conclusion)和 标题类(Key words: subject, theme, title, topic) III解题技巧: 1 辨认主题句(To grasp the main idea by identifying the topic sentence) 辨认主题句是获取段落主旨大意的一个有效的方法。凡表述段落主旨大意的句子叫主题 句。这种句子一般具有三个特点:1)表述的意思比较概括,相对其他句子来看,这种概括 性更为明显;2)句子结构较简单,多数都不采用长难句的形式;3)段落中其它的句子必定 是用来解释、支撑或扩展主题句所表述的主题思想(main idea)。 Just remember this: the topic sentence should state the main idea of the paragraph, and every idea, every detail, in the paragraph should relate to the idea set forth in the topic sentence. 主题句多出现在首尾,有时出现在中间,此时文章开首部分是topic introducer, 这时 特别需要注意but, however之类的关联词,因为他们后面一般就是作者真正想要表达的观点, 即主题句。 Example 2 给无主题句的段落构思一个标题(To grasp the main idea by writing a mental headline or title). A quick way to do this is: 1) Decide the person, place, or thing which is the subject of the reading passage. The subject can be spmething abstract, suxh as an idea. It can be a process, or something in motion, for which no single-word synonym exists. This person, place,thing,idea, or process then becomes the subject of the headline or title. 2) Decide the most important thing that is being said about the subject. Either the subject must be doing something,or something is being done to it. This action becomes the verb of the mental headline or title. Example 1(CET4 1998/1 Passage 2) A good modern newspaper is an extraordinary piece of reading. It is remarkable first for what it contains: the range of news from local crime to international politics, from sports to business to fashion to science, and the range of comment and special features (特写) as well, from editorial page to feature articles and interviews to criticism of books, art, theatre, and music. A newspaper is even more remarkable for the way one reads it: never completely, never straight through, but always by jumping from here to there, in and out, glancing at one piece, reading another article all the way through, reading just a few paragraphs of the next. A good modern newspaper offers a variety together in one place is its topicality (时事性), its immediate relation to what is happening in your world and your locality now. But immediacy and the speed of production that goes with it mean also that much of what appears in a newspaper has not more than transient (短暂的) value. For all these reasons, no two people really read the same paper: what each person does is to put together, out of the pages of that day‘s paper, his own selection and sequence, his own newspaper. For all these reasons, reading newspapers efficiently, which means getting what you want from them without missing things you need but without wasting time, demands skill and self-awareness as you modify and apply the techniques of reading. The best title for this passage would be ― ‖ . A) The Importance of Newspaper Topicality 用请下载www.docin.com/yongqing B) The Characteristics of a Good Newspaper C) The Variety of a Good Newspaper D) Some Suggestions on How to Read a newspaper 这一段文章是用演绎法写成的议论文。段首便提出主题句;当今的好报纸是一种特殊的 阅读材料。为什么这么说呢,接下来便是论证。首先,它的内容多种多样;再者,人们读报 的兴趣点不同。因此,没有两个人读的是真正“同”一份报纸。就因为报纸是如此的特殊, 所以要想从报纸上获取有效的信息而又不浪费时间,读报就需要技巧。综合看四个答案,A)、 C)和D)都太片面,不能统领全文;而只有B)才能涵盖全文的内容,是正确答案。 做这类选择标题的题目时,一个最简单的办法就是考生自己把所给的四个题目分别当成 作文题目,想像一下你会写到些什么内容。必定有一个标题能让你写出与文中内容大致重合 的内容,那么它就是能统领文章的标题。 PART TWO HOW TO FIND SPECIFIC DETAILS I 细节题的目的 一篇文章除了有主题思想以外,还必须有一些具体的内容用来说明、理解、证明或分析文章的主题,这些具体的内容就是文章的细节。在阅读理解测试中,大多数考题是针对这些细节而设计的,目的在于考查考生对组成文章的主体部分的理解程度。因此,在掌握文章主旨大意的基础上,还要抓住阐述和发展主题思想的主要事实或有关细节,领会文章的内在联系。文章主旨的事实或细节是主题的补充说明,也是加强主题的要素。一般情况下,说明主题的排列方式采用:因果关系、对比关系、简单列举、时间顺序或依据事件之重要性等。 II 细节类问题的命题方式有以下几种: 1. All the following statements are false EXCEPT… 2. Which of the following statements is false/not true, according to the passage? 3. All the following statements are true EXCEPT… 4. The author/writer mentions all of the items listed below EXCEPT… 5. In this passage, we find support for all of the following statements EXCEPT… 6. Which of the following statements is correct/true, according to the passage? 7. The statements made by the writer/author are based on evidence/example/fact… 8. The author/passage states/inform us that ______. 9. What does the passage say about …? 10. According to the passage, when(where, why, how, who,etc.)… 11. The reason for … is that______. 12. The cause for … is that______. 13. The report is given by______. 14. The difference between … and … lies in that______. 15. (something will happen) if/ when ______. III 技巧 为了准确地捕捉到辅助、支撑主题思想所需要的信息,在阅读中应注意: 1. 中心内容和具体内容的关系是辩证的,只有抓住文章的主要内容才能确定有关具体内 容的准确位置。同样,如果对文章具体内容有较为全面的理解,就能更好地判明文章的主 题。 2. 文章的细节,即某个具体事实并不是孤立的,它总要与其他事实前后呼应,相同性质 的事实总会在一起出现。 3. 看懂文章的组织结构对迅速捕捉到所需要的信息也同样起着重要的作用。 用请下载www.docin.com/yongqing 细节性问题是关于Supporting Details类的问题,通过Skimming找出主题后,应进一步掌握阐述 和发展主题的主要事实,或按要求找出特定细节。 在回答此类问题时,应采用查读法(Scanning), 因为这些具体内容(Detai1s)是用来说明、论证或分析文章中心的。这类题目常以"WH-"形式来提 问,如who, what, when, where, why及how等形式。这些问题的表达常不采用文章中的原 话提问,而是使用同义词语等,因此,在选择答案前应首先看准题干,看清问题所问究竟;然后, 在查读时注意寻找与题目相关的关键词语;最后,在充分理解原文、原题的基础上确定正确答案。 当然,这类细节性问题所涉及的面是很广的。有的涉及数字计算,如问时间、距离、次数、数 量等,认真计算后方可选定正确答案;有的涉及正误判断,要先看选项,根据选项提供的线索,寻 视文中相应部分,最后在题中选出肯定答案;还有的寻问事实、原因、结果、目的等。总之,做细 节题切忌通过自己对某类知识的主观了解和认识做出想像判断,一定要紧扣文章内容,不可随心所 欲。 针对阐述主旨的事实和有关细节提问的题型有一个特点,即:所提问题一般可以在文章中直接 或间接找到答案。故解题原则是:提倡―本本主义‖,以短文为唯一的解题依据,决不可想当然地跟 着感觉走。 Example 1: Just seven years ago, the Jarvik-7 artificial heart was being cheered as the model of human creativeness. The sight of Barney Clark -alive and conscious after trading his diseased heart for metal-and-plastic pump -convinced the press, the public and many doctors that the future had arrived. It hadn't. After monitoring production of the Jarvik-7, and reviewing its effects on the 150 or so patients (most of whom got the device as a temporary measure) the U. S. Food and Drug Administration concluded that the machine was doing more to endanger lives than to save them. Last week the agency cancelled its earlier approval, effectively banning ( 禁止 ) the device . The recall may hurt Symbion Inc., maker of the Jarvik-7, but it won't end the request for an artificial heart. One problem with the banned model is that the tubes connecting it to an external power source created a passage infection. Inventors are now working on new devices that would be fully placed, along with a tiny power pack, in the patient's chest. The first sample products aren't expected for another 10 or 20 years. But some people are already worrying that they'11 work- and that America's overextended healthcare programs will lose a precious $ 2.5 billion to $ 5 billion a year providing them for a relatively few dying patients. If such expenditures( 开支)cut into funding for more basic care , the net effect could actually be a decline in the nation's health. 1.According to the passage the Jarvik-7 artificial heart proved to be ________. A) a technical failure B) a technical wonder C) a good life-saver D) an effective means to treat heart disease Recall 召回,叫回,或召回之行为,尤指官方命令召回的 Cut into v.侵犯, 打断 题干中According to the passage … 提示我们本题是考核学生根据上下文全面理解和进行正确判 断的能力,问的是本文论述的Jarvik-7人造心脏性能和效果到底如何。文章一开头说Jarvik-7是一大 技术成就,但文章说到:…the .U.S. Food and Drug Administration concluded that the machine was doing more to endanger lives than to save them (Line -7-9).后面又提到:Last week the agency canceled its earlier approval , effectively banning the device.这两句话提供了这个问题的答案。因此,答案为A) 2.The new models of artificial hearts are expected_______. A) to have a working life of 10 or 20 years B) to be set fully in the patient's chest C) to be equipped with an external power source D) to create a new passage for infection 本题是考核正确理解事实细节的能力。题干中关键词为new models of artificial hearts,文章中与 之相应的内容在第二段第三句,即Inventors are now working…in the patient's chest.只不过题干中用了 同义词将new devices 题换了。因此答题时,应先找到关键词,再在文中找到相关句子,仔细阅读 这个句子以及它前后的句子,不难发现选项A)与文章内容相反,而C)和D)是关于前面提到的老设备 的情况,因此选项B)是答案。 细节题干扰项特点 用请下载www.docin.com/yongqing (1)与原句内容相反;(2)与原文内容一半相同一半不同;(3)与原句内容相似但过于绝 对化;(4)原文中根本没提到 虽然细节题在文章中能够找到答案,但正确选项不可能与阅读材料的原句一模一样,而是用 不同的词语或句型表达相同的思想,比如同是表示否定,可以用not clever, 也可以用fool, 或是averagely intelligent。命题人命题时就是利用这种手法造成与原作者在选词和句型结构 上 不同,但意义与原文相同。也就是说原文中作者表达具体事实和细节的信息值与问题的 信息值虽然在选词和句法上不同,但表达相同意义。 Example 2 Space is a dangerous place, not only because of meteors (流星) but also because of rays from the sun and other stars. The atmosphere again acts as our protective blanket on earth. Light gets through, and this is essential for plants to make the food which we eat. Heat, too, makes our environment endurable. Various kinds of rays come through the air from outer space, but enormous quantities of radiation from the sun are screened off. As soon as men leave the atmosphere they are exposed to this radiation but their spacesuits or the walls of their spacecraft, if they are inside, do prevent a lot of radiation damage. Radiation is the greatest known danger to explorers in space. The unit of radiation is called "rem". Scientists have reason to think that a man can put up with far more radiation than 0.1 rem without being damaged; the figure of 60 rems has been agreed on. The trouble is that it is extremely difficult to be sure about radiation damage-- a person may feel perfectly well, but the cells of his or her sex organs may be damaged, and this will not be discovered until the birth of deformed (畸 形的) children or even grandchildren. Missions of the Apollo flights have had to cross belts of high radiation and, during the outward and return journeys, the Apollo crew accumulated a large amount of rems. So far, no dangerous amounts of radiation have been reported, but the Apollo missions have been quite short. We simply do not know yet how men are going to get on when they spend weeks and months outside the protection of the atmosphere, working in a space laboratory. Drugs might help to decrease the damage done by radiation, but no really effective ones have been found so far. 1. According to the first paragraph, the atmosphere is essential to man in that ______. A) it protects him against the harmful rays from space B) it provides sufficient light for plant growth C) it supplies the heat necessary for human survival D) it screens off the falling meteors 解答此题时,首先要注意题干已点明了涉及范围,即第一段,说明这一题涉及到整个第一段 的主要意思,而不是只与某一两句话相关的细节,因此千万不要只抓住后面的‖atmosphere‖ 和 ―essential‖,而单单去看包含有这两个词的句子;然后关键词是‖atmosphere‖,问题的关 键在于‖in that‖,表示原因的,因此后面需要补充的内容应该构成前面一部分陈述,即“大 气对人类很重要”的原因。象这样仔细审过题后,再带着问题回去认真阅读第一段,较容易 找到正确答案A),大气层保护人类免受太空辐射的伤害。要注意四个选项中‖it‖指的是题干 的主语‖atmosphere‖大气层。“大气层”当然既不能提供“光”(B项),又不能提供“热”(C 项),也不能“屏蔽下落的流星”(D项)。 2. We know from the passage that ______. A) exposure to even tiny amounts of radiation is fatal B) the effect of exposure to radiation is slow in coming C) radiation is avoidable in space exploration D) astronauts in spacesuits needn't worry about radiation damage 这是一个典型的分散性细节题,题干中‖from the passage‖就说明了下面的各选项涉及到文章 主要话题的多个方面,通常分散在多个句子中。解答此类题时,要注意找出每个选项的关键 词,再回到文章中找到相关句子,仔细检查选项的陈述是否与文章相符。A项“即便微量的 辐射也是致命的”,‖…tiny amounts of radiation…fatal‖,这与文章第2段第2、3句的意思不 符;B项关键部分是‖effect…is slow in coming‖,即暴露在辐射线下的后果需要很长的时间 才会显现出来,这正好是归纳了第2段第4句话的主要观点,文中用了这样的句 型‖…not …discovered until…‖;C项和D项观点相似,前者说“在外空探险辐射是可以避免 的”,后者说“宇航员穿上宇航服就不用担心辐射的伤害”,通过第2段第1句话就可以排除 用请下载www.docin.com/yongqing 了。这个题的难点在于选项并没有直接采用原文中的说法,而是都进行了归纳或重述。这时, 考生尤其要记住不可主观推测,而应耐心地在原文中找出证据。 3. The harm radiation has done to the Apollo crew members ______. A) is insignificant B) seems overestimated C) is enormous D) remains unknown 此题是一个典型的局部细节题,题干问及辐射对阿波罗号上的宇航员产生危害的情况。文章 第2段第5、6句话是关于阿波罗号的,将阿波罗号上的宇航员身上积累了大量的辐射雷目, 但还没有达到危险量的报道。这说明只有D项“仍不得而知”是正确的,A项说“是微不 足道的”,B项说“看来是被夸大了”,C项说“非常之大(严重)”,这些都不准确。 Example 3(CET4 1998/1 Passage 1) In bringing up children, every parent watches eagerly the child‘s acquisition (学会) of each new skill—the first spoken words, the first independent steps, or the beginning of reading and writing. It is often tempting to hurry the child beyond his natural learning rate, but this can set up dangerous feelings of failure and state of worry in the child. This might happen at any stage. A baby might be forced to use a toilet too early, a young child might be encouraged to learn to read before he knows the meaning of the words he reads. On the other hand, though, if a child is left alone too much, or without any learning opportunities, he loses his natural enthusiasm for life and his desire to find out new things for himself. Parents vary greatly in their degree of strictness towards their children. Some may be especially strict in money matters. Others are severe over times of coming home at night or punctuality for meals. In general, the controls imposed represent the needs of the parents and the values of the community as much as the child‘s own happiness. As regards to the development of moral standards in the growing child, consistency is very important in parental teaching. To forbid a thing one day and excuse it the next is no foundation for morality (道德). Also, parents should realize that ―example is better than precept‖ It they are not sincere and do not practise what they preach (说教), their children may grow confused, and emotionally insecure when they grow old enough to think for themselves, and realize they have been to some extent fooled. A sudden awareness of a marked difference between their parents‘ principles and their morals can be a dangerous disappointment. Eagerly watching the child‘s acquisition of new skills . A) should be avoided B) is universal among parents C) sets up dangerous state of worry in the child D) will make him lose interest in learning new things 本题的测试点是文章第一段的段首句:In bringing up children, every parent watches eagerly the child‘s acquisition of each new skill—the first spoken words, the first independent steps, or the beginning of reading and writing. 句中的every parent表明这种现象很普遍,和B) 项的is universal among parents意思相同,因此,本题的正确答案应是B)。 需要注意的是,判断正误类型的题目的每个选项都对应着文中的一个细节,所以在 此将它归为细节类题目,做这类题的办法也是拿选项来一一对应文中的相应内容, 然后做出 判断。 PART THREE HOW TO ANSWER THE WORD-INTERPRTIONN QUESTIONS 四级考试阅读理解中对词汇或单名语义的测试,主要是考查学生利用上下文判断单词或 词组甚至是句子在特定语言环境中确切含义的能力。大体上,这类题目可以分为两小类:词 用请下载www.docin.com/yongqing 语释义类和确定指代关系类。 I词语释义类 这类题目中所测的词语一类是纯粹的生词,要求考生利用上下文判断该词的意思;另一 类词语对学生来说并不陌生,但在文章中该词的意思或者有所引申,或者此处用的是平时学 生很少用到的义项。其提问方式主要有: ?The word (term) ―…‖ in Line… most probably means… ?The word ―…‖ stands for (is closest in meaning to)… ?The expression ―…‖ can be interpreted as… ?The word ―…‖ can be replaced by… ?According to the passage, what is ―…‖? The word/ expression ―…‖in paragraph x can be interpreted as _______. 做这类题目主要是要利用好上下文的暗示或推理上下文的逻辑关系来确定词语或短句的意 思。看下面的例子: Example1(CET4 2000/6 Passage 1) When the officials got it right, they were, on average, 17 meters away from the action… ?The word ―officials‖ (Line 2, Para. 4) most probably refers to . A) the researchers involved in the experiment B) the referees of the football tournament C) the observers at the site of the experiment D) the inspectors of the football tournament 联系上文,这一实验研究的是足球裁判场上执法的情况, 而这一句说的是officials在 纠正场上队员犯规时,他大多是在17米以外的地方。很显然,此处officials指的就是足球 裁判。答案该是B)。 有时,你甚至可以利用上下文专门的提示,比如破折号,that is (to say) 等来进行判断。 Example2(CET4 1999/1 Passage 2) President Coolidge‘s statement, ―The business of America is business,‖ still points to an important truth today—that business institutions have more prestige (威望) in American society than any other kind of organization, including the government. Why do business institutions possess this great prestige? One reason is that Americans view business as being more firmly based on the ideal of competition than other institutions in society. Since competition is seen as the major source of progress and prosperity by most Americans, competitive business institutions are respected. Competition is not only good in itself, it is the means by which other basic American values such as individual freedom, equality of opportunity, and hard work are protected. Competition protects the freedom of the individual by ensuring that there is no monopoly (垄 断) of power. In contrast to one, all-powerful government, many businesses compete against each other for profits. Theoretically, if one business tries to take unfair advantage of its customers, it will lose to competing business which treats its customers more fairly. Where many businesses compete for the customers‘ dollar, they cannot afford to treat them like inferiors or slaves. A contrast is often made between business, which is competitive, and government, which is a monopoly. Because business is competitive, many Americans believe that it is more supportive of freedom than government, even though government leaders are elected by the people and business leaders are not. Many Americans believe, then, that competition is as important, or even more important, than democracy in preserving freedom. 用请下载www.docin.com/yongqing Competition in business is also believed to strengthen the ideal of equality of opportunity. Competition is seen as an open and fair race where success goes to the swiftest person regardless of his or her social class background. Competitive success is commonly seen as the American alternative to social rank based on family background. Business is therefore viewed as an expression of the idea of equality of opportunity rather than the aristocratic (贵族) idea of inherited privilege. ?The statement ―The business of America is business‖ probably means ― ‖. A) The business institutions in America are concerned with commerce B) Business problems are of great importance to the American government C) Business is of primary concern to Americans D) America is a great power in world business 看破折号后面的内容:在美国社会中,商业集团比其他任何的组织都享有威望。而所给 选项中与该意思相同的只有C)项:商业,对美国人来说,是最为重要的。故C)为正确答 案。 对于那些不是生词,但平时较少用的义项,也是利用上下文的逻辑关系来判断。 Example3(CET4 1998/6 Passage 1) Psychologist George Spilich and colleagues at Washington College in Chestertown, Maryland, decided to find out whether, as many smokers say, smoking helps them to ―think and concentrate.‖ Spilich put young non-smokers, and smokers deprived (被剥夺) of cigarettes through a series of tests. In the first test, each subject (实验对象) sat before a computer screen and pressed a key as soon as he or she recognized a target letter among a grouping of 96. In this simple test, smokers, deprived smokers and non-smokers performed equally well. The next test was more complex, requiring all to scan sequences of 20 identical letters and respond the instant one of the letters trans-formed into a different one. Non-smokers were faster, but under the stimulation of nicotine (尼古丁), active smokers were faster than deprived smokers. In the third test of short-term memory, non-smokers made the fewest errors, but deprived smokers committed fewer errors than active smokers. The fourth test required people to read a passage, then answer questions about it. Non-smokers remembered 19 percent more of the most important information than active smokers, and deprived smokers bested those who had smoked a cigarette just before testing. Active smokers tended not only to have poorer memories but also had trouble separating important information from insignificant details. ―As our tests became more complex,‖ sums up Splilich, ―non-smokers performed better than smokers by wider and wider margins.‖ He predicts, ―smokers might perform adequately at many jobs—until they got complicated. A smoking airline pilot could fly adequately if no problems arose, but if something went wrong, smoking might damage his mental capacity.‖ ?The word ―bested‖ (in Para. 5) most probably means . A) beat B) envied C) caught up with D. made the best of 根据上下文判断,抽烟有多种危害,此处的实验表明:非抽烟者比抽烟者记住的内容多, 而那些“被迫不抽烟者”又“bested”那些在实验前刚刚抽过烟的人。所以,此处的bested, 可以理解为“比谁表现得好;打败”。因此,选项A)就是答案。 总之,这类题只要出现,那必定在上下文中会有相关的提示性的内容,比如破折号、冒 用请下载www.docin.com/yongqing 号等符号,that is, refers to, means, or, for instance, for example, such as, like, as, likewise等提示 性词语,只要认真找,这类题目还是较容易解决的。 II确定指代关系类 这类题主要考查学生对英语中用得较多的指示代词的理解与辨别能力,其中较多的是代 词it, they, them等,有时也有名词或指示副词。做这类题目,主要是要具备这样的能力:能 搞清楚句子的主干结构以及句与句之间的逻辑关系,弄清每一句话的主谓宾定状补等句子成 分。只要平时锻炼出这种能力,再怎么考你都会得心应手地把题目答好。常见提问方式有: 1.According to the passage, the term/the word ―…‖ refers to ______. 2. By ―…‖, the author indicates that ______ Example4(CET4 1997/1 Passage ) Statuses are marvelous human inventions that enable us to get along with one another and to determine where we ―fit‖ in society. As we go about our everyday lives, we mentally attempt to place people in terms of their statuses. For example, we must judge whether the person in the library is a reader or a librarian, whether the telephone caller is a friend or a salesman, whether the unfamiliar person on our property is a thief or a meter reader, and so on. The statuses we assume often vary with the people we encounter, and change throughout life. Most of us can, at very high speed, assume the statuses that various situations require. Much of social interaction consists of identifying and selecting among appropriate statuses and allowing other people to assume their statuses in relation to us. This means that we fit our actions to those of other people based on a constant mental process of appraisal and interpretation. Although some of us find the task more difficult than others, most of us perform it rather effortlessly. A status has been compared to ready-made clothes. Within certain limits, the buyer can choose style and fabric. But an American is not free to choose the costume (服装) of a Chinese peasant or that of a Hindu prince. We must choose from among the clothing presented by our society. Furthermore, our choice is limited to r size that will fit, as well as by our pocketbook (钱 包). Having made a choice within these limits we can have certain alterations made, but apart from minor adjustments, we tend to be limited to what the stores have on their racks. Statuses too come ready made, and the range of choice among them is limited. ?In the last sentence of the second paragraph, the pronoun ―it ‖ refers to ― ‖. A) fitting our actions to those of other people appropriately B) identification of other people‘s statuses C) selecting one‘s own statuses D) constant mental process 纵观全文,it指的是离它最近的task,而这项task也就是前面的―we fit out actions to those of other people based on a constant mental process of appraisal land interpretation‖, 因此,答案 是A)。 总之,词汇和语义题形式多种多样,应根据上下文的同义、对照、比喻、定义、重述、 举例、词缀等关系来判断词句的意义,并根据逻辑推理来确定某词语和句子的意义。下面对 以上技巧各举一例加以说明: 1.根据定义猜测词义 In slang the term "jam" constitutes a state of being in which a person finds himself or herself in a difficult situation. 同样,从上下文的定义可知jam一词在俚语中的意思是―困境‖。 用请下载www.docin.com/yongqing 2.根据复述猜测词义 Semantics, the study of the meaning of words, is necessary if you are to speak and read intelligently. 此例逗号中短语意为―对词意义进行研究的学科‖。该短语与前面生词semantics式同位关系,因此我们不难猜出semantics指―语义学‖。 3.根据举例猜测词义 The consequences of epochal events such as wars and great scientific discoveries are not confined to a small geographical area as they were in the past. 句中―战争‖和―重大科学发现‖是生词的实例,通过它们我们可以猜出epochal的 大致词义―重要的‖,这与其确切含义―划时代的‖十分接近。 4.利用上下文词语意义的互相联系猜测词义 ogging has become very popular in some countries ,It is believed to be a good exercise for old J people . "Jogging"的意思通过"a good exercise for old people "可以推断出是一种适合老年人的的运动方式。尽管我们可能还不能肯定它的确切解释,但这一生词已经不会影响我们的阅读和理解了。 5.根据对比关系猜测词义 Andrew is one of the most supercilious men I know. His brother, in contrast, is quite humble and modest. 该例中supercilious对许多人来说可能是个生词,但是句中短语in contrast,(相对照的,相对比的)可以提示我们supercilious和后面词组humble and modest(谦卑又谦虚)是对比关系。分析出这种关系后,我们便能猜出supercilious意为―目空一切的,傲慢的‖。 表示对比关系的词汇和短语主要是unlike,not,but,however,despite,in spite of,in contrast 和while 引导的并列句等。 6.根据同义词的替代关系猜测词义 Although he often had the opportunity, Mr. Tritt was never able to steal money from a customer. This would have endangered his position at the bank, and he did not want to jeopardize his future. 作者为避免重复使用endanger一词,用其同义词jeopardize来替代它,由此推知其词义为―使. . . 陷入危险,危及、危害‖。 利用各种已知信息推测、判断词义是一项重要的阅读技巧。四级考试中的词汇与语义题往往不会单一地测试某一种猜词技巧,而是要求应试者综合运用各种技巧,这就要求我们在做阅读理解练习遇到难词难句时,要灵活运用,综合运用上面提到的几种猜测技巧,并进行一定程度的强化,从而使它成为一种本能,以排除生词的障碍,顺利理解文章的思想内容,提高阅读速度。 PART FOUR HOW TO DRAW AN INFERENCE THROUGH THE CONTEXT 与细节类题目不同,此类题目比较难,需要根据文章中提供的信息进行综合的分析推理,在完全理解文章内容的基础上做出合理的逻辑推断。测试点可以涉及整篇文章,如问作者的立场观点、态度、目的以及文章的语言风格等;也可能是只涉及到某个段落、句子或词组的细节问题;有时甚至可以是问没有出现过的前面或后续的内容会是什么。常见的提问方式有: 1. We can infer from this passage (paragraph, fact, example, evidence…) that 2. It may be inferred from this passage(paragraph…) 3. An inference which may not be made from this passage (paragraph…) 4. With which of the following statements would the author be most likely to agree? 5. Implied but not stated… 6. The passage (author) implies, but does not directly state that… 7. The passage (author) suggests that… 8. Which of the following can best describe the attitude of the author towards…? 用请下载www.docin.com/yongqing 9. The author probably feels that… 10. The author seems to be (in favour of)… 11. The tone of the passage (author) may be… 12. From this passage, we would say that the author… 13. One could conclude from this paragraph (passage…) 14. Upon reading this passage, one (we) might logically conclude that… 15. The following paragraph would most probably discuss… 归纳起来,推理判断类的题目又可以细分为几小类:细节类判断推理、对作者态度的判 断推理、对文章结论的判断推理。 I细节类判断推理 这一类题不同于考对细节的普通理解与转述,而是要根据某一细节进行合理的推断,看 能得出什么样合理的“言外之意”。 Example1(CET4 2000/6 Passage 1) Long after the 1998 World Cup was won, disappointed fans were still cursing the disputed refereeing (裁判) decisions that denied victory to their team. A researcher was appointed to study the performance of some top referees. The researcher organized an experimental tournament (锦标赛) involving four youth teams. Each match lasted an hour, divided into three periods of 20 minutes during which different referees were in charge. Observers noted down the referees‘ errors, of which there were 61 over the tournament. Converted to a standard match of 90 minutes, each referee made almost 23 mistakes, a remarkably high number. The researcher then studied the videotapes to analyze the matches in detail. Surprisingly, he found that errors were more likely when the referees were close to the incident. When the officials got it right, they were, on average, 17 meters away from the action. The average distance in the case of errors was 12 meters. The research shows the optimum (最佳的) distance is about 20 meters. There also seemed to be an optimum speed. Correct decisions came when the referees were moving at a speed of about 2 meters per second. The average speed for errors was 4 meters per second. If FIFA, football‘s international ruling body, wants to improve the standard of refereeing at the next World Cup, it should encourage referees to keep their eyes on the action from a distance, rather than rushing to keep up with the ball, the researcher argues. He also says that FIFA‘ s insistence that referees should retire at age 45 may be misguided. If keeping up with the action is not so important, their physical condition is less critical. ?What is one of the possible conclusions of the experiment? A) Age should not be the chief consideration in choosing a football referee. B) A football referee should be as young and energetic as possible. C) The ideal retirement age for an experienced football referee is 45. D) An experienced football referee can do well even when in poor physical condition. 全文介绍了研究者的几个发现:裁判在比赛中应在最佳位置、他的最佳跑动速度等。这 是文章的最后一段,而本题目就是对这一部分的考查。这位研究者认为:国际足联所坚持的 “裁判须在45岁退休”的原则可能是一种误导,因为如果裁判在场上跑动不再那么重要, 那对他们身体条件的要求也就不再那么严格了。但作者此处并没有D)的意思:一位有经验 用请下载www.docin.com/yongqing 的足球裁判即使在身体不好的时候也能做得很好。因此,符合此处的细节的A)项应该是合 乎逻辑的答案:年龄不应是挑选裁判时考虑的主要因素。 II对作者态度的判断推理 需要特别指出的是,在推理判断作者的立场观点、态度、目的时要仔细体会。因为有时 作者的态度不是明白地写出来的,而是隐藏在字里行间的Example2(CET4 2000/1 Passage 1) Unless we spend money to spot and prevent asteroids (小行星) now, one might crash into Earth and destroy life as we know it, say some scientists. Asteroids are bigger versions of the meteoroids (流星) that race across the night sky. Most orbit the sun far from Earth and don‘t threaten us. But there are also thousands of asteroids whose orbits put them on a collision course with Earth. But $ 50 million worth of new telescopes right now. Then spend $10 million a year for the next 25 years to locate most of the space rocks. By the time we spot a fatal one, the scientists say, we‘ll have a way to change its course. Some scientists favor pushing asteroids off course with nuclear weapons. But the cost wouldn‘t be cheap. Is it worth it? Two things experts consider when judging any risk are: 1) how likely the event is; 2) How bad the consequences if the event occurs. Experts think an asteroid big enough to destroy lots of life might strike Earth once every 500,000 years. Sounds pretty rare—but if one did fall, it would be the end of the world. ―If we don‘t take care of these big asteroids, they‘ll take care of us,‖ says one scientist. ―It‘s that simple.‖ The cure, though, might be worse than the disease. Do we really want fleets of nuclear weapons sitting around on Earth? ―The world has less to fear from doomsday (世界末日) rocks than from a great nuclear fleet set against them,‖ said a New York Times article. Which of the following best describes the author‘s tone in this passage? A) Optimistic. B) Critical. C) objective. D) Arbitrary. 纵观全文,本文主要介绍了:什么是小行星,小行星可能会撞击地球,避免这种撞击的 可能的办法以及人们对这种办法心存的忧虑。可以看到,作者几次用―say some scientists‖, ―the scientist say‖, ―says one scientist‖, ―said a NEW York Times article‖ 等来表明所有这些内 容,作者的观点都是从科学家的言论或报刊上得来的。所以本题答案是C)客观的。 III对文章结论的判断推理 这类题往往带有标志性的词conclude, 考查学生常规的逻辑推理能力。做这类推理题, 关键是要搞清楚前提和结论之间的逻辑关系。有了前提,那么合理的结论该是什么呢, Example3 (CET4 2000 12 Passage 4) In 1993, New York State ordered stores to charge a deposit on beverage (饮料) containers. Within a year, consumers had returned millions of aluminum cans and glass and plastic bottles. Plenty of companies were eager to accept the aluminum and glass as raw materials for new products, but because few could figure out what to do with the plastic, much of it wound up buried in landfills (垃圾填埋场). The problem was not limited to New York. Unfortunately, there were too few uses for second-hand plastic. Today, one out of five plastic soda bottles is recycled(回收利用)in the united States. The reason for the change is that now there are dozens of companies across the country buying discarded plastic soda bottles and turning them into fence posts, paint brushes, etc. As the New York experience shows, recycling involves more than simply separating valuable 用请下载www.docin.com/yongqing materials from the rest of the rubbish. A discard remains a discard until somebody figures out how to give it a second life—and until economic arrangements exist to give that second life value. Without adequate markets to absorb materials collected for recycling, throwaways actually depress prices for used materials. Shrinking landfill space, and rising costs for burying and burning rubbish are forcing local governments to look more closely at recycling. In many areas, the East Coast especially, recycling is already the least expensive waste-management option. For every ton of waste recycled, a city avoids paying for its disposal, which, in parts of New York, amounts to savings of more than $ 100 per ton. Recycling also stimulates the local economy by creating jobs and trims the pollution control and energy costs of industries that make recycled products by giving them a more refined raw material. It can be concluded from the passage that . A) recycling is to be recommended both economically and environmentally B) Local governments in the U. S. can expect big profits from recycling C) Rubbish is a potential remedy for the shortage of raw materials D) Landfills will still be widely used for waste disposal 纵观全文,尤其是从文章的最后一段可知:以填埋和焚烧方式处理垃圾的费用越来越高, 而垃圾的回收利用却既省了钱,又能推动地方经济的发展,还能减少环境污染。所以,综合 四个选项,A)的结论最合理:垃圾的回收利用不管从经济上还是从环境效益上讲,都值得 推广。 Part Five Reading Practice(综合练习) (see students‘ book) Part Six Affixes 一 英语中常见的前缀: 前缀 前缀的意思 例子 a- 不,没有 Amoral非道德的,atypical非典型的 a- 在„„状态 Asleep熟睡的,awake醒着的 ab-, abs- 离开,不 Abnormal不正常的,absent缺席 ante- 在„„之前 Anteroom前厅,antenatal产前 anti- 反对,反 Antibody抗体,antidote解毒药 arch- 主要的 Archbishop大主教,archenemy大敌,要敌 auto- 自动的 Automobile汽车,autonomy自治 Be- 使,视为 Belittle轻视,befriend亲近 Bene- 好 Benefit利益,benevolent仁爱的 Bi- 两个,两倍 Bicycle自行车,bilateral双边的 Bio- 使命的 Biotechnology生物工程,biography传记 Co- 一起 Cooperation协作,coeducation男女同校 Counter- 反对 Counteract抵消,counterbalance抗衡 De- 下行,离开 Descend下行,detach分开,debase贬损 Dis- 分开,不 Discourage劝阻,disagree不同意 Ex- 外的 Exclude不包括,export出口,exhale呼 Ex- 以前的 Ex-wife前妻,ex-president前总统 Fore- 在„„之前 Foreground前景,foremost最重要的 Homo- 相同 Homonym同音异义词,homosexual同性恋 用请下载www.docin.com/yongqing Hyper- 超越 Hypercritical过分苛刻的,hypersensitive过敏的 Il- 不 Illegal不法的,illiterate不识字的 Im- 不 Impossible不可能,imperfect不完善的 Indirect不直接的,inconsiderate不为别人考虑的,incorrectIn- 不 不正确的 In- Inbred先天的,injection注射,inhale吸气 Inter- 两者之间 International国际的,interact互相作用 Irresponsible不负责任的,irrational无理性的,irregular不Ir- 不 规则的 Misjudge错判,mislead引导错误,misconduct不规矩,misfitMis- 误,错的 不适合 Nonsense废话,non-essential非本质的,non-conductor非导Non- 不 体 Over- 过于 Overcome克服,overdo夸张,overestimate高估 Pan- 泛 Panorama全景,pan-island全岛 Poly- 许多 Polysemy多义词,polytechnic多种工艺的 Post- 之后 Postwar战后,postgraduate研究生 Pre- 之前 Predict预言,preview预习 Re- 再次,重复 Repeat重复,reflect反射 Semi- 一半 Semicolon分号,semifinal半决赛 Sub- 下 Sub-leading小标题,subnormal正常以下的 Super- 超过 Supersonic超声波,superfluous多余的 Syn- 和„„一起 Synthesis合成,sympathy同情 Tele- 从远处 Telegram电报,televise实况转播 Trans- 横过,转移 Transport运输,transfer迁移,transform变化 Unfinished 未完成的,unequal不平等的,unattractive没有Un- 不 吸引力的 Un- 逆向 Unburden卸除重组,undo复原 Uni- 统一 Uniform一致的,unison调和 Vice-president副总统,vice-captain副船长,vice-admiral海Vice- 副职 军中将 With- 退后,离开 Withdraw撤退,withhold抑制 二 英语中常见的后缀: 名词后缀1 加在相应的动词后面,构成名词, Acknowledgement, amendment, -ment 行为动作或结果具体事物,工具 instrument, equipment -sion Submission, conclusion 行为、状态、性质、结果 -(a)tion Education, construction 行为、状态、性质、结果 -ing Teaching, earnings 行为及其结果 -ure Pressure, mixture, procedure 行为及其结果 性质、状况、行动、过程、程度、Resistance, acquaintance, -ance inheritance, conveyance 总量 Emergence, reference, -ence 动作、性质、状态 maintenance, confidence 名词后缀2 加在形容词和名词后构成 - ness Eagerness, heaviness, sadness 性质、状态、程度 -ity Hostility, security, safety 性质、状态 -dom Wisdom, freedom, kingdom 职位、领域、性质、状态 -th Truth, length, youth, health 抽象概念、“„„的度” 用请下载www.docin.com/yongqing -hood Neighborhood, childhood, 身份、资格、性质、状态 manhood -ship Relationship, friendship, 情况、状态、性质、身份、技巧 Hardship 名词后缀3 加在名词或动词之后,构成形容 词 -er Banker, beginner, entertainer, computer, „„人,„„者,工具,用品 sprayer, cooker -or Editor, counselor, survivor, inventor, 人或物 translator, projector -ee 受动者,处于某一善下的人 Trainee受训人员,examinee考生, employee受雇人员 -ian=-an Musician, technician, American, „„地方的人,精通„„的人 Canadian, Australian, Russian ist Scientist, typist, capitalist, Buddhist, 信仰„„的人,从事„„研究的 communist, chemist 人 形容词加在名词或动词之后,构成形容 后缀 词 -ful Cheerful, disgraceful, doubtful 充满„„的,有„„的 -less Pointless, faithless, windless 没有„„的,不„„的 -al Emotional, intellectual, global 有„„属性的 -y Chilly, breezy, lazy, bossy, foggy, faulty, 有„„的,似„„的,带„„的 fiery, patchy -ie(al) Romantic, angelic, majestic, electric, 与„„有关的,具有„„的特征, physical, organic 含有„„的 -ary Secondary, primary, legendary 与„„有关的, -ory Introductory, satisfactory, congratulatory, „„性质的,属于„„的,作„„ inhibitory 之用 -ous Various, ceremonious, furious 有„„的,充满„„的 -ive Descriptive, creative, explosive, captive, „„的,容易„„的,„„性的 active, sensitive -able Suitable, countable, reliable „„的,能„„的 -ible Visible, divisible, reducible 可„„的 动词后缀 -ify Classify, simplify, beautify 使„„,„„化 -en Sharpen, blacken, weaken, strengthen, fasten, 使 redden -ize Organize, modernize, 变为„„,„„化 terrorize, dramatize, commercialize Key To Reading Comprehension 1(C 2(BABAB Part one How To Grasp The Main Idea Example 1—5 BCBBC ; Example 6—10CDCAC ; Example 11—12EC Part two How to find specific Details Example1 A B ; Example2 ABD; Example3 B ; Example4CDA; Example5 AD Example6 DBC; Example7 ADBCD; Example8 BAB; Example9 BADC; Example 10 ACD; Example11 D; Example12 BCBC; Example 13 CBC; Example14 B ; Example15 B; Example16 ACBAD 用请下载www.docin.com/yongqing Part three how To Answer Word—Interpretation questions Example 1-5 BCAAC; Example 6-11见讲义; Example12 D ; Example13 A; Example14 C ; Example15 CAAC Part Four How to Draw an Inference Through The Context Example1 A ; Example2 C ; Example3 A; Example4AB ; Example5 A ; Example6 D; Example7 BBB; Example8 B ; Example9C Example10A ; Part IV Reading Practice Example 1 内容大意 这是一篇谈论可能引起空难的原因的文章。文章说,当今威胁航空安全的最大问题很可能不是带枪的恐怖分子,而是携带便携式计算机的人。在过去的15年里,据飞行员的报告,可能由于电磁波干扰而引起的航空事件已有100多起。虽然这种干扰尚未得到证实,但越来越多的专家将矛头指向诸如便携式计算机、收音机、磁带录放机和移动电话等便携式电器。 RTCA—专门为航空业提供咨询的组织——向所有的航空公司建议,在飞行的关键时刻,特别是在起飞和降落时,禁止使用上述电器。有一些专家则走得更远,呼吁在飞行期间全程禁止使用上述电器。目前,有关使用这类电器的规定由各航空公司自行决定。虽然部分航空公司明令禁止在飞机起飞和降落时使用上述设备,但多数航空公司认为,许多乘客仍想在飞机上继续工作,对全程禁止还犹豫不决。 但问题的焦点是人们还不甚了解电磁场是如何影响飞机上的计算机系统的。专家们只知道这些便携电器设备发出的辐射会影响飞机上的导航和通信系统的波长,但在实验室却无法重复这种影响,因此也就无法确定这种干扰究竟有没有危险。 飞机可能很容易受到这种干扰,这就增大了一种可能的危险性:恐怖分子可能会利用无线电破坏导航设备。同样令人担忧的是,乘客可能因为音乐声太大而没有听到关闭无线电的指令。 试题分析 1. 正确答案:A。 本题问及全篇的主要文章主要谈论的是“各航空公司都要执行的一项新规定”(A项),是谈论“电器设备的缺陷”(B项),还是谈论“关于航空的安全措施” (D项),从文章内容分析可以看出,上述3项都不是文章谈论的主旨。文章主要谈论的是在飞机上使用便携式电子设备可能是引起空难的一个原因。因此,答案是C项。 2. 正确答案:D。 这一题问及一个细节问题,比较容易。考生只有读懂文章第 1段第2句话才能正确地选择答案D项。 3. 正确答案:B。 本题问:为什么只有很少的航空公司在飞行期间全程禁止乘客使用这类电子设备,看起来这一题问的是个局部问题,实质上这一题考核的是对文章全篇内容的正确理解。航空安全问题涉及到乘客的生命,如若事实证明使用这种设各是导致空难的原因,那么各航空公司全程禁止使用这种设备是不会遭到乘客的拒绝的,但正是这一点尚未得到证实,所以各航空公司对全程禁止使用这种设备尚犹豫不决。因此本题的答案是B项。A项和D项文章没有谈到。C项说“许多乘客都拒绝乘坐全程禁止使用收音机和录放机的飞机”。文章并没有谈到“许多乘客都拒绝”乘这类飞机,也没有提到“全程只禁止使用收音机和录放机”的事。文章只是提到许多航空公司“担心许多乘客想在飞行时(继续) 用请下载www.docin.com/yongqing 工作”(并非要在飞机上“听收音机或录放机”),因而对“全程禁止使用这类设备犹豫不决”。本题有一定的难度,阅读时不但要理解孤立的句子,而且要能够跟上作者思路的展开,提高对整篇文章的理解能力。 4. 正确答案:C。 这一题考核文章第3段的内容。试题问:为什么预知电磁场对飞机上的计算机系统产生可能的影响是困难的。本题比较简单。第3段的第3句就回答了这个问题,因此答案是选项C。 5. 正确答案:A。 这是一道要求考生在读懂文章的基础上,推导出作者对在飞机上禁止使用这类电器的态度的问题。从全篇来看,作者对在飞机上全面禁止使用便携式电子设备态度是肯定的,从作者对问题的提出(The biggest safety threat facing airlines today„),列举的数据以及谈论这一问题的口气,都可以看出这一点,因此选项A是答案。B项说作者“夸大了这种电子干扰的危险性”,这与文章内容不符;D项说作者“认为飞行时全面禁止是没有道理的”,这完全不是作者的本意。“某些航空公司对是否全面禁止这类设备还犹豫不决”这一句说的是航空公司的态度,而不是作者的态度,因此C项说作者对这一问题“尚未形成自己的看法”也不对。阅读时正确理解文章内容包括正确理解作者的观点和态度。 Example 2 内容大意 本文讲述了梦想当作家与写作,幻想与现实之间的距离。有许多年轻人曾告诉本文作者,他们想当作家。本文作者在鼓励这些年轻人的同时,也指出“当一名作家”和从事写作之间有天壤之别。这些青年人通常想到的是伴随名作家的财富和荣誉,而不是长时间单独地与打字机为伴。本文作者向那些年轻人指出:“首先应该有写的欲望”,而不是“想当作家”。 作者在第2段进而指出,在现实生活中写作是件孤独、单靠个体而且待遇菲薄的职业。 每一个幸运得以成功的作家的背后还有众多的人永远无法实现自己的作家梦。接下来,本文作者讲述了自己的写作生涯。他为了当一名自由作家放弃了在美国海岸警卫队从事了20年的工作。当时前途渺茫。朋友在纽约为他找了一间既没有取暖设施也没有卫生间的房子,但在购置一台旧打字机之后他就感到自己已是一个名副其实的作家了。 然而一年后,他因为没有能够取得进展而开始怀疑自己。但多年来强烈的写作愿望坚定了他实现自己梦想的信念,即使这意味着难以预料的前途和失败的可能。在未成功之前的写作生涯就是一种充满希望的虚幻世界,而任何想成为作家的人都必须学会在这样一个虚幻世界里生活。 试题分析 1. 正确答案:A。 本题问及本文的中心思想。第2段第1句就指出 writing is a lonely, private and poor-paying affair。第 2段的最后一句又说:it(指本文作者当时住的房间)was cold and had no bathroom,而第 3段第 2句则说:It was so hard„ enough to eat,故本题的答案应该是 A项。本文通篇说的是写作生涯的艰难和贫困;文章第1段第2和第3句只是说想当作家和从事写作这两者之间是有天壤之别的。在多数情况下,想当作家的人想的只是伴随作家的荣华和富贵而不是作家成名之前的孤军奋战的写作生活。文章并没有说作家追求名利是不现实的。因此选项C是错的。文章第1句说的是,作者对那些想当作家的年轻人总是予以鼓励,而不是鼓励所有的年轻人去从事写作。因此,选项D也不可能是答案。 2. 正确答案:D。 本题考核学生能否根据文章的内容进行正确的推论。第2段第2句说“每一个幸运成功的作家的背后还有众多的人永远无法实现自己的作家梦”,故 用请下载www.docin.com/yongqing 答案应该是选项D。文章通篇介绍的只是作家生涯的艰难和清贫,并无一处说著名的作家通常过的是贫困和孤独的生活。因此选项C不可能是答案,但该选项干扰性较大。文章中未提及A项和B项,故这两个选项也是错的。 3. 正确答案:B。 本题考核正确理解单句的能力。本题问的是本文作者在从事写作生涯的一年以后为什么会开始怀疑自己。文章第3段第1句说: After a year or so,however,I still hadn’t gotten a break and began to doubt myself. 然而一年以后,我仍然没有取得什么进展,于是就开始怀疑自己。这一句亦回答了本题所提出的问题。因此正确答案应该是B项。 4. 正确答案:B。 本题考核学生辨认重要细节的能力。浏览本文后便知道本题的答案在最后一段的第6句里,即:I would keep putting my dream to the test„。作者在前一句里声明自己不会成为那种在临死前还在想“假如我„”,而是如第6句所说的:“会坚持自己的理想,把自己的理想付诸实践”。由此可见,正确答案是B项。如果没有正确理解“„ people die wondering,hat if„”的意思,就会误选 C项。 5. 正确答案:C。 本题考核学生能否根据文章的内容进行正确的推论。本文最后一句中的this指的是前一句的 even though it meant living with uncertainty and fear of failure(„即使这种生活意味着难以预料的前途和失败的可能),因此本题的答案只能是选项C。 Example 3 内容大意 这是一篇科普性的短文,讲述的是关于奶油为什么很快地变质,而黄油则不然。文章指出,这一发现为在食品加工过程中不再使用化学防腐剂提供了可能性。 奶油和黄油所含的物质大体相同,为什么奶油变酸比黄油快得多呢,这一直是个谜。奶油和黄油都是乳剂,即一种液状小球体均匀分布在另一种液体之中。所不同的是,液状小球体内部及其外围液体的结构。 奶油中的脂肪小球体在水中自由漂移,而黄油中的水溶液小球体则禁锢在脂肪的汪洋之中。能使食物变质的细菌喜欢在混合物中的水域里生活。这就是说,这类细菌可以在奶油中自由地繁衍。如果情况相反,这类细菌则被深深禁锢在脂肪汪洋中的隔离仓内,无法扩展,营养很快就消耗殆尽,慢慢被自身排泄物毒化致死。黄油就是这种情况,它本身就有这样一种自我限制系统,阻止这类细菌繁衍。 研究人员正打算与某些公司合作,这些公司热衷于考虑如何利用改变食品本身结构的方法来生产能抵御细菌侵扰的产品。领导这项研究的 Brocldehurst 相信,将乳状液,例如用于色拉的奶油,做成与黄油一样是可能的,关键是保持色拉酱的液态状,而不是将其变为固态的块状。 试题分析 1. 正确答案:A。 本题问及由 Brocklehurt 领导的研究项目的意义。这是一道考核考生是否理解文章主旨的考题。从文章总体来看,第1段谈及科学家已经找到了奶油变质的速度比黄油快的原因,进而明确提出,这一发现为在食品加工过程中不再使用化学防腐剂提供了可能性。接下来的第2、第3、第4段具体阐明奶油容易变质而黄油不容易变质的道理。最后一段又说研究人员准备与食品公司合作,生产既能保鲜又不用任何化学防腐剂的食品。从4个选项来看,A项是答案。B项说“该项研究发现了奶油和黄油中的液状小球体”。文章虽然提到液状小球体,但未谈及是否是这项科学研究的成果,所以不是答案。C项说“该项研究揭示了奶油和黄油中细菌繁衍的秘密”。文章谈到细菌在奶油和黄油中繁衍的事,但只是为了说明奶油和黄油虽然化学成分大体相同但结构不同,因而一个容易变质。另一个则不容易的道理,并非是文章的主旨。D项说“该项研究发现奶油和黄油化学成分一样”。文章 用请下载www.docin.com/yongqing 提到这两者的化学成分一样,但未说明是否是这项科学研究的发现,因此也不是答案。可见找出并归纳文章的主旨并非是件容易的事。 2. 正确答案:B。 本题考核考生是否读懂了第3段的具体内容。奶油容易变酸是因为细菌在“奶油中比在黄油中繁衍起来容易得多”(B项)。这正是第3段、特别是该段最后一句所讲的内容(This means that in cream,the bacteria are free to grow throughout the mixture)。至于 A项“细菌在奶油中的分布比在黄油中均匀”,C项“细菌在奶油中依赖较少的脂肪生存”,和D项“细菌在奶油中产生较少的排泄物”等,虽然用了一些文章中出现过的词语,但并不是文章的原意,因此不是答案。 3. 正确答案:D。 本题考核考生理解文章细节的能力。试题问根据Brocklehurt的研究,用什么办法可以使奶油保鲜,文章的第4和第5段回答了这个问题。第5段中说:„ their products can be made resistant to bacterial attack through alternations to the food‘s structure. 这正是选项D(改变结构)的内容。至于A项“脱脂”、B项“灭菌”都用了文章中的词语,如 fat,bacteria 等,但都不是 Brocklehurt 研究的用以使奶油保鲜的办法,因此都不是答案。C项“脱水”虽然文章提到细菌喜欢在混合物中的水域里生活,但并没有说可用“脱水”的办法使奶油保鲜。 4. 正确答案:C。 本题要求考生根据上下文准确理解词义。colony 一词除有“殖民地”的意思外还有“聚居地,群体,菌落”等意思。从文章的上下文来看,这一个个既无法扩展又无法补充养料的 colonies,当然不是指那些“小球体”、“小隔离仓”,更不是指什么“水域”,而是指“细菌的群体,菌落”,即C项。要答对本题需要正确理解文章中的指代关系。指代关系可以由代词、同义词、反义词、上位词等来表示,个别的句子正是通过这种指代关系构成连贯的篇章的。正确理解指代关系是阅读理解能力的重要组成部分。 5. 正确答案:D。 本题考核的是文章最后一段的内容。文章说,Brocklehurst 相信,将乳状液,例如用于色拉的奶油,做成与黄油一样是可能的,关键是保持色拉酱的液态状,而不是将其变为固态的块状,也就是说,既能使奶油抗菌保鲜,又能使其保持液体状况,这项科研成果才能投入商业运用。因而D是答案。A、B、C这3项都不是文章的说法。 Example 4 内容大意 本文是一篇说明文。作者在本文中指出个人味觉在鉴别性品尝中不是十分可靠的。文章第一句开门见山,指出口味极具主观性。也正因为如此,所以不大会有人去调查一般人偏爱哪些食品。作者认为,就任何个人的爱好而言,充其量也只能说是他个人的意见。但由于可口可乐和百事可乐这两家公司推销自己的产品时十分积极,使人们不由得发问,对某种口味的嗜好在多大的程度上会促使人门只喝具有该口味的品牌的饮料, 为了回答这个问题,作者及其同事举行了一次品尝试验,看看那些自认为是专喝可口可乐或百事可乐的人能否在没有标明品牌的几种饮料中认出自己喜爱的品牌。 文章继而指出,自愿参加品尝试验的都是些要么非常喜欢喝传统的可口可乐或百事可乐。要么就热衷于喝低糖可口可乐或百事可乐的人。而且,这些人都认为在两种品牌的饮料中试尝出自己喜欢的品牌易如反掌。 参加测试活动的人最后定为19名专喝传统可乐和27名只喝低糖可乐的可乐饮者。他们先后试尝了4种没有标明品牌的可乐,然后被要求说出每次喝的可乐的品牌。事后对他们的答案进行了统计分析以便将他们的答案与随意猜测的可能结果进行比较。 要准确无误地确认4种样品的品牌不是一件轻而易举的事,但对这批十分自信的人来说似乎也不应该太难。结果表明,19名只喝传统可乐的人中只有7人能在4次品尝中正确认出自己的品牌,而27名低糖可乐的饮者中,只有7人能识别出自己喜欢的品牌。 虽然两组人的成绩要优于随意猜测可能得到的结果,但每组中都有将近一半的人会有两次或 用请下载www.docin.com/yongqing 两次以上的识别错误。还有两人没有一次答对。综观几轮品尝,一半参加品尝的人,第4轮和第1轮的品尝效果是一样的,因此味觉疲劳不是一个有影响的因素。总的看来,这次品尝活动的结果表明只有为数不多的百事可乐和可口可乐的饮者能根据口味和价格真正认出自己喜欢的品牌。 试题分析 1(正确答案:A。 本题是个局部性的问题,问的是进行这次试味活动的目的是什么。文章第1段第3句说:„we‘ve wondered how big a role„in brand loyalty。接着第 4句又说:We set up a taste test„in a blind tasting。这两句话提供了这个问题的答案。因此,本题的答案应该是选项 A:弄清口味在个人选用饮料时起什么作用。有53,的考生答对了本题。 2(正确答案:B。 本题是一个辨认重要事实的题目。读懂本文第4和第5段便可得出答案。由第4段第2句和第5段最后一句可见,正确答案应该是选项B。文中对参加品尝活动的人所作的选择的统计分析并未论及选项A、C和D的内容,因此是错的。 3(正确答案:C。 本题是一道判断题,要求学生根据第1段推断出正确的结论。第1段第1句的内容与选项C的意思相关,因此答案应该是选项C。第1段最后一句只是说要在一次不标明品牌的品尝活动中看人们能否凭自己的味觉真正识别自己喜爱的品牌,并没有说只有通过blind tasting才能分清百事可乐和可口可乐这两种饮料的爱好者。如没有看懂这一句,就会误选D项。文章第1段中并未谈及选项A和B,故这两个选项也是错的。 4(正确答案:D。 本题考核学生能否根据上下文推测 burnout 一词的意思。在阅读该词所在的句子时,即可看出 burnout 的前面有or 一词。Or 在这里的意思不是“或、或是”,而是“也就是说、换言之”,其作用是引出同位语以对前面的词语做进一步的阐释或者得更确切些。在本句中 or 引出 burnout 以进一步说明 fatigue 一词。因此,只要考生明白fatigue一词的意思,即可推出 burnout 一词在本文中的意思应该是“味觉疲劳’。所以选项D是正确答案。 5(正确答案:A。 本题问及本文的中心思想。本文的头两句己指出,本文最后一句亦指出只有极少数人能凭自己的味觉认出自己喜爱的品牌饮料,而本道题的4个选项中只有A项与这个内容相关,因此正确答案应该是A项。选项C虽提到taste and price,但与第5段最后一句的内容不符,因此是错的。文章未提及选项B和D的内容,故这两个选项也是错的。 Example 5 内容大意 岸本文是一篇论述美国公关公司为何落后的论说文。文章一开始就指出跨国公司的兴起、全球性的营销活动、新的通讯技术的出现和文化差异的日趋缩小,都使公共关系活动在全球范围内得到前所未有的发展。 本文的第2段论述说,现代的公关原本是美国人发明的,然而现在美国在这一领域的领先地位已开始受到其他国家公关事业迅速发展的威胁。例如,10年前世界上排位前5名的公关公司都是些美国公司,然而到了1991年只有一家是美国公司。文章继而指出,英国人在这方面不仅经验丰富,而且颇有创造性。不久前进行的一次调查表明一半以上的英国公司把公关列入他们公司的计划里。而相比之下,通常只有三分之一的美国公司才这样做。这样下去,可能用不了多久伦敦就会代替纽约成为世界公关事业的中心。 美国在全球性的公关竞赛中为什么会落伍呢,文章的第3段分析指出,美国之所以落伍,原因有3个。第1个原因是美国人总体说来较为偏狭,只关心本国、本地区的事情。例如,对世界地理的了解从来不是美国人的强项。第2个原因是因为美国人在掌握外语方面要远比他们的欧洲和亚洲同行落后。Burton(Marshall公司的美国雇员中只有不到5,的人 掌握了一门外语。另一家大公司,即 Ogilvy and Mather,情况也大体相同。与此 用请下载www.docin.com/yongqing 相反,在一些欧洲公司中至少有一半或更多的雇员懂一门外语。第3个原因则是那些外国公关公司的雇员比较注意跟踪国际事务。例如,在金融界公关领域里工作的大多数美国人通常只读《华尔街日报》,而他们的欧洲同行不仅读《华尔街日报》,而且还看在美国很少有人问津的伦敦《金融时报》和《经济学家》杂志。 文章最后一段指出,美国的公关界应从 CNN的老板 Ted Turner 说的一句话得到启示。Ted Turner不久前宣布说,在CNN的新闻广播中将不再使用“外国”这个词,因为全球性的通讯已使世界各国相互依存,不再存在有什么可称之为外国的东西。 试题分析 1. 正确答案:D。 本题考核正确理解事实细节的能力。第2段第1句指出,尽管现代的公关事业是美国人的发明,然而由于其他国家在这方面所做的努力而使其领先地位受到威胁。选项D的意思与此相符,故答案应该是D项。选项A和B都源于第1段,然而这一段所说的意思是,“„„新的通讯技术的出现和文化差异的日趋缩小,使公共关系活动在全球范围内得到前所未有的发展”,这些并不是“领先地位受到威胁”的原因,因此选项A和B都不是答案。文章中未提及C项,因此C项也不是答案。 2. 正确答案:B。 本题是针对第2段的一个局部性的问题。本段中的第3句指出,英国人在公关方面不仅经验丰富,而且颇具创造性。第4句则说,一半以上的英国公司把公关列入他们的计划里,而通常只有三分之一的美国公司这样做。因此,很显然英国公司比美国公司更为重视公关。伦敦取代纽约成为世界公关事业的中心也只是时间问题。因此答案是B项。文章内容未涉及选项 A、C和 D,因此这 3个选项都不是答案。 3. 正确答案:A。 本题要求考生根据上下文推测词义。要准确推出provincial一词在本句中的词义,就必须读懂该词前后部分的意思。前一句问及美国在全球性的公关竞赛中为什么会落伍,回答是,因为美国人end to be provincial。Provincial 以后的部分则进一步说美国人只关心本国、本地区的事情,而且对世界地理了解不多,又很少人掌握外语。由此可见,美国人对本国以外的事情了解甚少,观点和见解肯定有局限性。因此,答案只能是A项。Provincial 一词的语境并无与选项B、C和D相关的内容,故这3个选项不可能是答案。根据上下文正确推测词义也是一项很重要的阅读能力。 4. 正确答案:C。 本题考核学生推论出隐含表达的意思的能力。本文第3段主要列举了美国在全球性的公关竞赛中落伍的3个主要原因,即美国人总的来说,比较偏狭,懂外语的人极少,并且很少阅读其他国家的各种有关报刊。因此,和他们的欧洲同行相比,当然就会是less sophisticated,因此答案应该是C项。由于美国人比较偏狭,美国公关行业里只有5,不到的雇员掌握一门外语,并不是说所有雇员“至少流利地掌握一门外语”,所以选项A不是答案。此外,他们多数人只读《华尔街日报》,阅读面很狭,所以选项 D也不是答案。文中只说美国人对世界地理了解甚少,并不是一无所知因此选项B不是答案。 5. 正确答案:A。 本题是一个较为容易的推理题,涉及文章的最后一段。本文最后一段,一开头就问,人们应从 CNN的老板 Ted Turner 所说的话得到什么启示,Ted Turner 说,由于全球性的通讯设施极为发达,世界各国之间的关系非常密切,已形成相互依存之势,因此不再存在有什么可称之为外国的东西。言下之意是美国的公关公司应建立对国际事务的关注。由此可见,答案应该是A项。 Example 6 内容大意 本篇属报导性体裁的文章,介绍了心理学领域的敏感问题:该不该给学生某些外界的鼓励。在这个问题上心理学家持两派截然不同的意见。研究行为与结果之间的关系的行为主义者认为:奖励能促进提高工作和学习表现;而研究精神生活各个方面的认知学派研究人员则认为:由于奖励促使学生对别人给予的肯定和礼物有依赖心理,常常会破坏他们 用请下载www.docin.com/yongqing 的创造性。后者博得了许多人的支持,尤其是在教育界。但据《个性和社会心理学杂志》6月一期刊载的研究论文称:谨慎地使用少量的金钱奖励能刺激生的创造积极性,这说明适当的物质刺激确实有助于启发创造精神。特拉华大学的艾森伯格先生也有同感,不过他进一步指出:“如果对不佳的表现也给予鼓励或者造成对奖励的过分期待,则可能扼杀创造性。”实际上艾森伯格先生更重视奖励带给学生的有害的副作用,为了说明自己的观点,他指出主要的几所大学正设法严格升级标准并恢复留级制度。既然是客观报导,作者在结束本文时又援引了特拉华大学这位心理学家的话,以补充赞成使用奖励办法一方的意见。 试题分析 1. 正确答案:D。 本题是个局部性问题,考查对文章第1段的理解,也可以说考查考生是否看懂了第1句话,因为第1句话是主题句,它概括了本段的意思、第1句话说:“来自外界的奖励,无论是热情的赞誉或是钞票,对学生的学习积极性和创造性究竟有什么影响,在这个问题上心理学家持有截然相反的观点(take opposing views of)”。而本题问“心理学家在对待„„的态度上是有分歧的”,正确答案自然是D项:外界的奖励对学生的学习表现有什么影响。我们知道,背景知识在阅读理解中起着十分重要的作用。但是,阅读中还是要依靠语言能力、读懂文章内容,才能正确答题。 2. 正确答案:B。 本题也是个细节问题,问及教育工作者对待采用外界奖励作何反应,其答案在第二段第一句话:“后一种观点赢得许多支持者,尤其是在教育工作者当中。”也就是说,教育工作者支持后一种观点。后一种观点是:“由于奖励促使学生对别人的赞许和馈赠有依赖心理,奖励常常会破坏创造精神。”也就是说,后一种观点不主张采用外界奖励刺激学习的积极性,对外界奖励持怀疑态度。所以,B项是本题的正确答案。C项说 “他们赞同他们(they approve of them)”,其中有两个代词,必须弄清指代关系,才算看懂了选项的意思。They当然是指 many educators,them是指 external rewards。如果把them理解成many supporters(则可能误选C项。由此可见,阅读中准确地弄清代词的指代关系也是正确理解文章和题目的重要环节。 3. 正确答案:C。 本题检查考生对本文第3段的理解,第3段的主要内容是直接陈述文森伯格的观点:“如果孩子们知道他们为得到奖赏而学习,并能专心去完成相对而言具有挑战性的任务,他们则能表现出最大的创造力。但是对欠佳的表现给予奖励或造成对奖励的过分期待,很容易扼杀创造力”。意思是说奖其所值才能最大限度地提高学生们的创造性,因此本题的正确答案是C项。如果对文章没有正确理解,只受到个别字句的吸引而主观猜测,后就会误选B项和D项。 4. 正确答案:A。 本题依据文章的展开考查学生对第4段的理解,关键词是第2句中的the latter point。首先,艾森伯格赞成用奖励的办法;其次,他主张奖其所值,不能无条件地施予奖励。the latter point 指的是后一半观点。如果考生在阅读时读懂了这层含义,便能推断出正确答案A项:各主要大学正设法严格升级标准是因为他们认同奖励欠佳的学业表现会扼杀学生的创造力这一观点。反之,如果在阅读时只注意读懂每个句子,没有顺着作者的思路展开,不明白艾森伯格为什么要举各主要大学采取改革措施为例,在做本题时会感到茫无头绪,于是只能靠主观猜测了。因为孤立地看,C项和D项也不无道理。 5. 正确答案:B。 本题考查猜测词义的能力。猜测词义应在文章里找到提示,所以考生在答题时一定要仔细阅读上下文,本题的提示就在紧随其后的 in which 从句中。分析误选其他几项的原因,有些考生可能未看懂这一从句中的 receive performance-based points toward valued rewards, 这里的 points 是“点数”或“分数”的意思,整个从旬意为:“(采用这种办法时)学生解决一定要动脑筋才能解决的问题(handle challenging problems),同时根据付出的大小得到一定的分数直到获得宝贵的奖赏”。可见,token economies正是“奖 用请下载www.docin.com/yongqing 励制度”的意思。归根结底,任何一项阅读技能的提高都缺少不了对词汇的准确理解和掌握,而一个词在不同的上下文中其词义又是千变万化的。所以,要大幅度地提高阅读理解能力,一定要进行大量的阅读,在阅读中尽可能多地熟悉词的各种用法。 Example7 内容大意 这是一篇报道体裁的文章,作者在报道巴西成功地控制了人口增长的同时,精辟地分析了个中原因,从而向世人举荐了一种新的降低人口出生率的思路。巴西是发展中国家成功地降低了人口出生率的国度之一,不过它的成功与其说是计划的结果不如说是意外的收获。许多国家均为降低出生率做出了努力,然而巴西下的功夫不大却取得了较好的效果。 巴西的人口增长率已从50年代的年增长2(99,降至80年代的年增长1(93,,目前巴西妇女平均每人只有2(7个孩子,这个数字到了90年代还要下降。这个成就是其他许多第三世界的国家羡慕不已的。 在哈佛的马丁先生把巴西的这一成果归功于肥皂剧和自70年代开始实施的分期付款购物计划。巴西是世界上最大的肥皂剧出产国之一。仅巴西最受欢迎的Globo电视联播公司一家每周6个晚上每晚播放3个小时的肥皂剧,还有其他3家电视公司每晚至少播出1小时的肥皂剧。这些肥皂剧多以住在大城市里的富人或高消费人群为描述对象。马丁说:“那些肥皂剧虽然并未刻意要传递有关人类繁衍问题方面的信息,它们只是描述中、上层阶级的价值观,比如不要许多孩子、采取不同的态度对待性生活、妇女要工作等等,但是肥皂剧把中、上层阶级的形象传播到巴西的各个角落,让人们知道尚有别样生活方式和别种价值观念的存在。 同时,分期付款购物计划鼓励穷人参与消费,使人们的消费方式产生了很大的变化。而消费能力的提高与无节制的生育是相互排斥的,你要高消费则必须节制生育。所以,肥皂剧和分期付款制度渐渐地改变了人们的生育观念,结果巴西的人口出生率全面下降。 试题分析 1. 正确答案:D。 本题考核辨别重要细节的能力。文章说到巴西成功地降低了人口出生率,但与众不同的是巴西的成功不是靠国家的计划生育政策取得的。因此,巴西靠什么手段或途径取得成功便是重要细节。本文的第1句话开门见山:“巴西已经成为第三世界国家中了不起的成功者”。在哪方面,“在降低人口出生率方面”。靠什么手段,“与其说靠国家计划不如说出于偶然”(more by accident than design)。本题的答案是D项,因为by chance 是 by accident 的另一种说法。如果不是确切理解文章原意,而是凭想当然,就可能误选A项“通过对公民进行教育”或B项“通过周密的生育计划”,也可能凭一知半解而误选D项“通过开发电视节目”。这说明确切理解文章内容的重要性。 2. 正确答案:C。 这是一道考核推断能力的题目。文章只有两个地方提到第三世界国家,一个在第1段,谈的是第三世界国家在控制人口方面做出了努力,而巴西没费劲效果却很好,言外之意是第三世界国家的效果差一些。另一个地方在第2段最后:“(巴西出生率下降的)数字到90年代还会有发展,这是个了不起的成就,是许多第三世界国家羡慕不已的”(an achievement that makes it he envy of many other Third World countries)。言外之意是那些国家还没有找到控制人口的得力措施,这也正是本题的答案C项。本题比较容易。 3. 正确答案:A。 本题考查猜测词义的能力。阅读时不可能不遇到没学过的语言现象,因为语言是十分活跃的信息载体。我们应该学会从上下文中辨别词义。例如本文中的puts it down to,可以用上挂下连的方法准确地判断它的含义。首先要弄清这个短语动词中的it指什么,它指上一段最后一句话中的 an achievement;然后看这个短语的下一句:Both played an important, although indirect, role in lowering the birth rate. Both指上句话中的 soap operas和 instalment plans。现在可以判断 put it down to 的含义了,A项;“归功于”便 用请下载www.docin.com/yongqing 是答案。而B项“把它定为„„的理由”、C项“把它总结为”和D项“把它比做”都是讲不通的,因此均不是答案。 4. 正确答案:B。 本题是一道概括性的问题,不可能从某一句话中找到答案,必须在理解全文的基础上找到问题的出处,即该问题是针对哪里提出的。然后仔细阅读那个段落,从中悟出正确答案。本文的第1段讲的是控制人口出生率的一种现象,巴西未必做出过多少努力,可是它是这方面的成功者,正如中国一句老话所说:无心插柳柳成荫。第2段介绍巴西人口出生率下降这一事实,第3段探讨巴西人口出生率下降的原因,第4段则开始分析肥皂剧在降低人口出生率方面所起的作用。本题谈论的正是肥皂剧的作用——帮助降低人口出生率。为什么肥皂剧有助于降低人口出生率,要回答这个问题,就要正确地概括出第4段的意思。“虽然肥皂剧从未有意传达有关生育方面的信息,它们描述中层和上层阶级的价值观,诸如不要太多孩子、对待性生活的不同态度、妇女工作等等,并把这种形象传递到各个角落,让人们知道有别样的生活方式和别样的价值观念存在,同时给予那些生活方式和价值观念美丽的包装,使其对人们有吸引力。”这段话的概括就是本题的答案B项:“因为肥皂剧逐渐改变了人们的生活方式”。 如把问题的出处放在了第3段,就会错选A项。既然几个电视网络均长时间地播放肥皂剧,它们可能是占用了人们的时间。如果找对了问题的出处,未领会整段的用意,受到本段个别词语的吸引,则可能误选C项。想当然地以为电视总是会宣传节制生育的措施。就会误选D项。 5. 正确答案:B。 本题考查理解文章隐含意思的能力。从本文结构上分析,最后一段阐明分期付款购物计划如何影响了出生率的下降,最后一段也是本文的结尾段落。作者通篇引Martine对巴西人口出生率下降的看法:巴西降低人口出生率的做法是成功的,与其他第三世界国家不同的是,巴西采取寓教于乐的手法,收到意想不到的效果,从根本上改变了人们的观念。同时从引导消费入手,改变消费模式,使穷人也加入消费者的行列。而消费与无节制的生育不相容的。在文章的最后一段Martine实际上道出了他对巴西人口增长率下降的结论性的意见。因此,本题的答案是B项“消费欲望有助于降低出生率”。 Example 8 内容大意 这是一篇关于机器人的文章。文章一开始就指出人的大脑是极为复杂的,由无数细胞组成,而且每个细胞可能还会有上千个接头或端子。由于大脑的构成如此复杂,使人们感到泄气,以致放弃了制造具有人类大脑功能的机器人的想法。然而,现代科技发展如此迅速,使人们认为或许10至20年后就有可能装配出一台和人的大脑一样复杂的机器。当然,要使这台机器具备智能还需设计相关软件或硬件加以改进,但所有这些只不过是时间问题 作者接着指出,在不远的将来,即几十年后,就会出现可与人类媲美,甚至超过制造它们的人类的硅(即电子计算机控制的)机器人。一旦硅机器人超过人类,它们将能自行设计,亦即自行繁殖,并最终结束以碳元素为基本元素的人类对地球多年来的统治。至此,人类再也不能自称是这个已知宇宙的最高智慧的代表了。 作者最后指出,随着机器人智慧的不断增长直至达到可与人类智慧相比的水平,以及机器人的大量使用,从而使其成本降低,人们将可利用它们来首先开拓地球上那些环境恶劣,不适宜人类生存的疆域。进一步展望未来,通过这个新时代所产生的巨大财富和新技术的结合,人类可望在太空建立一个可供千百万人居住的世界。 试题分析 1. 正确答案:C。 本题考核正确理解重要细节的能力。选项 C的意思与本文第 1段最后一句中的 to render intelligent by loading„ or by altering the architecture 的意 用请下载www.docin.com/yongqing 思相符,因此选项 C是正确答案。在第 1段里虽然提到 in only 10 or 20 years,但这里只是说或许在 10至20年后有可能装配出一台和人一样的机器人,并未说只要努力工作10至20年便可使机器人有智慧,故选项B不是答案。虽然在A和D这两个选项中有些单词亦在文中出现,与题干所问的问题毫不相干。 2. 正确答案:A。 本题要求考生根据文章内容进行正确的推论。文章的最后一段说:„ we may use them to expand our frontiers„ harmful to ourselves和 Thus deserts may bloom„ be mined(人们将可利用它们来开拓在地球上那些环境恶劣,不适宜人类生存的疆域,使沙漠变绿洲并使海底资源得以开采)。由此可推断出正确答案应该是A项,即相信智慧机器人将造福人类。由于文中提到硅机器人将结束人类多年来对地球的统治,故选项B有一定的干扰作用。文章开头虽然提及“人们曾放弃制造具有人类大脑功能的机器人的想法”,但这一点已被下文所否定,因此选项D也是错误的。选项C在文章里找不到根据,因此不是答案。 3. 正确答案:D。 本题要求考生根据上下文推断 carbon 在本文里的意义。文章第2段指出在不久的将来将会出现可与人类媲美,甚至超过人类的机器人,而且到那时硅机器人将结束以碳元素为基本元素的人类对地球多年来的统治,人类将再也不能自称是这个已知宇宙的最高智慧代表。由此可看出本文的carbon不是指一般的碳元素,而是比喻人类,故正确答案应该是D项。如未仔细阅读课文,只是根据常识推理,就会错选B项。 4. 正确答案:A。 本题是个局部性的问题,考核学生对第 3段第 1句的理解,即 As the intelligence of the robot increases„ we may use them to expand our frontiers„,句中指出一旦机器人的智慧达到人类的水平,成本又降低到可接受的程度,便可利用它们开拓„疆域,故本题的答案是A项。该句虽然提到了withstand environments,harmful to ourselves,但机器人能在恶劣环境下工作的这一特点是原来就有的,因此B项是错的。文章中虽提到 a machine as complex as a human brain(L(5),但与题干所问的问题没有联系,因此C项也是错的。 5. 正确答案:B。 本题要求考生根据文章内容进行正确的推论。文章第1段第2句后半句和第1段第3句前半句说:„ but now that we have grown used to moving forward at such a pace we can be less sure(Quite soon,in only 10 or20 years perhaps,we will be able to assemble a machine as complex as the human brain… „„然而,现代科技突飞猛进的发展使我们不再认为没有可能制造有人类大脑功能的机器人。在不久的将来,或许只需10至20年的时间我们将能装配出一台和人的大脑一样复杂的机器„„。根据这两句可推断出答案应该是B项。选项 A、C和 D都不是答案,因为文中并未提及这些问题。 Passage 9 DCCCB Passage 10 AAAAA passage11 评析:这篇文章介绍了美国高校的一些情况,其中谈到了学生的成绩将会直接影响他们今后的工作,谈到了学生在校就学期间担任社会工作也有利于将来的职业选择,还谈到了学生组织在维持纪律方面所作的工作。 1(B. 这是一道计算题,根据第一段的三,四句便可算出准确数字。 2(B. 本文第一段的结尾有这么一句“to move between one university and another”,从而就否定了答案A和C。“during his degree course”中的“course”一词为单数形式,便又否定了答案D。 3(A. 根据第二段第一,二句,学生成绩的优劣直接影响到学生所找的工作的好坏,这就使学生感到一种压力,一种紧迫感。其他答案均不合题意。 4(C.短文结尾句已经明确告诉,在大学任社会职务的学生都会受到尊敬 (much 用请下载www.docin.com/yongqing respected),而且也有利于今后择业或谋职。 5( B.从第二段的第四句得知,学生组织在维持纪律方面作出了卓有成效的工作,后面还举例说明了这一点。“Evaluate”意为“对(((作出评价”。又根据文章内容,当有人发现学生舞弊时,才会诉诸其学生法庭,故能推断出B是符合题意的。 passage12 评析:这是一篇介绍一位名叫Florence的老太太,她已年过七旬,但仍孜孜不倦读书拿文凭,真可谓活到老学到老。 1(B.文中所说这位老妇人已经77岁,却还在读书拿文凭,学习精神实在可佳。她还说,“The house takes second place while I put the studying first (我总是把学习放在第一位,做家务就只能放在第二位了)”。从这句话看得出这位老妇人是多么渴望知识。A和C自然可以排除,而D也不符合题意。 2(D.文章的第二段明确告诉,她虽然考上了Manchester University,但由于第一次世界大战,她选择了工作,每周一英镑薪水,这在当时战乱期并不算低。她很满意那份工作和自由,于是就拒绝了(turn down)Manchester University为她保留的学籍。 3(A.本题问她学习怎样,我们知道学习对于一个77岁的老人来说,无疑具有一般人所想象不到的困难。她觉得“tired more frequently”,“can‘t do more than an hour‘s work at a time”,“The memory is shocking”所有这些句子都说明,由于年龄的关系,她已力不从心。 4(D.要写论文,必须思路清晰,结构严谨,有理有据才行,但根据文章第四段内容,她却是“just let myself go and get excited”,“feel more emotionally than I do mentally”思想不能控制感情,实际上就是她很难来组织论文的结构和观点。 5(C.本题可以从第五段第一句中找到证据:Florence认为她自己是相当的“lazy”。 passage13 评析:这篇短文饶有趣味,文中谈到世上有两种类型的人,他们的生活观和处世哲学截然不同,因而得出的结果也不一样。一种人整日开心惬意,而另一种人却总闷闷不乐,苦恼抱怨。 1(D.本题的答案参看第二段的三,四两句话,那些感到闷闷不乐的人,他们的想法与言谈和常人完全不同。因此,他们总是牢骚满腹,玩事不恭,对人抱怨,处处使自己与众人格格不入。足见,他们通常养成了一种爱挑剔的习惯。 A和B回答不够具体,C在文中未提及。 2(B.本题是道词组释义题,做这种题型要根据上下文以及词组中的某个关键词来判断。这里的“sour the pleasures of society”中的sour,意为“对((((((苦恼或不满”。弄清该词义,一切问题也就迎刃而解了。 3(A.本题问“根据短文下列哪句话不真实,”,碰到这类题目时,务必要根据文中的观点,千万不要凭自己的好恶来决定。根据文章第二段第六句“If this turn of mind were founded in nature, such unhappy persons would be the more to be pitied”,可以断定A是不真实的。 4(B.这是一道词组解释题。主要是想检查考生是否能理解好“scarcely”一词,这是一个否定意义的词,A和C自然可以排除。“scarcely”等于“almost not”这样又排除了D。由此可见,选择答案时一定要慎重。 5(A.如果这种闷闷不乐的人不改掉如此恶习,作者的解决方法就是劝人们不要和他们交往。本题答案的证据在文中最后的两句结尾语。 passage14 评析:这是介绍如何使用我们大脑的科普文章,本文论述了Taiju Matsuzawa教授对人脑研究的结果,它表明大脑保持活力防止萎缩的最佳途径就是不停地用脑。 1(B.根据第二段最后一句,Taiju Matsuzawa 教授希望找出“how the process of aging could be slowed down”,也就是如何使人们延长寿命。 2(C.本题问他的研究结果是以什么为基础的,从第三段的“set about measuring brain volumes of a thousand people of different ages and varying occupations”中,就能得知本题的答案。 用请下载www.docin.com/yongqing 3(D.这位教授的试验说明了有些人大脑的萎缩要比另一些人快。根据文章倒数第二段列举了Taiju Matsuzawa教授的研究结果,说明了哪种人衰老得快些而哪种人则衰老得慢些。 4(C.这是一道词义解释题,文中提到Taiju Matsuzawa教授利用计算机对人脑进行测量,既然实验对象是人,这里的“subject”也应是指人,而且还有其下文“in their thirties”,这便更加清楚地说明了这一点。 5(A.根据本文,说说下列哪种人衰老慢些,从本文的倒数第二段看出律师的衰老要慢。 passage15 评析:这是一篇介绍亚洲学生在美国如何学习和奋斗以及受到何种待遇的文章,文中列举一些事实来分析美国社会对于亚裔学生的不公正待遇和歧视。 1(C(亚裔美国人在校期间学习优异,成绩突出,但他们依然担心在社会上将会受到不公正的待遇。从文章结尾段第一句中可以看出这种心理上的担心。 2(A(第一段和第二段都告诉读者,亚洲学生在去美国之前就已有了坚实的数学基础,但英语懂得不多,因此在理科方面出成果要比在文科方面出成果容易得多。由于文化背景的不同,亚裔学生的父母对孩子的要求也要严格,督促也更多。 3(B(亚裔学生很少攻读文科(human science)专业主要是因为文科方面的成就比理科少,多靠别人的主观评价,而且由于长期形成的对少数民族的歧视使这些亚裔学生担心将会受到不公正的待遇。 4(A(从结尾段第一句可以看出本题的答案。 ? take on…呈现,增加,接纳,采用。E.g. The city took on a festive air. 城里呈现一派节日气氛。 She took on five pounds in one month. 她一个月增加体重5磅。 The bus stopped to take on the passengers. 公共汽车停下载客。 He soon took on new ways of life. 他不久就开始了新的生活方式。 ? take over…把…带过去,接收。E.g. She took her children over to her mother‘s. 她把孩子带到外婆家去。 The new department head took over yesterday. 新的部主任昨天接任。 ? take to…开始,照料。E.g. Xiao Li took to studying English with great zeal. 小李开始以极大的热情学习英语。 The mother dog took to the puppies jealously. 狗妈妈小心翼翼地守护着小狗。 ? take up for…站在…一边。E.g. He has no body to take up for him. 他没有人支持。 ? take up with…致力于,开始与…来往。E.g. He took up with natural sciences. 他致力于自然科学。 Jack took up with a girl of whom his family disapproved. 杰克开始与一个姑娘交朋友,可他家人不赞成。 5. D.短文从头到尾见不到作者主观的评价,而是客观地陈述亚裔学生在美国的真实情 况,不带有一点儿个人的感情色彩。 用请下载www.docin.com/yongqing 四级词汇技巧 Yu Chunhua 一、 四级考试对词汇有何要求, 根据最新的《大学英语教学大纲》(修订本)和《大学英语四级考试大纲》的要求,学生必须“领会式掌握4200单词(其中复用式掌握的单词为2500以及一定量的常用词组,并且有按照基本构词法识别生词的能力”。 二、 词汇题的形式与特点 A. 语义题 a. 同义词、近义词类 同义词、近义词类是大学英语四级考试词汇部分的一个难点,它的困难之处在于相似的词汇太多,而词义间的差异又不易区分。在解这类试题时,应当首先从词汇的意义入手,了解词义的内涵和用法,注意结合具体的语言环境,掌握词义在具体的文章、段落和句子里的特殊用法,从而分辨和体会其区别。请看下列试题: 1)America women were _______ the right to vote until 1920 after many years of hard struggle.(CET-4曝光题——1991年6月,No.59) A) ignored B) neglected C) refused D) denied 2) In the advanced course students must take performance tests at monthly ______. (CET-4曝光题——1993年6月,No.60) A) gaps B) intervals C) length D) distance 3) The ______ of blood always makes him feel sick. (CET-4曝光题——1995年6月,No.64) A) sight B) view C) look D) form 4) Although they plant trees in this area every year, the tops of some hills are still ______.(CET-4曝光题——1999年6月,No.49) A) blank B) hollow C) vacant D) bare 题1)的四个选项意义相近,都有“否定”的含义,因此必须进一步研究题目的结构。从语法结构上来看,这是一个被动句,空格中的动词必须是能带双宾语(American women和the right)的动词。D项deny意为“拒绝给予”,是双宾语及物动词。句中American women原为deny的间接宾语,在其被动结构中转化为主语;the right为deny的被动结构中的保留直接宾语;A项ignore意为“忽略,不管”;,项neglect意为“疏忽,忽视”,它们都不是双宾语及物动词;C项refuse表示想要但遭到别人的拒绝,它可作双宾语及物动词,如:He refused us admittance.他拒绝让我们入场。但refuse没有“拒绝给予”的意思,故答案选D。 题2)的四个选项中都含有“距离”的意义,但B项intervals意为“间隔”,at intervals表示“每隔„时间(或距离)”,at monthly intervals意为“每隔一个月”;A项gap有“间隔,间隙”或“差距”之意,但它不可与at连用;C项length意为“长度”;at length意为“详细地”;D项distance意为“距离;远方”,at distance意为“在一定距离外”,故答案选B。 题3)的A、B、C三项都有“看”的意思,但它们的涵义却不尽相同。A项sight指“看到”;B项view指“视域”;C项look指“(主动地、有意识地)看”;D项form意为“形状、形式”,血是没有固定的形状的,故答案选A。 题4)的四个选项词义相近。A项blank意为“空白的,空的”,指上面没有写字或做记号的空白表面,也指空着准备写字或做记号的空白处;B项hollow意为“中空的”,指某物中间是空的;C项vacant意为“空缺的,空的”,强调某物或某个位置没有被占用;D项bare意为“光秃秃的,没有遮盖的”,指脱光了的、(叶)落光了的或(树木)伐光了的。如:a bare hillside“光秃的山坡”。根据题意,D项最为恰切。 b. 相似词类 相似词容易混淆,这类词的误用主要是由于拼法相近、词根相同和不规则变化中有相同拼法而引起的。请看下列试题: 用请下载www.docin.com/yongqing th1)Many Europeans _____ the continent of Africa in the 19 century. (CET-4曝光题—— 1992年6月,No.56) A) exploded B) expanded C) exposed D) explored 2) In Britain, the best season of the year is probably ______ spring. (CET-4曝光题—— 1995年6月,No.65) A) later B) last C) latter D) late 3) In general, the amount that a student spends for housing should be held to one-fifth of the total _____ for living expenses. (CET-4曝光题——1997年6月,No.33) A) acceptable B) available C)advisable D) applicable 4) Our new house is very _____ for me as I can get to the office in five minutes. (CET-4曝 光题——1999年元月,No.69) A) adaptable B) comfortable C) convenient D) available 题1)四个答案的前三个字母都相同,可是它们的意义却大不相同,D项explore 意为 “勘探,探测”;A项explode意为“使爆炸”;B项expand意为“扩大,壮大”;C 项expose意为“揭露,暴露”;ABC三项不合题意,故答案选D。 题2)的四个选项看上去都是一样,可是它们的意义却大有区别。D项形容词late 作“晚的”讲,意指 happening or done after the usual or normal time,如in late autumn (在 晚秋);A项later作形容词时,表示“更迟的、更晚的”之意;B项last spring 意为“上 一个春天”,本题讲的是一般情况;C项latter意为“后面的,后者的”,故答案选D。 题3)的四个选项形式上也相似,但A项acceptable意为“可接受的”;C项advisable 意为“可取的,适当的”;D项applicable 意为“能应用的,可适用的”,A、C、D三项均 不合题意。B项available本意为“可得到的,有用的”,此处可引申为“可供花费的”,正 合题意,故答案选B。 题4)中A、D两项和B、C两项形式上相似。A项adaptable 意为“可适应的;可 改编的”;B项comfortable意为“舒适的,舒服的”;D项意为“可得到的;有用的”;A、 B、D三项均不合题意。C项convenient 意为“便利的,方便的”,正合题意,故答案选C。 B(搭配题 英语中的词组是固定搭配,它不能随意拆开,取舍或更换,要完整地记忆和使用。 搭配题据实际情形可再分为短语动词类、名词短语类、形容词短语类以及介词短语和短语介 词类四种形式。 a(短语动词类 这一类考题主要目的是要求学生熟记并学会使用一定数量的动词与介词、副词式名 词的搭配。测试短语动词的题目在词汇、语法项中占有很大的比例。请看下列试题: )The French pianist who had been praised very highly ______ to be a great disappointment. 1 (CET-4曝光题——1993年6月,No.56) A) turned up B) turned in C) turned out D) turned down 2) The destruction of these treasures was a loss for mankind that no amount of money could ______.(CET-4曝光题——1995年6月,No.60) A) stand up to B) make up for C) come up with D) put up with 3) The soldier was ______ of running away when the enemy attacked. (CET-4曝光题—— 1997年6月,No.41) A) scolded B)charged C) accused D) punished 4) Frequently single-parent children ______ some of the functions that the absent adult in the house would have served. (CET-4曝光题——1998年6月,No.45) A) take off B) take after C) take in D) take on 题1)的四个选项中,C项turn out意为“生产;结果是,(最后)证明是”,这里取“结 果是,(最后)证明是”这意,全句意为:那个曾经受到很高评价的法国钢琴家结果却使人 们大失所望。A项turn up意为“出现;开大”B项turn in 意为“交还”;D项turn down意 为“拧小;拒绝”;A、B、D三项均不合题意,故答案选取C。 题2)中的四个选项都是由“动词+副词(up)+介词”构成的短语动词。B项make up for 意 为“补偿,弥补”,正合题意;A项stand up to 意为“经得起‖;C项come up with意为“提 用请下载www.docin.com/yongqing 出,提供”;D项put up with意为“容忍,忍受“,A、C、D三项均不合题意,故答案选B。 题3)中的四个选项都要求后面跟名词加介词,这类短语动词形式为:动词+名词+介词,名词一般为人称代词;C项accuse„of 意为“控告”,正合题意;A项scold后接介词for,意为“责骂”;B项charge后接介词with,意为“控告”;D项punish后接介词for, B、D三项都不能接介词of构成短语,故答案选C。 意为“惩罚”,A、 题4)的四个选项中,D项take on意为“呈现;承担;雇用”,这里取其“承担”之意,全句意为:单亲家庭的孩子经常承担一些义务,而这些义务原本是由他的父亲或者母亲履行的。A项take off意为“脱下(衣服等);(飞机等)起飞”;B项take after 意为“(性格,长相等)像;模仿”;C项take in 意为“接受,领会,理解”;A、B、C三项均不合题意,故答案选D。 b(名词短语类 一定的名词与一定的介词搭配构成特定的名词短语。常跟名词搭配的介词有on, to, in, from, with等。请看以下试题: 1) A ______ to this problem is expected to be found before long. (CET-4曝光题——1998 年元月,No.48) A) result B) response C) settlement D) solution 2) Does everyone on earth have an equal right ______ an equal share of its resources? (CET-4 曝光题——1998年6月,No.55) A) by B) at C) to D) over 题1)四人选项中,A、C两项后常接介词of ,不接介词to,可首先排除;B、D两项虽都可接介词to,但B项response意为“反应”,与题意不合;D项意为“解答,解决办法”,正合题意,故D项为正确答案。 题2)中right后应跟介词to 与之搭配,表达“„的权力”之意。全句句意:地球上的每个人是否都有分享资源的平等权利,故C项为正确答案。 C(形容词短语类 与名词一样,一定的形容词跟一定的介词搭配构成具有特定意思的形容词短语。常与形容词搭配的介词有to, with, for, at, form, in, of, about等。请看下列试题: 1)By 1929, Micky Mouse was as popular ______ children as Coca-Cola. (CET-4曝光题—— 1997年6月,No.59) A) for B) in C) to D) with 2)Tom is very disappointed ______ the results of the exam. A) with B) for C) toward D) on 题1)的四个答案中,只有答案D项with才能与popular构成形容词词组,意为“受„欢迎的”,题中Micky Mouse即“米老鼠”,,Coca-Cola即“可口可乐”,其它三个答案都不能与popular构成搭配。 题2)的四个答案中,只有A项with才能与形容词disappointed构开形容词词组, 表达“对„失望”之意。其它三项均不能与disappointed构成搭配。 D(介词短语或短语介词类 特定的介词与特定的名词或再加上介词可以搭配成介词短语或短语介词,每年的考 题都有这方面的内容。请看下列试题: 1) We regret to inform you that the materials you ordered are ______. (CET-4曝光题— —1993年6月,No.63) A) out of work C) out of reach B) out of stock D) out of practice 2) Being a pop star can be quite a hard life, with a lot of traveling ______heavy schedules. (CET-4曝光题——1999年6月,No.50) A) with regard to B) as to C) in relation to D) owing to 题1)的四个答案,是介词与不同名词的搭配,因此就构成不同意义的介词短语。 B项out of stock意为“脱销”,正合题意;A项out of work 意为“失业”;C项out of reach 用请下载www.docin.com/yongqing 意为“够不着的”;D项out of practice意为“缺乏练习”,A、C、D三项均不合题意; 故答案选B。 题2)四个选项均为短语介词。D项owing to 意为“因为,由于”,正合题意,全 句句意:因为繁忙的日程需要安排大量的旅行,所以一个流行歌星的生活相当紧张。A 项with regard to 意为“关于,对于”;B项as to 意为“至于,关于”;C项in relation to 意为“关于,有关;联系到”;A、B、C三项均不合题意。故答案选D。 三、 词汇试题解题技巧 A(根据构词法识别生词 英语中常见的有三种构词法:转化法、派生法和合成法。熟悉并掌握一定的构词对识别生词、扩大词汇量有明显的帮助。在做词汇题时,构词法知识也可起作用。请看下列试题: 1) Jane was hit on the head by the robber and was knocked ______. A) mindless B) unaware C) brainless D) unconscious 分析:在这道题中,四个选项都是派生词,即通过加前缀或后缀构成的新词,它们都带 有表示否定意义的前缀或后缀。A项mindless由mind加后缀less构成,意为“不注意;忘 却的”;B项unaware由aware加前缀un构成,意为“不知道的;未觉察的”;C项brainless由brain加后缀less构成,意为“无头脑的;无意识的。”根据句意,D项是正确答案。 2) There has been a terrible accident all because of his ______. A) care B) careful C) careless D) carelessness 分析:在这道题中,四个选项是以care为词根的。除A外,B、C、D三项都加有后缀。 B项careful和C项careless中的后缀-ful和-less都是形容词后缀,它们的词性都是形容词, 而根据语法要求,because of后需接一个名词作其宾语,B、,两项可以排除;,项和,项 都是名词,,项carelessness由care加形容词后缀-less,再加名词后缀-ness,意为“粗心; 没有小心”,根据题意,,项是正确答案。 B(充分利用语境 在阅读技巧里,有一种方法是根据上下文来判断某个词或句子的意义,学生能在一个比 较宽广的空间里来进行判断,而词汇题一般都只有一两句,所给的语境(上下文)不如阅读 理解所提供的多,而相当部分试题仍然提供一定的语境,在解题时必须充分利用。 1) As a mother, she s too ______ towards her daughter, she should let her see more of the word. A) hopeful B) protective C) modest D) confident 分析:从四个选项来看,似乎每一个空格填入空格处都符合语法,哪一项符合题义呢, 题句的后半句she should let her see more of the world的意思是:她(母亲)应该让她(女儿) 多见点世面。我们可知母亲对她女儿太溺爱或保护了。B项protective意为“爱护的”;AQ 项hopeful意为“希望的”;C项modest意为“谦虚的”;D项confident意为“自信的”,故 D项是正确答案。 2) She is a very ______ secretary; she never forgets anything or makes a mistake. A) anxious B) effective C) adequate D) efficient 分析:从四个选项来看,它们都是形容词,都可作定语。题后半部分解释前半部分,意思是:她从不忘记任何事或任何一点错误。我们可知:她是一个有能力的、胜任的秘书。确D项 efficient意为“(指人)有能力的;能胜任的”;A项anxious意为“忧虑的;挂念的”;B项 effective意为“(措施等)有效的,奏效的”;C项adequate意为“足够的”故D项是正确答 案。 C(根据语法结构解题 有些词汇题的选取项初看起来似乎都可填入空格处,令学生感到迷惘。但若借助语法结构知 识来分析句子,说不定问题就迎刃而解了。 1) It was ______ that wages be raised. A) agreed B) announced C) proposed D) planned 分析:把四个选项分别填入空格中,句子的意思都是符合逻辑的。但通过分析句子的语法结 构,不难发现that引导的是一个真实主语从句,谓语动词有时态的变化,而此题明显是一个 用请下载www.docin.com/yongqing 虚拟语气的句子,wages后省去了should。在有些表示愿望、建议、命令等的动词、形容词 和名词后面的从句常用虚拟语气,本题的四个选项中,只有C项属于此类情况,故C项是 正确答案。 D(运用逻辑推理 英语中有些词本身就表现出一定的逻辑关系,如因果关系、递进关系、对比关系和让步关系 等。较为常见的这类词有连词、副词、介词短语等。分析句子时,运用一点逻辑揄的方法, 也会有助于理解。 1) When I was very young, I was terribly frightened of school, but I soon ______ it . A) got off B) got across C) got away D) got over 分析:因为but是表示转折关系的词,but的前后两个分句所表示的动作与状态必然会发生 变化。在本题中,but的前面的分句句意是:当我很小时,我很怕去学校。那么but后面的 分句所表示的动作将会改变前一分句所在的状态,D项get over意为“克服,战胜”,正符 合此要求。A项get off意为“(人„)下来;脱下”;B项get across意为“(使)通过;解释 清楚”;C项get away意为“逃脱;脱身,离开”,A、B、C三项不合题意,故D项为正确 答案。 2) The house is very ______, and furthermore, it‘s too far from the town.. A) neat B) spacious C) expensive D) fashionable 分析:因为furthermore是表示递进关系的词,furthermore的前后两个分句所说明的问题应 是同一类,并且两者之间的关系是递进的。在本题中,furthermore后面分句的句意是:它(房 子)离城太远。所说的房子的缺点,而A项neat意为“整洁的”;B项spacious意为“宽敞 的”;D项fashionable意为“流行的”,A、B、D三项都是描述房子的优点,与题意不符; C项expensive意为“昂贵的”,正合furthermore所表现出来的逻辑关系,故C项为正确答 案。 3) I don‘t think that flat is cheap, ______ I think it is rather expensive. A) in conclusion C) in addition B) on the contrary D) on the other hand 分析:在本题中,cheap与expensive是一对反义词,两个分句表现出明显的对比关系。 A项in conclusion意为“总之”;C项in addition意为“而且”,它们不能表示对比关第,可 首先排除;D项on the other hand虽表示的也是对比关系,但它指的是一个问题的两个方面, 不合句意:B项on the contrary意为“相反地”,正好表达cheap和expensive的对比关系, 故B项为正确答案。 4) ______ his great wealth, he always remained a man of simple tastes. A) Except for B) With regard to C) Despite D) Although 分析:根据句意,本题要表达“他很有钱,但他仍过普通人的生活”的意思,两者之间含有 让步关系。A项Except for 意为“除„之外”;B项With regard to意为“关于”;但它后面 要接句子,不能接短语;C项Despite意为“尽管”,在句中表示让步关系,其后可接短语, 故C项是正确答案。 E(利用固定搭配 英语中的词组多系固定的搭配,它们通常要完整地使用,如能识别它们,对解题大有帮助。 1) The students were participating ______ an international energy-saving competition between towns in New England and Canada. A) for B) in C) to D) at 分析:participate in是一个多动词词组,意为“参加”;participate后不接其它三项中的词。 2) I hope my teacher will take my recent illness into ______ when judging my examination. A) regard B) counting C) account D) observation 分析:take„into account是一个词组,意为“考虑到;体谅”,into后不能接其它三项 中的词。 3) It was difficult to guess what her ______ to the news would be. A) impression B) reaction C) comment D) opinion 分析:从句子意思的角度看,四个选项都有可能是答案。但若把介词to 与各选项的名词一 用请下载www.docin.com/yongqing 起考虑,情况就不同了。A项impression应与on, upon, of搭配,表示“印象”;B项reaction应与to搭配,表示“评论”:D项opinion应与of, about搭配,表示“意见,见解”,根据以上分析,B项是正确答案。 四、 结论 I. 如何记忆词汇 1. 读音记忆法 brake bike like question addition resolution 2. 构词法 词缀法: unlimited preview anti-fat (减肥)antipollution supercomputer, super-model ir- (多加在以字母r开头的单词前),如:irregular, irresponsible, irrelevant, irresistible il- (多加在以字母l开头的单词前),如:illegal, illiterate, illogical im- (多加在以字母p, b, m开头的单词前),如:immoral, immortal(不朽的,不死的), impossible, impatient, impractical, improper, impolite sub- subtitle, submarine(潜水艇) 复合词: makeup, manuscript, byproduct, shorthand(速写), safeguard(保护者,保障),handout (分发的印刷品,新闻稿), offspring(后代), intake(引入口,入口), go-between(做 中间人,居中调停), editor-in-chief 类别词: animal: swan, shrimp(小虾), shark, locust(蚱蜢) clothes: blouse, garment, underwear, jean flower: tulip(郁金香), lily(百合花), violet(紫罗兰), rose, lotus(荷花) occupation: blacksmith(铁匠), ambassador, historian, deputy(代理人,代表), magician, electrician(电机师) subject: aviation, statistics, ecology(生态学), psychology 3. 引伸法 escalator 自动扶梯 逐步升级 heart a heart of gold 道德高尚的人 to break one‘s heart 使某人伤心 heart and hand 热心地 heart and soul 全心全意地 heart to heart 贴心地,坦率地 learn sth. by heart 记住,背下 to lose heart 丧失勇气 get 使 get across 被了解,讲清楚 get along 前进,进展,过活 get away 走开,离开,逃脱 get down 从„„下来 get down to 开始,着手 get into 对„发生兴趣,卷入(使进入) 用请下载www.docin.com/yongqing get over 克服 get through 干完 balance 天平 平衡 mouthpiece——口状物 喉舌,代言人 coat——外套 涂层 beast——兽 凶残的人 pig——猪 贪吃懒做的人 jewel——宝石 受珍视的人 4. 借助上下文(这一点在阅读理解中尤为重要) , 根据生词前后出现的其他词语的意思或整个句子的意思猜测词义。如: He is successful as a businessman because of his dynamic(有力量的,动力(学)) personality. He seems to have unlimited energy. , 利用信号词is, is called, means, that is, i.e., or, defined as, refer to等对某一词汇的定义、解 释或说明猜测词义。如: The word biotechnology(生物工艺学) refers to a combination of biology and technology. , 利用信号词in contrast, on the other hand, rather than, however, yet, although, while, unlike, but, whereas, as opposed to等判断反义词的意思。如: My sister Marie is an optimist, while her boyfriend is one who is always gloomy(沮丧的, 消沉的) and expects the worst to happen. , 根据举例猜测词义。如: Condiments(佐料,调味品) e.g. pepper, salt, and mustard(芥末,芥菜), make food taste better. : 根据标点符号猜测词义。如: The principal(资本,本金) , money he put in his savings account to earn interest , was safe even though the bank was closed by the police. ,根据定语从句猜测词义。如: The type of meter is called multimeter(万用表), which is used to measure electricity. , 根据同位语猜测词义。如: The invention of snorkel(潜水艇之通气管,潜水艇之呼吸装置), a long air tube that reaches up to the surface, has made it possible for submarine to use their diesel(用柴油引擎 推动的) engine even when they are submerged(在水中的,淹没的). , 根据上下文的逻辑关系猜测词义。如: You should cut out this paragraph in order to make the essay succinct(简洁的,简明的). , 根据常识猜测词义。如: Fish breathe with gill(鱼鳃). 用请下载www.docin.com/yongqing Key to Classified Exercises for Vocabulary Pre-Test 1: AACAA Pre-Test 2: CCBDA Pre-Test 3: BDCBC Pre-Test 4: DCADD Pre-Test 5: DACBB Pre-Test 6: BABCD Pre-Test 7: DBBDA Pre-Test 8: CDAAA Pre-Test 9: DADB Pre-Test10: CDCC Pre-Test 11: DCBC Pre-Test12: CBDA Pre-Test 13: BBBAC Pre-Test14: CACBA Pre-Test 15: ADCBBA Pre-Test 16: DBBDC Pre-Test 17: DAADB Pre-Test 18: ABDBA Pre-Test 19: CDABD Pre-Test 20: DCABD Pre-Test 21: BDDBD Pre-Test 22: BDACB Pre-Test 23: ABCBBD Pre-Test 24: DDCAD Pre-Test 25: BBCDA Pre-Test 26: CBBB Pre-Test 27: BDAB Pre-Test 28: BBBBC Pre-Test 29: DDCCC Pre-Test 30: BDDDA Pre-Test 31: BABBD Model Test 1 (0106) 1-5: ABBAC 6-10: BAADB 11-15: ADBAC 16-20: ABCCD 21-25:ACBDC 26-30: BDDAC Model Test 2 (0206) 41-45: DBCBA 46-50: DBABD 51-55: DADAD 56-60: DCBBC 61-65: ABADC 66-70: DCBCC 用请下载www.docin.com/yongqing 语法 Que zijiang Hu Jiahao 一、出题规律与预测 首先我们对近几年来英语四级试题中“词汇与结构”部分语法项目与词汇的测试频率进行统计: 词 类 98(6 99(1 99(6 00(1 00(6 00(12 01(6 02(1 总题数 动词(短语) 7 7 7 5 8 7 9 6 56 形容词(短语) 2 1 1 4 4 3 3 3 21 名词(短语) 4 1 2 4 2 2 3 7 25 副词 2 1 1 1 1 1 2 9 介词(短语) 3 6 6 2 1 4 1 1 9 连词 3 1 1 1 2 3 11 非谓语动词 2 2 4 2 2 3 2 4 21 时态、语态 1 1 1 3 2 2 2 1 13 虚拟语气 2 3 2 5 3 1 2 2 20 各类从句 3 1 2 2 5 2 3 2 20 省略与倒装 1 1 1 3 一致关系 1 1 特定句型 2 2 2 3 2 1 12 其他 2 2 2 从上表可以看出:语法题的考点主要集中在非谓语动词、时态和语态、虚拟语气及各类从句上。 二、四级语法考查内容 四级考试对语法的考查与《大纲》要求是十分一致的。综观近年来的四级考试题, 我们不难发现四级考试语法部分有以下特点: 1(涉及面宽 近年语法考题的考点几乎覆盖了所有词类、三种动词的非谓语形式、名词从句、形 容词从句、副词从句、独立主格、一致、倒装、强调等基本语法知识。 2(重点突出 语法考试的重点为内容庞杂较难掌握的项目,这些项目还反复出现如:虚拟语气, 状语从句,定语从句,独立主格,情态动词。 3(具体考点多为以上项目中的特殊用法 1)虚拟语气的常见考点为: ? would rather,that从句,一般过去时: It is vital,necessary,important,urgent,imperative,desirable,advisable,natural, essential,that,动词原形; It is time,about time,high time,that,一般过去时: 用请下载www.docin.com/yongqing proposal,suggestion,that,动词原形; lest,that,should,动词原形; if only,that,would,动词原形。 2)状语从句的常见考点为:非if引导的条件状语从句,此类句子多用 at times,provided, so long as,in case,once等来替代if;由even if,so,now that,for all等引导的让步状 语从句;(just,hardly…)when引导的时间状语从句;more than,as…as,not so much as, the same as,as much as等引导的比较状语从句。 3)独立主格结构的考点多为 ―逻辑主语 + 分词‖。 4)情态动词的考点多为“情态动词 + 完成式”。 5)定语从句重点考查介词,关系代词(which)和as作为关系代词。 4. 考题呈现新的趋势 近年来语法题的新趋势为:若干考点混合出现;一些交际用语也时常出现在题干中。 三、复习思路 1(全面掌握基本语法点。 2(掌握常用习惯用法和词组。 3(注意在阅读中培养语感,因为在语篇层次上培养的语感往往可直接帮助答题。 4(研究以往考试试题,适当做些练习,记住典型题例。 I. 时 态 常见考点: 1(过去完成时 had + done 过去完成时表示在过去某一时间或动作以前已完成了的动作。这个过去的时间可以用by, before等介词短语或一个时间状语从句来表示。 By the end of last year I had collected more than a thousand stamps. He had finished his homework before he went to play. 过去完成时可以表示由过去某一时间开始,一直延续到过去另一时间的动作,常和for(有 时可省去)或since构成的短语或since引导的从句连用。 It rained yesterday after it had been dry for many months. He told me he had learned a lot of things since he entered the college. 3) 过去完成时常用于以下固定句型: ? hardly / scarcely / barely + 过去完成时(倒装形式)+ when + 过去时; Hardly had I arrived when I had a new problem to cope with. ? no sooner +过去完成时(倒装形式)+ than +过去时; No sooner had the words been spoken than he realized that he should have remained silent. ? by (the end of ) + 过去时间,主句谓语动词用过去完成时。 By the end of that year, Jack had collected more than a thousand foreign stamps. 2(将来完成时: will/shall + have done 表示在将来某一时间以前已经完成的动作。常与将来完成时连用的时间状语有by (the time/the end of) + 表示将来时间的短语和句子;before (the end of ) + 表示将来时间的短 用请下载www.docin.com/yongqing 语和句子;when, after等 + 将来动作的句子 By this time tomorrow you will have arrived in Shanghai. When you come tonight at eight o'clock, I shall have written my report. 3(完成进行时态:(has/have + been doing) (had been + doing) 表示动作在现在或过去某一时间以前开始,并延续到那个现在或过去的时间,而且可 能还要继续进行下去。 It has been raining for three hours. He told me he had been waiting for me for two hours. 4(在由 after, as, as soon as, before, once, until, when, while 引导的时间状语从句和由as long as, if, unless 等引导的条件状语从句中,若主句用将来时,则从句要用一般现在时表示 将来。 I shall tell him when he comes. 5( 由This (That, It) is (was) the first (second…) time 引导的that从句往往用完成时态。 This is the first time (that) I‟ve drunk Californian champagne. There was a knock at the door. It was the second time someone had interrupted me that evening. 6. 一般将来时的替代形式: 1) be going to do sth. 在口语中广泛使用,表示准备做或即将发生的事情; I‘m going to buy a new house when we save enough money. 2) be to do sth. 表示计划安排将要做的事; thThere is to be a rail strike on July 18. 3) be about to do sth. 表示即将发生的事情; We are about to start. 4) be due to do sth. 表示预先确定了的、必定要发生的事; His book is due to be published in October. 5) be on the point / verge of doing sth. 强调即将发生的某种事态; The country is on the verge of civil war. 6) be going/ coming/ leaving/ starting/ arriving/ returning等动词的进行时可表示计划或安 排即将要发生的事情。 They are leaving for Hong Kong next month. Examples:(见学生用书) II. 虚 拟 语 气 语气(mood) 表示讲话人对某一行为或事情的看法和态度,语气分三种: 直陈语气:表示讲话人认为他所说的话是一个事实。 祈使语气:表示讲话人对对方的请求或命令。 虚拟语气:表示讲话人所说的话不是一个事实,而只是一种愿望、假设、怀疑、建议、可能 或纯粹的空想。 常见考点: 1( 一般虚拟 用请下载www.docin.com/yongqing 一般虚拟也就是由if引起的虚拟。按照不同的时间,主、从句中动词的形式为: 时 间 从 句(If) 主 句 did/were (一、三人称可用was) would/should/could/might do 现在 had done would/should/could/might have done 过去 将来 did/were to do/should do would/should/could/might do 注:1)当条件句中含有were,had或should时,可省略if并倒装。 Were I in your position, I would go. 2)根据内容的不同需要,从句与主句可有不一致的错综时间。(混合虚拟) eg: If you had taken your medicine yesterday, you might be well now. e.g. If there were no gravity, we should not be able to walk. If you had come a few minutes earlier, you would have met him. If it rained tomorrow, I should/would stay at home. 2( 特殊虚拟 情况,: 虚拟条件有时不是通过if从句明显地表达出来,而是隐含在副词、介词短语或上下 文中。经常表示这种含蓄条件的有:otherwise 或or/ with(如果有了)或without (如果没有)but for (要不是,倘若没有),相当于‖if it were not for…‖(表示与现 在事实相反),或if it had not been for… (表示与过去事实相反) I would have failed in the examination without your help. But for your timely rescue, the boy would have drowned. But for your guidance, we should/ would fail. 情况,: 情 况 动词形式 对将来发生的事情表示祝愿或愿望时,宾语从 wish/ if only后的宾词从句谓语动词用 句 would(could, might)+ 动词原形; 表示与现在事实相反的愿望时,宾语从句的谓 语动词用一般过 去时或过去进行时; 对过去发生的 事情表示遗憾或后悔时,宾语 从句的谓语动词 用过去完成时或could/ would have done; 在 It‘s (highly/ about) time that从句中的动词常用一般过去时 ( that)从句中 would rather, would sooner, would just as 常用一般过去时表示现在或将来 soon后的句中 用请下载www.docin.com/yongqing 在suggest, demand, require, I insist等动词后的宾语从句 中 ?坚持(insist) ?命令(command, order,direct) ?建议(advise, propose, suggest, move) ?要求(demand, require, request, ask, urge) 在与suggestion, proposal, motion(动议), requirement, order, decision, request, demand, (should) + 动词原形 plan, recommendation 等词有关的表语从句和同位语从句中 在下列形容词、过去分词和名词作表语的 “It's…that…”:important, crucial, vital, necessary, essential, advisable, reasonable, proper, impossible, strange, natural; suggested, requested, proposed, ordered, insisted, desired; a pity, a shame, no wonder的句型中 lest, for fear that, in case 引导的从句 例句: Many congressmen seconded Jefferson's motion that a special committee (should) be set up to look into the problem. It is necessary that he should be on time. It is essential that a guard (should) be on duty at all hours. It is a pity that Peter should be so careless. Every attention must be paid to him, lest he feel that he is inferior to my other guests. Examples及Exercises: (见学生用书) III. 非 谓 语 动 词 常见考点: 1(动词后接动名词还是不定式,或两者均可,这是个约定俗成的习惯用法问题。根据实际 应用的规律,可分为以下三类: 1). 只能接不定式做宾语的动词有: 决定,希望,学,打算,设法,供给,准备(完); 同意,拒绝,未,假装,碰巧,答应,计划(办)。 decide, hope, learn, plan, manage, offer, prepare agree, refuse, fail, pretend, happen, promise, plan 2). 只能接动名词做宾语的动词有: 错过没介意,喜欢提建议;不断搞实践,坚持(别)放弃; 承认谅不禁,推迟完成期;避免忆冒险,设想(多)考虑。 miss, mind, enjoy suggest, keep on practice, persist, quit admit, cannot help, delay/postpone, finish, avoid/prevent/evade, recall/recollect, risk, consider 注意:下列句型结构:It‘s no good/use in doing sth…. 用请下载www.docin.com/yongqing There is no need/point/ (in) doing sth. be busy, be worth doing sth. have difficulty/trouble in doing sth…. spend/waste time in doing sth… 3). 既可接动名词又可接动词不定式做宾词的有: ? 接动词不定式和动名词作宾语而意义差别不大的动词: love, like, dislike, hate, begin, start, continue, intend, prefer, propose, etc. ? 可跟动词不定式和动名词作宾语但意义有差别的动词: ? forget to do 忘记要去做某事(此事未做) forget doing 忘记做过某事(此事已做过或已发生) ? stop to do 停止、中断(某件事),的是为了做另一件事 stop doing 停止正在做或经常做的事 ? remember to do 记住去做某事 remember doing 记住做过某事 ? regret to do 对要做的事遗憾 regret doing 对做过的事表示遗憾 ? try to do 努力、企图做某事 try doing 试验、试一试某种办法 I remember posting the letter for you. (我记得为你寄过信。) I remember to post the letter for you. (我没忘记为你寄信。) ? need, want, deserve + 动名词/ +不定式被动态, 表被动意义。 The pencil needs sharpening. (to be sharpened) 4) 不定式和动名词的一些特殊的用法: ? 在had better, would rather… than, cannot but, can but 等词语后和句型“Why…?‖或 ―Why not…‖中,动词不定式须省掉to;介词except和but前若有动词do的某种形 式,他们后面的不定式一般不带to,反之则需带to. I would rather die than surrender. ? 在某些及物动词的复合宾语中,动词不定式须省掉to, 这些动词有表示生理感觉: see, watch, hear, feel 及使役动词:make, let, have。 I heard him speak in the next room. Make the past serve the present and foreign things serve China. 但上述感觉动词和使役动词转换成被动结构时,其后的不定式一般须带to,如: He was seen to come. The boy was made to go to bed early. ? 下列词组中的to是介词,其后要接动名词: agree to, object to, close to, come to, lead to, refer to, equal to, stick to, belong to, devote to, look forward to, thanks to, be used to, be accustomed to ? 英语中如果不定式与其所修饰的名词或代词之间是动宾关系,我们称之为反射不定 式。反射不定式如果是“不定式(及物动词、不及物动词、形容词)+ 介词”结构, 这里的介词一般不能省。例如: He hasn‘t decided which hotel to stay at. In prehistoric times a group of about 60 people had many kilometers of empty land to wander and search for food in. There are some things to be grateful for. 用请下载www.docin.com/yongqing I have got a loaf of bread: Now I‘m looking for a knife ______. A) to cut it with B) to cut with it C) with it to cut D) it to cut with 2(现在分词和过去分词 分词也是一种非限定动词,它兼有动词和形容词的特征。 及物动词write 不及物动词go 主动语态 被动语态 现 一般式 writing going being written 在 分 完成式 having written having been written 词 having gone 过去 written 分词 gone 1). 现在分词和过去分词的区别: 语态:现在分词表示主动语态的意思,而过去分词多由及物动词变来,表示被动语态的 意思。e.g. a moving film 一部感动人的电影 a moved audience受感动的观众 时间关系:一般来说,现在分词所表示的动作往往正在进行,而过去分词所表示的动作, 往往已经完成。 e.g. boiling water 正在开的水 boiled water 已经煮开过的水(可能是凉开水) Seeing these photos, I could not help thinking of my childhood days in Los Angeles. Not knowing what to do, she went to the godfather for help. Frustrated, he went back to his hometown. 注意:修饰过去分词常用much,修饰现在分词和形容词化了的过去分词(如interested, excited)则用very。 2). 分词作状语 ? 分词和分词短语作状语时,可以表示时间、原因、条件、让步、和伴随情况。在表 示时间、原因、条件、让步、方式时,通常可以换为相应的状语从句,表示方式和 伴随情况时可以转换成一个并列的谓语成分。 Seeing the teacher entering the room, the students stood up. (When the students saw the teacher entering the room, they stood up.) Heated, ice will be changed into water. (When it is heated, ice will be changed into water.) ? 分词作状语与主语的关系 ? 现在分词作状语时,现在分词的动作就是句子主语的动作,它们之间的关系是主 动关系。 Not knowing what to do, he went to his parents for help. ? 过去分词作状语时,过去分词表示的动作是句子主语的承受的动作,它们之间 的关系是被动关系。 Given more attention, the trees could have grown better. ? 分词作状语时前面可用连词if, while, when, once, unless等引导的状语从句,若其 用请下载www.docin.com/yongqing 主语与主句的主语相同时,可保留该连词,其余部分则简化为分词短语,相当 于省略的状语从句。 If falling ill, I‘ll see my doctor. Even though given every opportunity, they would not try. Examples及Exercises: (见学生用书) IV. 情 态 动 词 常见考点: 1. 情态动词的完成式 1) must have done & can‘t / couldn‘t have done must have done 表示对过去事情较有把握的、肯定性的推测,常译为“一定,准是,肯定”,其否定形式是can‘t / couldn‘t have done,意为“决不可能,不太可能”。 e.g. Since the ditch is full of water, it must have rained last night. 2) should / ought to have done & shouldn‘t / ought not to have done should / ought to have done 表示过去应该做的却没有做,常含有后悔、责备、埋怨的口吻,意为“本该当„„”,“应当„„就好了”; shouldn‘t / ought not to have done表示过去不该做的事却给做了,也常含有责备、不满之意,意为“本不该„„”。 e.g. With all the work finished, I should have gone to your birthday party last night. You shouldn‘t have gone back to work yesterday without the doctor‘s permission. 3) may / might have done may / might have done 表示对过去事情的推测,但不十分有把握,意为“可能,或许已经„„”。 e.g. A traffic accident happened yesterday and a driver may have been hurt. 4) could have done could have done 表示过去有可能或有能力做某事,而实际未做、未能 实现的事;有时也表示轻度批评,意为“本来可以”。 e.g. The gas leakage could have been avoided if the gas company took immediate measures after the warning call. 5) needn‘t have done needn‘t have done 表示过去做了某事,但没有做的必要,意为“本没有必要„„”。 e.g. I needn‘t have bought all that wine---only three people came. 2. 可作情态动词用的短语 would rather, would sooner, would (just)as soon, had rather, had better, had sooner, cannot but, may/might as well Examples: (见学生用书) ?. 名 词 从 句 名词从句是在句子中起名词作用的主谓结构,根据句法功能,可分为主语从句,表语从句,宾语从句和同位语从句。 用请下载www.docin.com/yongqing 主语从句 宾语从句 It's generally believed + People generally believe + that smoking contributes to lung cancer People's general belief + is scientifically proved People's general belief is + 同位语从句 表语从句 常见考点: 1. 同位语从句: 在某些抽象名词后常用连接词that引出同位语从句,具体说明这些名词的内容。同位语的先行词通常为: belief (相信), conclusion (结论), evidence (证据), fact, hope, idea, 普通型 message, news, opinion, point (论点), problem, proof (证据), theory, thought, rumor (谣言) 疑问型 doubt (怀疑), question 虚拟型 demand, motion (提议), order, proposal (建议), suggestion, etc ※ 注意:区分同位语从句和that引导的定语从句。在同位语从句中,that 只起引入从句的 连接作用,在从句中不担当任何成分,也没有具体词义。而在定语从句中,that 是关系代词,在从句中充当一定成分,其词意来自于所修饰的先行词。 ※ 同位语从句与先行词的分隔现象 原 因 例 句 1)主句谓语较短 Suddenly an idea occurred to me that she might lose her husband. 2)先行词后嵌入一状语 There is a rumor in the town that her husband will desert her. He refused to consider the suggestion put forward by his 3) 先行词后有一作定colleagues that the party not be called off. 语的短语或从句 Everyone present was amazed at the evidence he revealed in court that the judge was the very murder. 2. 介词后的宾语从句往往用which引导,除以下:but that(而不是), except that(只是,除了), in that ( 因为,在于), save that。 Examples: (见学生用书) ?. 定 语 从 句 定语从是在复合句中起定语作用的主谓结构,它所修饰的名词或代词是它的先行词。定语从句由关系词引入,关系词的作用是引导定语从句并在从句中充当某一句子成分,其词汇意义等于先行词或介词加先行词。如:This is the book which tells about the moon.句中,which 用请下载www.docin.com/yongqing 引导定语从句,在从句中作主语,其词汇意义就是the book。 先行词 关系词在句中的功能 关系词 主语 who, that 宾语 whom, that 人 定语 whose 表语 that 主语,宾语 which, that 事物,动物 定语 whose…of which 表语 that 时间名词 主语,宾语 which, that 状语 when, 介+ which 地点名词 主语,宾语 which, that 状语 where, 介词+ which 人,物 主语,宾语,表语 that reason 状语 why, for which way 状语 that, in which 前有as, such, the same 主语,宾语,表语 as 一个句子 主语,宾语,表语 which, as 是all, everything, nothing, that(不用which, who) anything, something, little, much, that等 被序数词,最高级形容词,all, every, that(不用which, no, any, little, much, only, last等修饰 who) 是who, which开头的问句 that(不用which, who) 无先行词 what 注意事项: ? 必须准确认定关系词在定语从句中充当的成分,否则就会选错关系词。 ? 关系代词在从句中作主语时,从句谓语通常与先行词保持人称和数的一致。 ? 关系代词在定语从句中作宾语,从句谓语是不及物动词时,不要遗漏介词。如: I have lost the pen with which I often write. ? as 引导的非限制性定语从句含有“正如…”之意,并可置于主句前。Which无“正如…” 之意,也不能置于主句前。 ? 表部分关系的定语从句 “数量词 + of which (whom)…” The students, most of whom were girls, enjoyed the film. Examples: (见学生用书) ?. 状 语 从 句 根据其用途,我们可将状语从句分为时间状语从句、地点、原因、结果、目的、条件、 让步及方式状语从句九种。 1. 时间状语从句 when, whenever, as, while, before, after, until, till, since, once, as soon as, the moment, the minute/instant, instantly… 用请下载www.docin.com/yongqing 2. 地点状语从句 where, wherever, anywhere, everywhere 3. 原因状语从句 because, as, since, for, now that (既然;由于), considering that, on the ground that, seeing that, in that 4. 结果状语从句 that, so…that, such … that, with the result that 5. 目的状语从句 so, so that, in order that, lest, in case, for fear that, seeing that, 目的状语从句常用情态动词may(might), shall(should), will(would). 6. 条件状语从句 if, unless, supposing, as long as, on condition that, suppose, provided(that), but that 7. 让步状语从句 though, although, even though/if, no matter…, 8. 方式状语从句 as, as if/though, how, the way, Just as…so“正如…,…也…” 9. 比较状语从句 as…as, as, not so…as, than, the… the… Examples: (见学生用书) ?. 倒 装 1(语法结构上的倒装 1)省略if的虚拟语气条件状语从句 ―Were there no air or water, 2)表语,as/ though +主语,link.V. 让步状语从句“Busy as he is,‖ V,as +主,谓语(部分) “Try as I might,‖ 原形 3)So, Nor, Neither 引导的句子。 2(修辞上的要求 1)以否定词或带有否定意义的词位于句首。 never, not, nowhere, hardly, rarely, scarcely, barely, little, seldom under no circumstances, in no case, at no time, by no means, in no way in vain, not only…but also, hardly/scarcely…when, no sooner…than, not until(引导的从句主谓不倒倒装,主句的主谓结构必须倒装。 --Not until the child fell asleep did the mother leave the room. ※ -not否定主语时不倒装。 ※ 否定意义有抵消时:not infrequently(经常) 2)only + 副词/ 介词短语/ 状语从句,主谓部分倒装; ※ only + 主语时不倒装 3)以there, here, away, up, down, off, out, in, now, then开头的句子, 动词是go, come, be等时,主谓部分要倒装,但主语若为人称代词时,则不倒装。 4)So + adj./adv. + that.句型结构 --So excited was he that he couldn‘t say a word. 用请下载www.docin.com/yongqing Examples及Exercises: (见学生用书) ?. 比 较 结 构 倍数/分数用法 80 percent as much as that of 1998. The nation‘s grain output in twice what it was in 1998. 1999 was 4 fifths more/less than that of 1998. The nation‘s grain output was reduced/ cut, went up/rose, by 80% / 4 fifths . increased/decreased to 80% / 4 fifths. by three times. 常考的比较句型 1). would sooner…than, would as soon…as (宁可…而不愿) I‘d sooner/rather resign than take part in such dishonest deals. 2). more of A than B (与其说像B,倒不如说更像A) The newcomer is more of a salesman than a scholar. 新来的人与其说是位学者,不如说是 位推销员. 3). as much A as B (不仅B,也同样A) The fault may be as much with the whole society as with the young people. 4). no so much A as B (与其说A, 不如说B) The selfish man did that not so much for his wife as for himself. 这位自私的人那样做与其说是为他妻子,倒不如说是为了自己. 5). more than + adj./adv./v./n. (不只/止是, 非但…) He is more than anxious about it. 对此他不止是着急而已. He is more than complained, he threw the whole book of rules at me. 6). more than to do sth. (很…而不致于…) Kate knew better than to go with such a rude man. Your are wiser than to believe his story. 特殊比较级 英语中一部分来自拉丁文以ior结尾的词,本身已是比较级, 与比较介词to 联用,并不能 再在这些词前用more. inferior, superior, junior, senior, prior, posterior She would make a doctor far superior to the average. He is 7 years junior to me. X. 一致关系 一致关系(Concord or Agreement)是指两个或几个语法成分须在人称、数、性等方面保持一致。现归纳如下: 1( 由or, nor, either…or, neither… nor, not only…but (also)连接的并列主语,谓语动词的单、 用请下载www.docin.com/yongqing 复数要与它邻近的主语的数一致。如: Either you, or I am going to be in charge of this matter. Not only the nurses but also the doctor is patient enough to attend to the invalids. 2(当主语后面跟有with, along with, including, as well as, in addition to, except, no less than, as much as, besides, like, but, rather than, together with等引起的短语时,其谓语动词不受这 些词组的影响,而是依主语的单、复数而定。如: Justice, as well as the law, demands that these bandits be severely punished. The father, rather than the brothers, is responsible for the loss. 3(ics结尾的学科名称,如physics, statistics, linguistics, mathematics, economics, phonetics, semantics等作主语时,谓语动词通常用单数,但如果这些词表示“学科”以外的其他意 义时,谓语动词用复数。例如: Statistics is not as difficult as some people think. Statistics show that there is a great increase in production. 4 英语中单复数同形的名词,如means, series, species, crops, aircraft, sheep, works等作主语 时,谓语动词用单数还是复数要根据它们所隐含的意义来确定。如: Every means has been tried, but in vain. All available means have been adopted to save the badly-hurt workers. 5. 表示时间、距离、价格、度量衡等名词的复数作主语时,通常作整体看待,动词用单数 形式。如: Ten dollars seems a fair charge for the repair. Three weeks was allowed for making the necessary preparations. 有些形式为单数,意义为复数的名词,如people, police, cattle, militia, clergy, poultry 等作主语时,后面的谓语动词总是用复数。如: The police are making enquiries about the murder. The cattle are grazing in the fields. 如果主语由“the + 形容词(或分词)”结构担任时,谓语动词一般用复数。如: 13) The very wise avoid such temptations. each或由some, any, no, every构成的复合代词作主语时,谓语动词用单数。如: 14) Each of the girls whom I invite to take part in the discussion has indicated that she will be happy to come. 15) Anyone who fails the examination can lose their licence. either和neither作主语时,谓语动词通常用单数。如: 16) Although both clocks strike cheerfully, neither keeps good time. Examples及Exercises: (见学生用书) XI 平 行 结 构 英语中的平行结构是指表达平行概念时要用相同的语法结构,即词与词、短语与短语、 句子与句子,前后都要对等。常见的平行结构形式主要有两种。 1(连词连接的平行结构 并列连词and, but, as well as, or else, both and, either or, neither nor, not only but also, whether or, rather than 等以及从属连词than均可连接平行结构。 用请下载www.docin.com/yongqing 1)连接的词或短语的形式一致 e.g. She is a beautiful and elegant woman. 她是一位美丽而优雅的女子。 2) 连接的非谓语动词形式一致 e.g. Some find swimming more enjoyable than sitting at home reading. 有人觉得游泳远比 坐在家里读书有趣。 3) 连接的谓语形式一致 e.g. This is the lady who listens to the poor and ministers to their wants. 这就是那位能倾听 穷人意见并能照顾他们需要的女士。 4) 连接的句子结构一致 e.g. He asked me whether I had received the check and whether I had cashed it. 他问我是否 受到了支票并把它兑换成现金了。 2(特定的平行结构 除了以上四种平行结构现象外,在某些词后面以及某些结构中都要用固定的平行结 构。这类常用的词和结构有: 1) prefer后面接的平行结构 prefer to do sth. prefer sth. to sth. prefer doing (sth.) to doing (sth.) prefer to do (sth.) rather than do (sth.) 2) 在would /had rather/sooner 以及would /had as soon as 结构中要用平行结构 would/ had rather /sooner do (sth.) than do would/ had as soon do (sth.)as do (sth.) e.g. I would as soon stay at home as go shopping. 我宁愿呆在家里也不愿去逛街购物。 Answers 时态 Examples: 1-5 A C C C D 6-10 D A B A A 11-15 C D A B A 16 B D B 虚拟语气 Examples: 1-5 B C D A A 6-10 A D B B D 11-15 A D C A A 16-20 D C A A C 21-25 D C C D B 26-30 B D A C D Exercises: 1-5 C C A C D 6-10 A D C A D 11-15 A B C C D 16-20 C C B A D 21-25 D B D C D 非谓语动词 Examples: 1-5 A D B B B 6-10 D D B A B 11-15 A B A B C 16-20 C A B B A 21-25 A A A B D 26-30 B B D D C 31-35 D A B D C 用请下载www.docin.com/yongqing Exercises: 1-5 D A A D D 6-10 A C B C A 11-15 B C B A C 16-20 D A C C C 21-25 D B B B C 26-30 B B A D D 31-35 A D C D C 36-40 C B B B B 41-45 C C A D B 46-50 C D B C B 51-55 B B A D A 56-60 B C C B C 情态动词 Examples: 1-5 A A D C D 6-8 D D A 名词从句 Examples: 1-5 D A B C C 6-10 C C A D C 11 B 定语从句 Examples: 1-5 C C B A D 6-10 C A A B B 11 C 状语从句 Examples: 1-5 D B D B C 6-10 C A C A A 11-15 C C A B D 16-19 C B C A 倒装 Examples: 1-5 B C A C C 6-9 A A A D Exercises: 1-5 D B A C D 6-10 D B D C B 11-15 C C A B C 16-20 A C D D B 21-25 C B D D B 26-30 B A B C D 比较结构 Examples: 1-5 A A C D C 6 B 主谓一致 Examples: 1-5 B B A C B 6-10 A A D C B 11-15 B A D B A 16-20 B C C A B Exercises: 1-5 A B D A A 6-10 D A C A D 11-15 B D D A D 16-20 C A B B C 21-25 A A A C D 26-30 A D D A B 平行结构 Examples: 1-5 C C A C A 6-10 A C B B C 11-15 C A B A D 用请下载www.docin.com/yongqing 完形填空 Cloze Cheng Jin Zhang Lanling Cloze主要测试考生综合运用英语语言的能力,特别是对篇章的理解能力,这也是它与单句形式的试题的根本区别。考生不但要掌握相当的词汇量和牢固的语法知识,而且要具备一定的语篇分析能力、语感、英语背景知识和社会常识。 CET-4的cloze一般安排在试卷一的Part IV,通常为一篇200词左右的短文,包括20个空,答题时间为15分钟,总分为10分。 出题类型 Smoking is considered dangerous to the health. Our tobacco-seller, Mr. Johnson, therefore, always asks his customers, if they are very young, whom the cigarettes are bought 1 . One day, a little girl whom he had never seen before walked 2 into his shop and demanded twenty cigarettes. She had the 3 amount of money in her hand and seemed very 4 of herself. Mr. Johnson was so 5 by her confident manner that he 6 to ask his usual question. 7 , he asked her what kind of cigarettes she wanted. The girl replied 8 and handed him the money. While he was giving her the 9 , Mr. Johnson said laughingly that 10 she was so young she should 11 the packet in her pocket in 12 a policeman saw it. 13 , the little girl did not seem to find this very funny. Without 14 smiling she took the 15 and walked towards the door. Suddenly she stopped, turned 16 , and looked steadily at Mr. Johnson. There was a moment of silence and the tobacco-seller 17 what she was going to say. 18 at once, in a clear, 19 voice, the girl declared, ―My dad is a policeman,‖ and with 20 she walked quickly out of the shop. 1 A)with B)to C)for D)by 2 A)nervously B)hesitatingly C)heavily D)boldly 3 A)exact B)some C)large D)enough 4 A)ashamed B)sure C)fond D)glad 5 A)worried B)annoyed C)surprised D)pleased 6 A)forgot B)came C)feared D)remembered 7 A)Therefore B)Instead C)Anyway D)Somehow 8 A)readily B)patiently C)softly D)slowly 9 A)change B)warning C)bill D)cigarettes 10 A)as B)while C)for D)though 11 A)cover B)hide C)dip D)take 12 A)time B)case C)fear D)consequence 13 A)Nevertheless B)Moreover C)Therefore D)Then 14 A)ever B)some C)little D)even 15 A)packet B)advice C)money D)blame 16 A)away B)round C)over D)aside 17 A)wondered B)considered C)doubted D)expected 18 A)And B)So C)But D)All 19 A)weak B)firm C)joking D)humble 用请下载www.docin.com/yongqing 20 A)which B)him C)that D)what (keys: CDABC ABADA BBADA BADBC) 题目分析 词类 测试重点 实词 虚词 逻辑 词义 语法 动 名 形 副 介 连 代 1 ? ? 2 ? ? 3 ? ? 4 ? ? 5 ? ? 6 ? ? 7 ? ? 8 ? ? 9 ? ? 10 ? ? 11 ? ? 12 ? ? 13 ? ? 14 ? ? 15 ? ? 16 ? ? 17 ? ? 18 ? ? 19 ? ? 20 ? ? 题型归纳: 1. 逻辑关系类题 这类题涉及语篇的逻辑关系或过渡,是对全文整体语篇意思的考察,不仅需要词汇知识、 语法知识,更需要对文章上下文逻辑关系的发展有一个清晰的认识。只有在理解了该句 上下文或全文的中心意思后,才能作答。过渡词常由副词、介词短语和连词担任,分别 有表示转折的、表示递进的、表示因果的等。 2. 词汇题 这类问题测试词汇知识,如词义/词形辨析、搭配关系(动词与介词的搭配、动词词组、 介词词组等)、习惯用法等。正确答案的选择往往要结合上下文才能确定。 3. 语法题 这类问题考察在上下文中运用语法知识的能力,语法结构测试点如虚拟语气、主谓一致 关系、从句等。 答题技巧 1(通读全文,从短文的内容和结构方面理清脉络,整体着眼,以求得对文章题材、体裁、内容有一个全面的了解,从而把握全局,把握文章发展的基本线索。论说文一般按照逻辑推 用请下载www.docin.com/yongqing 理关系展开,叙事文一般按照时间顺序展开。 2(前后照应,寻找解题信息。做题时,要善于从短文中寻找信息。一个空格的答案往往可以在前面或后面的一、二句中直接发现或得到暗示,所以应注意行和句的整体性概念,从上下文中找对应关系,而不是仅看某一句的内容。 3(充分利用文章中各种线索,即语义、结构、逻辑三条线。注意词与词之间、句与句之间的各种修饰关系及语法关系。 练习答案及分析 Passage 1 CET-4样题,这是一段小故事 (keys: CDABC ABADA BBADA BADBC) 1. C. 根据句子意思,给谁买烟,所以是for. 2. D. 根据下文的her confident manner可以判断。 3. A. 根据词语搭配,some 和enough 均不能与the搭配。根据题意“钱数正好”,选A. 4. B. sure of herself与上文的 boldly以及下文的 her confident manner相呼应。 5. C. 根据上下文的内容,小女孩的boldly, sure, confident的举止会给Mr. Johnson什么样的感觉呢,可排除表示负面意义的worried和 annoyed ,而pleased不合情理。 6. A. 根据本句的结构可知,that 从句的内容是surprised的结果,B)和C)与句意不符, D)不合逻辑。 7. B. 此处需填一个带有转折意义的连接词。 8. A. 根据上下文小女孩充满自信的表现,应是A。 9. D. 根据上文,小女孩带的钱正好买一盒烟,因此不用找零钱,老板给她的自然是香烟。 10. A. 根据句子结构,此处需填一个引导原因状语从句的连词。而for引导的是并列分句。 11. B. 根据句意,把烟藏起来。 12. B. 固定搭配in case。 13. A. 这里需要一个表转折的连词。 14. D. 根据句意,需要一个表示程度加强的词,even 甚至连微笑都没有。 15. A. 一包烟 a packet of cigarettes。 16. B. 转过身来,turn around。 17. A. wonder +疑问词表示“想知道”。 18. D. all at once 词组,意思“突然地”。 19. B. 根据全文,小女孩的预期应该是坚定的。 20. C. 根据句子结构,with 和所选词构成介词短语作状语,with that 意为“说完这些话就…”. Passage 2 (keys: ADBBC DCCBA DACDA DBBCA) 1. if 引导条件状语从句。 2(Suggestion“建议”,context“上下文”,abstract“摘要”,information“信息、知识”。 3(Ideal “理想的”符合题意。 4. One作代词时的一种常见用法。 5. For the sake of learning “出于学习之兴趣”。 6. With brief written comments与下文的“without a grade”相对应。 7. Be responsible for“对…有责任”。 8. Assign“分配任务、工作等”,distribute“分发某事物”。而且assign在上文也出现过。 9. 整篇文章讲的是学生自主学习能力,当然教授希望提供minimum guidance。 用请下载www.docin.com/yongqing 10(根据上题,学生应自己负责去找资料,而不是他人。 11(How表示方式,意“大学图书馆是如何运作的”。 12(Particularly表强调,“尤其是,特别是‖; essentially 本质上。 13(Reference sources “原始参考资料”。 14(Like不能跟宾语从句,prefer that “较喜欢、宁愿”。 15(Too“过于、太”。 16(根据意思“教授们除了教学工作之外还有其他许多任务”,用besides. 17(根据上下文,需要一个表示因果关系的连接词。therefore 18(既然很忙,所以时间有限。(limited) 19(Approach“接洽、交涉” 20(根据句型either…or…。 Passage 3 (keys: CAADB BACCD DCBAD CBABD) 1. 词义题,“这些在促使英国成为工业革命的中心的过程中都是重要的因素”,factors。 2. But, 表转折 3. 四个选项均可作something的后置定语,B、D意思不符,extra 指“额外的”,something else指上文未提到的其他因素。 4. Creative创造性的,与下文的invent相呼应。Effective有效的,motivating使产生动力的。 5. Source 来源,origin起源、渊源。 6. Created与多次提及的creative, invent呼应。 7. Come from出生于某地,(sth) stem from sth来自或起源于某事。 8. 语法结构more A than B,与其说是B,不如说是A。 9. 词义,pure纯理论的,practical实用的,genuine真正的。 10(科学家做实验,应是accurately 准确地,而非occasionally偶尔地,reluctantly不情愿地。 11(连接词,so that 表目的,now that 既然。 12(表频率,usually常常。 13(与上文的纯理论科学家相对照,发明家志在发明一些有用、实用(use)的东西。 14(Of, theories of science, 科学理论。 15(Specific特定的、明确的,single单一的,specialized有专门用途的(如specialized tools,但与result搭配不妥),sole唯一的。 16(Many一定数量的、泛指,those表示确指的那些事物/人。 17(Develop的另一词义“开发,研制”。 18(Little 与后面的no意义相近。 19(语法题,If连接一个表示与过去事实相反的虚拟条件句,“如果没有科学家早年打下的基础,那些在科学上接受过很少或没有接受过训练的人就不可能有所发明创造。” 20(Before 与过去完成时连用;ago 通常与一般过去时连用,表示从现在角度看过去的某个时间;ahead表示将来的时间,in the years ahead;past不作副词。 Passage 4 (keys: CDACB ABBCD DACBD CDACC) 1.Irritated使烦躁,综观上下文,此项合意。Insult侮辱,tease取笑,flatter奉承。 2. Smoked与下面的ashtray烟灰缸符合。 3. Followed me into the kitchen 跟随我进了厨房。 用请下载www.docin.com/yongqing 4. Make tea/coffee意为“沏茶、煮咖啡”。 5. Help oneself to sth为谁取食物,固定搭配。 6. Be successful with sb与……相处很好。 7. Hang about with 与某人呆在一起。 8.句型refer to sb as,称某人为……。 9. Choose one‘s company 选择某人为伴。(keep sb company) 10(Nobody else没有别的人。 11(In that 在于、因为。 12(Distinguished著名的,distinguishing区别的,distinct清楚的,distinctive独特的。 13(Would not have liked,与过去事实相反的虚拟条件句。 14(Take every opportunity利用一切机会。 15. nearly几乎,almost几乎,beyond远处,around在周围。 16. for my part就我而言。 17. 尽管烦她,但did not dislike 并不厌恶,下文表示此人尚有可爱之处。 18. charm为不可数名词,且such…that结构。 19. 根据上下文,此处应是表示积极意义的词汇,而polite不符。 20. like介词,象…..一样。 Passage 5 (keys: CBDDC BABAC DCCAC CABDD) 1. used to表示过去经常性的动作。 2. effect of aging老化的结果。 3. result in导致。 4. organ器官 5. did替代词(替代functioned). 6. protection (against) 抵御……的能力。 7. be inclined to 易于……的。 8. contribute to导致。 9. replace作“代替”讲,且后面句子中也出现过。 10. the number of “……的数量”,接可数名词复数,谓语用单数; a number of …许多,谓 语是复数;the amount接不可数名词。 11. other cells与上文的some cells of the body相照应。 12. the new cells与上句对应。 13. capable of表示“有能力做…….”。 14. factor因素。;Element要素 15. themselves他们本身。 16. 根据语法结构,be known to change 17. hang loose, hang在这里作系动词。 18. shrink收缩、萎缩。 19. pass on information传递信息。 20. process过程(特别是事物的自然发展过程);procedure办事的程序、手续 用请下载www.docin.com/yongqing 简短回答 Wen Lingling Weng Caihong 一、简答题概述 简短回答问题(简答题)为四、六级考试近年来新增设的主观题型,考试时间为15分钟,安排在“阅读理解”之后,每次考试为一篇文章,大约300-350词左右。文章后有五个问题或不完整的句子,要求在阅读完文章之后用简短的英语(可以是句子,也可以是单词或短句)回答所提的问题或补足不完整的句子。每题2分,共计10分。该题型旨在测试学生对英语书面材料的确切理解能力及一定的英语表达能力,答案比较开放,可有多种答题方式,学生应灵活掌握答题技巧,以自己的话进行总结。 二、简答题题型分析 简答题解题的关键是对文章的确切理解以及对答题技巧的把握,并能以最简洁的词或句子表达最准确的含义。答题时,要尽量避免语言错误,注意字母的大小写、回答的字数是否符合要求,还应注意所答句子是否需要进行适当的增加或省略。简答题题型主要分为以下几类: 1、主题思想题(main idea) 此题型主要是用来检验学生高度概括的能力,是简答题的重要内容之一。每一篇文章都有它的主题思想,一般能在文章每一段第一句找到。有时,主题句也出现在文章某一段落的末尾或中间。在回答这类题时,一般能用一个词充分表达某一个意思,决不用一个短语;能用一个短语表达的话,决不用一个句子。常见提问方式有: What does the passage mainly discuss? What is the best title for the passage? What is the main topic/subject of the passage? What is the main idea expressed in the passage? 以1997年1月S1为例(注:简答题示例均选自六级考试全真试题): S1: What is the passage mainly about? 通读原文可以看出vehicle theft一词出现频率极高,文章的中心讨论美国汽车盗窃的问题,答案可以是: Vehicle theft and security system in the U.S. Car /vehicle /motor theft in America. Vehicle theft is a common phenomenon in the U.S. 2、事实细节题(specific details) 细节题是根据短文提供的信息和事实进行提问。此类题目涉及范围广泛,考试中所占比例大,提问方式多样。细节题的答案常常隐含在某些词语中,要求考生细读若干个句子,弄清题目意图,抓住关键词,对这些事实和根据做出评估,然后再作综合概括。本题型是考试中使用最多的题型,它们常围绕以下方面的内容展开: Why was it … ? When did …? Where did …? Who was …? According to the passage, …? 例如:1998年6月S2题 用请下载www.docin.com/yongqing S2: why are newspapers considered as an important medium according to the passage? 3、推理题 此类题旨在考查学生归纳、演绎与综合分析等逻辑推理能力,其答案一般根据已知信息来推理。这类题的答案要求考生不仅弄懂文章字面的意思,还要知道文章潜在的含义和作者所给的提示。解题的关键是:靠推断而不是原文照搬,把握文章的主题思想和每段的内容;抓住作者的观点;分析文章的有关信息,用自己的话来叙述。推理题一般有以下方式: It can be inferred from the passage that ___. What does the passage imply about? According to the author, what does the sentence suggest? The author implies that ___. What is the author‘s main purpose in the passage? 例如:1997年6月S3: S3: what made it possible for the TIME reporters to come up with so many interesting stories about pets? 本题要求考生仔细阅读、进行合理的推断,对文章第二段1、2句有透彻的理解。文章说:每一位宠物的主人都有类似的故事并渴望听的人也有同感,《时代周刊》的记者们一下就给出了25个动物的故事,每个故事都说自己的宠物是世界上最聪明的动物。从这里我们可以推断,正是因为宠物的主人急于把故事说给别人听,记者才有机会收集了这么多的故事。所以本题的答案可以是: That the owners want others to share their stories. Pet owners are willing to share their stories (with others). Pet owners want to show their pets are smart. Pet owners are proud of raising clever pets. 4、词汇题 词汇题考查学生理解多义词的能力。在英语文章里,为了避免重复或费解,作者会尽可能地运用不同的表达方式来表达同一意思。做词汇题时,如果能从原文上下文中找到原词的同义词,则用此词来做答。如果找不到,则需考生用自己的话进行解释。所以阅读时要注意破折号(其后经常是解释),同位语从句(有解释、说明、补充作用),定语从句(起修饰、限定作用)。 5、作者的态度题 涉及此类题,文章的感情色彩都比较强烈。一般说来,作者的态度鲜明,有个人看法,不会不偏不倚。因此,判断感情色彩,可以从文中有些词语看出。 三、评分标准 1、给分标准 1)答出全部内容,语言正确——2分 2)答出部分内容,语言正确(如果内容正确,有语言错误同时词数超过)——1分 3)如内容正确,有语言错误同时词数超过又有无关信息——0.5分 4)没有答对问题——0分 2、扣分标准 1)语言错误扣0.5分,每题语言错误不超过0.5分。 用请下载www.docin.com/yongqing 2)涉及无关内容者扣0.5分;如果答案中有相互矛盾的内容,则内容矛盾的部分不得分。 3)整题原封不动照搬应扣分,照搬一句扣0.5分,照搬两句以上不得分。(若有标点符号、字母大小写、人称或时态变化,则不算照搬。) 4)答题一定要用最简短的文字。 四.解题技巧导引 考生在做简答题时所出现的问题,有许多类似于在阅读理解中所出现的问题,比如阅读速度慢,方法不当,对生词的恐惧,句子与句子、段落与段落之间的逻辑关系不够清楚。但简答题也还有它的特殊问题,如对问题及问题所涉及的内容和范围不明确。有的考生甚至是以文章-问题-文章-问题的顺序来做简答题。那样,很难在15分钟之内完成所有的问题。考生的问题主要在于把握不住短文的主旨,用词不当,或语法结构有问题,或给出了2个或2个以上的答案, 针对这些问题,可以参考以下的策略: 首先,将文章的问题看懂,了解问题所涉及的范围,包括主旨、细节、观点、语气等;带着问题阅读文章,阅读过程中,理清文章的脉络层次,准确理解文章的中心思想,从文章本身去找主题句和关键词;看完文章后,开始做后面的简答题,;问题涉及到某个部分时,根据阅读时对文章层次的理解,总结出这部分的大意,然后找出关键词来回答问题;当问题涉及到某个句子时,一定要找到它的准确出处, 这类问题往往以补充句子的形式出现,一定要与问题形成一个整体。 在做简答题时,一定要注意以下问题:一定要确切理解短文的内涵,考生平时就要养成积极思考和推敲的习惯。回答问题时,能用短语不用句子,能用短句不用长句,能用简单句不用复合句,能用主动语态,不用被动语态;在时态上,要遵照问题的时态;无论用短语和句子回答问题,最重要的是选词,关键词最好用文章中的;只能写一个有把握的答案,因为两个不同的答案,即使其中有一个正确答案,也不会得分。 Keys to Short-Answer Questions Passage 1 1.The school‘s influence has been replaced by the home‘s. 2. They are well established before the children enter school. 3. In subjects such as science. 4. Parents had been warned not to educate their children. 5. Teach reading at home Passage 2 1. From 1930 to 1934. / Between 1930 and 1934. 2. Driving tests and pedestrian crossing were introduced. 3. exceeds the speed limits 4. Speed limits reduce accidents. 5. The increase in traffic density. Passage 3 1. His humor. 2. story-telling / telling stories. 3. Delivering humorous lectures. 用请下载www.docin.com/yongqing 4. He studied by himself. 5. His reading, personal experience and great imagination. Passage 4 1. By way of hanging them. 2. Because they think hanging is cruel but more humane. 3. It‘s a burden on the long-suffering taxpayer. / The murderer may escape and murder again. 4. To prevent a would-be murderer. 5. Because people may feel safe in their life. Passage 5 1. whether or not one was a native speaker. 2. The same status as their native counterpart. 3. Ones who can teach and have the required English level. 4. It singled out the unqualified (Or: It liberated non-native English teachers). 5. qualified English teachers because they were non-native speakers. Passage 6 1. Because an estimated 90% of all illnesses may be preventable. 2. Live an inactive life-style. 3. A person may be pressured by friends to accept some choice. 4. attempting suicide 5. A statistical probability of increased vitality and longevity. Passage 7 1. Economics / Survive. 2. --Because the increased cost will be more than Marge‘ income. --They might end up with less money. 3. Economic factors and emotional needs. 4. that homemaking gives them the deepest satisfaction 5. stay home / take care of her daughter at home Passage 8 1. Car theft in the U.S. / Vehicle theft. 2. His carelessness. / Leaving his key in the truck. / His stupidity. 3. More than 2 million cars would be stolen. 4. Take them to pieces and sell the parts or smuggle them out of the country. 5. A device that transmit radio signals. Passage 9 1. --They were spoiled and indulged. 2. --Because there were unpredictable twists and turns/ wild rapids in the river. --Because of the wilder rapids. 3. --They lost many games. 用请下载www.docin.com/yongqing --The team suffered from years of adversity. 4. Distrust (between labor and management). 5. Team spirit. / Team work. Passage 10 1 they want to know about the world / construct the world in their heads 2 Because they are a source of news and other information. 3 selective and influenced by different factors 4 The social context of exposure. 5 hardly planned Passage 11 1. He developed television and was the first to demonstrate it. 2. News broadcast, films, travel programs, cultural and sport programs, etc. 3. people could actually see the people and places talked about 4. Well-known persons / Famous people. 5. More people went to watch sports events. Passage 12 1 From the Greeks. 2 they excelled in engineering and architecture 3 Because they needed them for their frequent wars. 4 their neglect of philosophy and pure mathematics 5 He considers them as a practical people. Passage 13 1. Computer crimes. / Computer criminals. 2. are untouched by human hands / are handled by computer 3. claiming a computer error / moving money from one account to another 4. Leave her job. / Quit her work. 5. Computer experts. 用请下载www.docin.com/yongqing 写作Writing Gong Yanping Huang Gengxin I. Structure of CET-4 composition , 开篇的写法 写好文章或段落的开头很重要。一般来讲, 文章开头既要写出文章的主题,又要 能够引起读者阅读的兴趣。不同的文章开头的方式不同, 常见的有以下八种:引语法 (Quotation)、设问法(Questions)、数据法(Figures and statistics)、背景法(Relevant background information)、定义法(Definition)、主题句法(Topic sentence)、比拟法 (Analogy)、列举法(Listing the main items)。 一、引语法(Quotation):指通过在开头段引用一些与主题相关的名人名言、成语或谚 语等来开始一篇文章。引语法可应用于各种文体。 E.g.: As the proverb says, ―No one knows the value of health until he loses it.‖ In other words, nothing is more valuable than health. It is clear that health is the foundation of one‘s future success. … Now that we know that health is the resource of our energy, what should we do to maintain and enhance our health? … In conclusion, health is more important than wealth. (Health) It is known to all that life is short. School life is still shorter. We can obtain many a thing more than once, but we cannot get back the lost time even once. An old saying goes, ―Art is long, but life is short.‖ We have only limited time to learn unlimited knowledge. (Make the Most of Our School Days) A proverb says, ―Time is money.‖ But in my opinion, time is even more precious than money. Because when money is spent, we can earn it back; but when time is gone, it will never return. Thus we must value time. (The Value of time) 二、设问法(Questions): 只用提出问题的方法来引出文章的主题,以激起读者的兴趣。 设问法可用于各种文体。 What Is Success What is success? Different people might give different answers to this question. Some people think that one is successful if he can make a great deal of money. Other people think that success means holding an important government post. Still others consider one who has got a high academic title as success. However, I do not think that money, power, or status is the symbol of success. In my opinion, success means achieving brilliant results in one‘s speciality, making contributions to the development of the country and bringing happiness to the people. To become successful men in the true sense, we should be both perseverant and industrious. We should keep in mind that perseverance is the mother of success and industry is the key to 用请下载www.docin.com/yongqing success. If we follow these principles, we can certainly achieve success in the future. 三、数据法(Figures and statistics) 数据法就是用数字或数据开头来引出文章的主题。数据法多用于描写文、说明文和议论文。 Honesty: Is It Going out of Style? According to a recent poll, 61% of American high school students have admitted to cheating on exams at least once. It cannot be argued that such a response may not mean much. Children today seem to be glued to the silver tube called television. By kindergarten age (five years old), the average child who started watching television at age three will have consumed 5,000 to 8,000 hours of television a year, and watched more than 30,000 commercials per year. 四、背景法(Relevant background Information) For so long as there have been liars, there have been attempts to find the truth. Those on trial in ancient China were made to chew rice powder while testifying. If the powder was dry when the suspect spat it out, he was judged guilty because it was assumed that nervousness over telling lies had dried that saliva in the suspect‘s mouth. 五、定义法(Definition) Self-confidence Self-confidence means trust in one‟s own ability. As a very important psychological quality, it brings your creative power to play, arouses your enthusiasm for work and helps you overcome difficulties. When asked for the secret of her success, Mme. Curie answered: ―My success lies in perseverance and self-confidence, the latter in particular.‖ Self-confidence gives you light when you are in dark and encouragement when you are dismayed. It is self-confidence, rather than genius and extraordinary ability, that inspires you to cope with any arduous task. With self-confidence, the goal you intend can be reached, and so can the hard nut be cracked. Some people find most of things impossible, saying: ―It is beyond my ability to do the task.‖ or ―I am not cut out for it.‖ In many cases, there are only lame excuses. When our heart is void of self-confidence, every hope is gone, and everything we are laying our hands on is spoilt. On Patriotism Patriotism means the love of one‟s country. It makes one ready to sacrifice one‘s life for one‘s country and to do many other deeds which can bring benefits to the people. The effect is very great when it is exercised by all the people of a country. It makes a country strong, and her history glorious. It also improves a man‘s character and secures for him a widespread fame. To love our country, to work so as to make her strong and rich, to support her government, to obey her laws, to pay fair taxes into her treasure, to treat our fellow-citizens as we wish to be treated ourselves—this is to be a real Chinese patriot. If the people of a country are not patriotic, the country must be very weak. History tells us how some countries of ancient times were, owing to a lack of patriotism, conquered by other peoples and how the peoples of the conquered countries were made slaves under the yoke of their victorious masters. 用请下载www.docin.com/yongqing 六、主题句法(Topic sentence) On Happiness Happiness is easy to find and easy to miss. It depends on the way you look at life. It is true for most people that, for most of life, nothing wonderful happens. If you do not enjoy getting up and studying, and finishing your work and sitting down to a meal with family or friends, then the chances are you are not going to be very happy. If someone bases his happiness or unhappiness on major events like a great new job, huge amount of money, a flawless lover or spouse or an American passport, that person is not going to be happy much of the time. If, on the other hand, happiness depends on a good breakfast, flowers in the garden, a drink or a nap, then we are more likely to live with quite a bit of happiness. 七、比拟法(Analogy) A traditional story describes a foolish man lifting a rock too heavy for him and having his own feet squashed. Hitler was like that foolish man, but he was different in that, before he destroyed himself, he destroyed millions of other people. Have you ever crossed a steam by stepping on the stones? It surely is a wonderful experience to walk across a steam of turbulent current without getting wet. In doing so, however, it is important to choose the stepping stones. If there are enough stones and the distance between each two stones is not longer than a man‘s stride, you can be sure of your steps. If the gaps are too wide, or if some of the stones do not have a solid foundation, you should look before you leap, or you would fall into the water. Putting your ideas across to other people is just like stepping across the stream. The words, idioms and grammar points are the stepping stones. If you have a solid foundation of everything, it would not be difficult to put your ideas across, and you would appreciate your own ideas being understood and accepted by your audience. On the other hand, the deficiencies in your language learning are the big gaps between the stones. Your have to be very careful if you don‘t want to fall into the water of ineffectiveness or of being misunderstood by your audience. 八、列出文中要讨论的主要内容法(Listing the main items) 简单列出扩展段要论述的主要内容,使读者对整篇文章有个概括的了解。例如: Learner Differences Although language learners share some common language learning principles, individual variations can never be ignored. In fact, it is these individual variations that may be able to explain the phenomenon that some language learners are very successful in their learning, while others can never manage to grasp what they should grasp. The influential factors in terms of learner differences can be manifold. Among them, I would like to discuss, in this paper, the factors associated with personality, cognitive style, age, and learner strategies. These factors, I think, have significant implications for second and foreign language learning. , 扩展段的写法 扩展段是一篇文章的主体段。文章的主题在扩展段展开,常用的展开段落的方法有下面九种: 列举法(Listing) 用请下载www.docin.com/yongqing 举例说明法(Examples) 因果关系法(Cause and effect) 定义法(Definition) 分类法(Classification) 比较对照法(Comparison and contrast) 时间顺序记叙法(Time order) 空间顺序描述法(Space order) 过程描述法(Process description) 一、列举法(Listing) 列举法是指在主题句之后列举一些足以支持主题句观点的具体细节。这种具体细节一般 是事实描述、数据、例子等。一般来讲, 所列举的具体细节按照重要性递增的顺序排列。 列举法常常和举例法结合起来使用。列举法常用的过渡词有以下几组。一般要相对呼应, 不混用。 1. First, second, third, … last 2. firstly, secondly, thirdly, … finally 3. the first …, the second …, the third …, …the last 4. to begin with/ to start with/ in the first place, then, next, in addition (to),besides, also, moreover, furthermore, what is more, finally 运用列举法展开段落的写作模式如下: Topic sentence Detail 1 Detail 2 Detail 3, … Concluding sentence Watch TV Football Watching a football game on television may seem like the easiest thing in the world. However, like the game of football itself, watching a game correctly is far more complicated than it appears. First is the matter of the company. The ideal number of people depends on the size of your living room. Also, at least one of your companions should be rooting for the opposite team. There‘s nothing like a little rivalry to increase the enjoyment of a football game. Next, you must attend to the refreshments. Make sure to have on hand plenty of everyone‘s favorite drinks, along with the essential chips, dips, and pretzels. You may even want something more substantial on hand, like sandwiches or pizza. If you do, make everyone wait until the moment of kickoff before eating. Waiting will make everything taste much better. Finally, there is one last piece of equipment you should have on hand: a football. The purpose of this object is not to send lamps hurting from tables or to smash the television screen, but to toss around outside during half-time. If your team happens to be getting trounced, you may decide not to wait until half-time. 二、 举例说明法(Examples) 举例说明法是指用具体的事例来阐述主题句中的中心思想,举例说明法常与列举法结合使 用。使用这种写作方法要注意所选事例应与文章主题密切相关,具有典型性和代表性。举 例法常用的句型句式和词语有: This can be illustrated by … An example of this is/involves … 用请下载www.docin.com/yongqing …can be illustrated/shown by the following examples/instances. There are many examples to show that … One example is … . Another example is … The … shows/illustrates … For example, for instance, say, suppose, such as, such … as, to provide a specific example, as an illustration 运用举例法展开段落常用的模式如下: Topic sentence Example 1 Example 2, … Concluding sentence The Balance of Nature It is only during the last few years that man has become generally aware that in the world of nature a most delicate balance exists between all forms of life. No living things can exist by itself: it is part of a system in which all forms of life are joined together. If we change one part of the natural order, this will in its turn almost certainly bring about changes in some other part. The cutting down of forests reduces the supply of oxygen. The killing of weeds and insects by chemicals leads to the widespread poisoning of animals and birds. The throwing of waste products into the ocean damages life in the sea, while exhaust fumes change the chemical balance of the atmosphere and shut out some of the sun‘s essential life-giving rays. And so we could go on, adding more examples, until in despair we might feel like giving up the struggle to control and keep within limits these harmful human activities. Man is very clever at changing the world around him to satisfy his immediate needs, but he is not so clever at looking far ahead, or at thinking about what the future results of his actions might be. Man may well, in his attempt to be too ambitious, destroy himself. 三、 分类法(Classification) 当要说明的事物比较复杂或包括的内容比较多时, 常常将其分成更小的类别, 这便是分 类法。使用这种写作方法要注意分类的标准一定要明确,而且标准只有一个。分类法常用 的句型句式有: 1. … may be classified into several types according to … divided the following kinds on the basis of … categorized two categories depending upon … put three main groups 2. … consists of … comprises … 3. The classification is based upon … 4. … may be divided … sub-divided … further sub-divided 分类法的写作模式如下: 主题句+所分类别+各个类别的具体事例+结束句 Ways of communication We can communicate in three basic ways. Each way has its advantages. One way is speaking. 用请下载www.docin.com/yongqing When we speak, we have a chance to judge our audience‟s reactions. If they don‟t understand, we can repeat as necessary to make our points. We can also invite additional questions and clarify additional aspects for our listeners. Another way of communicating is by writing. We can think of our subject thoroughly. We can modify our development and polish our phraseology until we get it the way we want it. The third way of communicating is nonverbal. Communication can be quite effective in a very noisy atmosphere, or when we don‟t want to dignify a response with actual words 四、 比较对照法(Comparison and contrast) 比较法是将人们熟悉的事物与其他事物放在一起分析其相似之处, 比较的目的在于表现 不同事物的相似之处;对照法则是将人们熟悉的事物与其他事物放在一起分析其不同之处, 其目的在于表现同类事物的不同方面。两者强调的重点不同,但结构相似。常用的结构有 两种:第一种称之为块式结构, 假如比较的事物为A和B,文章首先把有关事物A的全部 要讨论的内容讨论完, 再转入B事物。第二种结构称之为点式结构, 即首先就事物A和 事物B的要比较的内容的第一方面进行比较或对照,然后在比较或对照A和B的第二个方 面, 比较: Both A and B are … A and B seem to have much in common. A and B seem to have some features in common. A is similar to B in … There are a lot of similarities between A and B. A has something in common with B. A and B share similar … A shares some common features with B. A is like B in … A is the same as B in … Both, too, compare with, similarly, likewise, also, in comparison, just like, just as, also, both … and, similar to 对照: A is different from B in … A differs from B in … There are many differences between A and B. A has nothing (little) in common with B. The difference between A and B lies in … Compared with A, B … Unlike, on the contrary, in contrast, on the other hand, while, whereas, however, although Professor Wright and Professor Hunter Professor Wright, who teaches English poetry, is about forty. He has an easy-going personality. He is lively and friendly. He mixes more with the students. However, he doesn‟t work hard. Professor Hunter, on the other hand, is also about forty. He teaches management. He has a dignified personality and is always serious-looking. I never see him smile. There is always a 用请下载www.docin.com/yongqing sore look on his face when a student is late or when a student makes a mistake. But he works like a beaver. (块式结构) 块式结构模式: Topic sentence Subject A Feature 1 Feature 2, … Subject B Feature 1 Feature 2, … Concluding sentence Professor Wright and Professor Hunter Professor Wright, who teaches English poetry, is about the same age as Professor Hunter, the teacher of management. Professor Wright‟s personality is quite different from Hunter‟s. Professor Hunter is much more serious-looking than Professor Wright. I never see him smile. There is always a sore look on his face when a student is late or when a student makes a mistake. Professor Wright is more lively and friendly. He mixes more with the students than Professor Hunter. But he doesn‟t work as hard as Professor Hunter. Professor Hunter works like a beaver. (点式结构) 点式结构模式: Topic sentence Feature 1 Subject A Subject B Feature 2 Subject A Subject B Feature 3, … Subject A Subject B Concluding sentence 五、 因果关系法 因果关系法是通过分析事物的因果关系来展开段落。如果是一果多因的情况,一般先写结果后分析多种原因;如果是一因多果, 一般实现写原因后写结果。运用因果分析法展开段落最好把主题句放在开头,这种段落展开方法的结构常见的有两种:即分块法和连环法。 分块法指的是先说原因后说结果(一个原因引起一个或多个结果);或者先说结果后说原因(多个原因引起一个结果)。连环法指的是先叙述的一个原因及结果,再叙述第二个原因及结果,以此类推。 形象一点表示,运用因果关系法展开段落常见的模式有3种: 模式1 Topic sentence Effect 1 Effect 2, … 用请下载www.docin.com/yongqing Concluding sentence 模式2 Topic sentence Cause 1 Cause 2, … Concluding sentence 模式3 A Cause B Effect C (Cause) Effect D (Cause) Effect E (Cause) Effect 因果关系常用的句型句式和词语有: 原因: The explanation is that … There are several reasons why/ for … The cause of … is … The reasons for … are … … is the result of … … is caused by/ due to/ because of … because, since, now that, because of , as a result of , on account of , for this reason, owing to, due to, for, as, thanks to, result from 结果: The result/ effect/ consequence of … is … … will result in … Therefore, … As a result (of), … Lead to, result in, contribute to, so that, thus, hence, consequently, so, accordingly, in view of … Advertisements on Television Today, more and more advertisements are seen on television. Whenever we turn on TV, we will see advertisements. Every fifteen minutes we will be interrupted by commercials. Advertisements are one of the most frustrating parts of watching television. In the first place, the advertisements waste time. For instance, in order to see a 90-minute movie, we have to spend another 20 to 30 more minutes watching advertisements. In the second place, the advertisements interrupt the viewers. For example, the viewers may forget the plot of a show during the advertisements. Even worse, the frequent appearances of ads may make watchers lose their appetites for a good movie. That is the bad psychological influence of ads. In the third place, they make people under constant economic pressure. The ads make many products look more attractive than they really are. They always ask those that their young parents cannot possibly afford. But life should not be controlled by advertisements. Since ads could not disappear from TV screen, the television viewers must be aware and critical of the advertisements in order to endure them. (第一种模式) 用请下载www.docin.com/yongqing Smoking results in a series of negative effects. To begin with, it has been proved that poisonous nicotine contained in cigarettes can reduce your fitness. And what is worse, it can even cause lung cancer if you smoke constantly. That is why such warning as “smoking is harmful to your health” must be printed on the cover of the cigarette case in western countries. Further more, passive smoking occurs in your family members and other people as a result of your smoking at home or in public places. Consequently, their health will be unfavorably influenced. In addition, smoking adds to your financial difficulty if you happen to be short of money. Even if you are rich, you are encouraged to spend your money on valuable books, nutritious food, high quality TV sets, etc., not on meaningless smoking. Therefore, giving up smoking is a good way to make you and your family happy. (第二种模 式) Supermarkets, a Way to Improve Service With the development of commercial network, a number of supermarkets have been springing up in big cities in our country. It is a hopeful signal of commercial modernization. Supermarkets on an average have increased their sales much more than those offering traditional behind-the-counter services. The reasons for the increase are diverse. First, self-service requires fewer salespeople than the traditional counter service. The reduced number of employees means an increase in profits. Second, in supermarkets, customers often buy things they haven‟t planned to buy, as they have had the chance to look at things closely. Moreover, goods in supermarkets are all conveniently and attractively packed, adding to the temptation. The customers really enjoy their shopping in the supermarkets. However, supermarkets in some cities are still fairly small. More and larger ones are needed. I think that supporting the development of such markets is an important step to commercial reform, and people will feel much more convenient to buy things than ever before. (第三种模式) , 结尾 ? 结论 1. From what has been discussed above, we may reasonably draw/reach/come to/arrive at /the conclusion that… 2. Therefore, we can come to a conclusion that… 3. Thus, it can be concluded that… 4. In summary/conclusion/a word, it is important… 5. To sum up, In short, In a word, All in all…. 6. As has been mentioned above, a conclusion can be drawn that … 7. In my opinion, 8. As far as I am concerned, 9. From this point of view, 10. This conclusion is also suggested by the fact that … 用请下载www.docin.com/yongqing ? 号召性和意义 1. It is time that we urged/put an immediate end to… 2. It is necessary that some effective steps should be taken … 3. It is hoped that efforts will be made… 4. We may have a long way to go before… 5. Only by doing this can we… ? 展望,后果 1. Any person who ignores/fails to learn the warning would…. 2. I think/ believe/ that… 3. We rest assured that … 4. This is the prospect that man now faces. ―…‖ is avalanching upon our heads and most people are absurdly unprepared to cope with … 5. The future will be a better place if we do. 6. I only hope that some day there can be a solution to this problem. 7. No doubt, … will continue to be a feature / problem of … for some time to come. Its causes are rooted in the opportunities of our society, and in the rapid pace at which society changes. ? 反问 1. Why don‘t we double our efforts to learn it? 2. Is it any wonder that June is my favorite month? II. Sentence writing , Common mistakes in students‟ composition: 文章无论长短,都是由句子组成的,句子是表达思想的最基本的单位。因此,句子是否 能写得正确、达意和清楚,将直接影响整篇文章的写作质量。大学英语四、六级考试和研究 生入学英语考试的实践都表明,考生写作成绩长期得不到明显提高的主要原因是欠缺写好单 句的能力。 为改变这种状况,我们将从剖析考生作文中的典型病句入手,对写作测试中的基本句子 结构和写法进行评议和分析,来帮助考生进一步提高句子写作能力。 from the grammatical perspective: 1. sentence fragment: a sentence fragment is only part of a sentence. It is a ―wannabe‖ sentence that lacks either subject, verb, or sense. Examples: , Then pour about 3/4 water into a pot. Put some thing into the pot when it‘s boiling. , One is that it is stuffed with fascinating things and interesting jokes. Because everyone can enjoy oneself without any restrict. , All of us need recreation. Because we have relax our nerves. , Love our dormitory. Enjoy our dormitory life. 用请下载www.docin.com/yongqing [ways of correcting fragments: by joining the fragment to the sentence before or by rewriting the fragment as a separate sentence.] 2. run-on sentence: Run-on sentence is actually two sentences written as one. Fused sentence consists of two sentences joined without any punctuation between them. Comma splice: two full sentences separated by a comma instead of a period. , In the dormitory, you can‘t do anything you want, you are not one. , Lin is one of our members, he likes reading history story and English magazines, also he is one of the best friends of mine. , For example, I was very tired at night, I wanted to fall asleep quickly. [ways of correcting: put a period at the end of each sentence; use a coordinating conjunction to join two sentences.] 3. lack of subject-verb agreement: subjects and verbs must agree in number: A singular subject always takes a singular verb; a plural subject always takes a plural verb. , When one of us have troubles, the others often help him and make him feel better. , Then he will feel that dormitory life bring us so much benefit. 4. faulty parallelism: words in a pair or series should have a parallel structure. When you rewrite, you should try to put matching words and ideas into matching structures. , Most of its members are friendly, warmly and usually help each other. , We thought she was charming, intelligent, and a very capable young woman. , A man is judged not only by what he says but also by his deeds. 5. shifting point of view: writing is easier to read if it uses the same point of view throughout. You may choose a first person, second person, or third person point of view. , But we should try our best to understand my roommates or talked with them about the problem. , After one and a half year, I think dormitory life is important to us, because we live with them and see them everyday and all the time. 6. misuse of part of speech: , More courses is so terrible and fewer courses is so benefit. , More courses can open our eyes. enough our knowledge. from the perspective of sentence structure: 1. :There be:结构 考生病句: 1. There are many people like to go to the movies. 2. There are different kinds of vegetables can be bought on the market by people. 用请下载www.docin.com/yongqing 正确表达: 1. There are many people who like to go to the movies. 2. There are different kinds of vegetables that people can buy on the market. 这两个例句的错误比较有普遍性,因为在历次考试中有不少考生不能正确运用there be 这一最常用的句式。在这种结构中,there是引导词,没有实际意义。be在句中作谓语,有 时态和数的变化。 例如: 1. There was no school in the village at that time. (=there was not a school...) 注意:在否定句中,否定词用no,也可用not a或not any。not a后接单数名词,not a 后 接复数名词,no后面的名词单复数都可以。 2. There is not a moment to be lost. 3. There are many people rushing into the cities every year. 4. There are many things we can do to prevent traffic accidents. 5. There is no use holding back the wheel of history. 从以上例句还可看出,句中的主语后面可接多种修饰语,如介词短语、不定式短语、定 语从句、分词短语等等。这无疑使该结构增加了表现力,使句子表达内容更加丰富。 在运 用这一结构时,考生最容易犯的错误是在there be之后又用了一个动词作谓语,使句子结构 出现严重错误。这里列举的考生的典型错误均属这种情况,对此我们在写作中要格外注意。 2. 比较结构 考生病句: 1. Comparing with the bike, the car runs much faster. 2. The climate in Walton is colder than other cities. 正确表达: 1. Compared with the bike, the car runs much faster. 2. The climate in Walton is colder than that of other cities. 评议与分析:许多考生在作文中用compare或than表示比较,但相当多的表达有误。 在例1中,对两个事物进行比较的句式为Compared with A, B...,只能用compare的过去 分词,不能用现在分词,因为B是分词的逻辑主语,只能被比较。在例2中,考生误将"天 气"与"城市"进行比较,而二者没有可比性,只有将后者改为"其他城市的天气"才符合逻辑, 很显然,考生的错误是受了汉语表达习惯的影响。 比较结构是常用结构,正确地使用这一结构可以使文章的句式增加变化,有利于提高写 作成绩。一般说来,考生若能恰当、正确地运用这一结构,其写作成绩应在5分以上。 下面是比较结构的一些常用的表达方法。 1( 同级比较 1) In 1998 we produced as many cars as we did in the previous five years. 2) We have accomplished as much in the past three years as would have taken ten years in the past. 2( 比较级 1) Children now enjoy better medical treatment than before. 2) We can live longer without food than we can (live ) without water. 3( 最高级 1) This is the most interesting book I've ever read. 用请下载www.docin.com/yongqing 2) Of all his novels I like this one best. 4( the more…the more…结构 1) The harder you work, the greater progress you will make. 2) The more a man knows, the more he discovers his ignorance. 5( 选择比较 1) I prefer staying at home to going out. 2) They prefer to work rather than (to) sit idly. 3) He prefers to work alone. 注意:这里的1)句用的是Prefer A to B结构,to为介词,后接名词或动名词;2)句是 以不定式作prefer的宾语;3)句用法同2),只是不把rather than部分表达出来。 6( 对比 1) Motion is absolute while stagnation is relative. 2) He is tired out, whereas she is full of vigour. 注意:while 和whereas均可用于连接两个意义对立的分句,相当于汉语的"而"字。许 多考生能较好地运用这一句式,尤其在图表作文中。 3. 表达原因的结构 考生病句: 1. The real reason to our failure is not far to seek. 2. The reason for this is because some people want to earn plenty of money without working hard. 正确表达: 1. The real reason for our failure is not far to seek. 2. The reason for this is that some people want to earn plenty of money without working hard. 评议与分析: 以上两个病句分别引自92年1月和97年12月四级考试的考生作文。从遣词造句上看, 这两位考生具有一定的写作能力,not far to seek, plenty of money以及without working hard等均运用正确、恰当。但令人遗憾的是,第一位考生不知道reason不与to搭配而应接介词 for,第二位考生犯了一个中国学生常犯的错误,就是用because引起表语从句,because这 个词不能引起表语从句,在本句中只能改用that才正确。 掌握好表达原因的结构是十分重要的,几乎所有的写作试题都要求写原因或可以写原 因。在大学英语四、六级考试、研究生入学英语考试以及TOEFL考试中,写作的文体基本 上是议论文,而议论文的基本模式是摆事实、讲道理,讲道理就是说明原因。写作测试的文 体决定了表达原因结构的重要性。 英语中用来表达原因这一概念的结构有多种。我们可用as , because, since, seeing that, considering that, now that, not that...等词组引出表示原因的从句。例如: 1. Now that we have seen these great achievements with our own eyes, we feel more proud than ever of our country. 2. Professor Liu is strict with us because he wants us to make rapid progress. 3. Since we live near the sea, we enjoy a healthy climate. 4. Pollution is still a serious problem, not that we don't have the ability to solve it, but that some people have not realized the consequences of the problem. 我们还可以借助某些词语用简单句表达原因结构。例如: 1. The reason for this change is quite obvious. 用请下载www.docin.com/yongqing 2. Diligence is the key factor of success. 3. Idleness is the root of all evils. 4. He was ashamed to have made the mistake.(=He was ashamed that he had made the mistake. =He was ashamed because he had made the mistake.) 除了上述的例句外,英语中还有很多或易或难的表达原因的结构。我们在进行写作训练 的时候,不能满足于一知半解,要讲究书面语言的正确性和准确性。比如,because是最常 用的引导原因从句的连词,语气最强,表示直接的原因,若because置于句首,后面的主句 不能再用so。用as引导的原因从句语气较弱,所说明的原因是附带的,而since表示的原因 暗示着是稍加分析之后才能推断出来的原因。 4. 否定结构 考生病句 1. Some people think we needn't to worry about fresh water. 2. Nowadays many people don't like to go to the movies, too. 正确表达: 1. Some people think we needn't worry (或don't need to worry) about fresh water. 2. Nowadays many people don't like to go to the movies, either. 评议与分析: 例句1选自96年1月四级考生作文,例句2选自92年1月六级考生作文。例句1 的 错误在于该考生混淆了need作为情态动词和作为普通动词的用法。need作为情态动词时, 主要用于否定句,后面的动词不带to, needn't worry,作we的谓语。need作为实意动词时, 可用于肯定句、否定句和疑问句,don't need to worry 中的to worry作don't need的宾语。例 句2的错误在于该考生混淆了too和either的区别,这两个词都表示"也"的意思,但是在英 语中too, also只能用于肯定句,而either只能用于否定句。 否定结构除了在助动词、情态动词,be和have后面加not之外,还有许多不含not的 否定结构。若能正确使用他们,文章会显得生动活泼,增加写作的闪光点。下面我们就来看 看: 1( 含有否定意义的词汇和短语 以下列举的词和词组本身就具有否定的含义,因此无需用否定词。 介词against, beyond, but, except, without,... 形容词和动词absent, deny, differ, different, fail, free, ignore, miss, refuse, the last, used to, reluctant, lack, want,... 短语keep...from, protect...from, prevent...from, let alone, at a loss, in vain, instead of, out of the question, rather than, too...to, by no means, anything but,... 我们看以下例句: 1) Women fail to get the equal rights in some countries. 在一些国家里妇女没有得到平等的权利。 2) This is by no means the best way to solve the problem of energy crisis. 这不是解决能源危机的最好的办法。 3) We should protect trees from being destroyed. 我们应保护树木,不让它们受破坏。 4) In old China we could not make a nail, let alone(make) machines. 在旧中国,我们连一个钉子都造不了,更不用说制造机器了。 用请下载www.docin.com/yongqing 2( 含有半否定意义的词语 barely, hardly, few, little, rarely, scarcely, seldom, not all, not everyone, not everything,...具 有半否定的意义。例句: 1) We could hardly see any fresh vegetables in winter on market several years ago. 几年前在冬天市场上很难见到新鲜蔬菜。 2) These young people know little about how to choose good books to read. 这些年轻人几乎不知道如何挑选优秀的书籍来读。 3( 不含否定意义的否定结构 有些词和词组形式上是否定结构,但其含义是肯定的,比如:cannot but, can't help, no sooner...than, not...until, in no time, none other than, nothing but,等等。例句: 1) We can't but face the reality. 我们只有面对现实。 2) These old buildings will be replaced by modern apartment buildings in no time. 这些旧建筑将很快为现代化的公寓所代替。 4( 否定结构的倒装语序 我们有时为了强调而把否定词和词组放在句首,这时句子结构应倒装。例如: 1) On no account should we follow blindly.我们决不应当盲从。 2) No where has the world ever seen such great enthusiasm for learning as in our country.没有任何其他地方有我国这样高的学习热情。 5. 含有it的结构 考生病句: 1. As is known to all of us that science and technology play an important role in the development of society. 2. It is known to us, practice makes perfect. 正确表达: 1. It is known to all of us that science and technology play an important role in the development of society. (或:As is known to all of us, science...) 2. It is known to us that practice makes perfect. (或:As is known to us, practice...) 评议与分析: 例句1是93年12月六级考试11分作文的评分样卷句子,例句2选自97年1月四级考 试作文。很显然,两个考生混淆了it和as的用法。如果用it作形式主语,后面的主语从句 必须由that引起;如果用as,则后面不能用that,因为as是关系代词,代表practice makes perfect。 It 在英语中是个相当活跃的代词,在写作中我们常要使用它。以下几种用法应熟练掌 握,并能灵活运用。 1(作形式主语 It is necessary for us to master a foreign language skillfully. It makes difference whether we could purify the air or not. 2.作形式宾语 We find it rather difficult to prevent people from doing that. Modern science has made it possible for babies to grow healthily and for people to live longer. 3.引导强调句 用请下载www.docin.com/yongqing It is only by this way that we can achieve success. It was then that people began to realize the importance of controlling population. 从语法结构来看,上述的句子都不难,或者可以说是考生相当熟悉的。然而在写作考试 中,多数考生缺乏运用自己已掌握的句式的意识,而以自己头脑中拼凑的中式英语取而代之。 其实,只要能恰当地运用上述的任何一个句式,考生的作文就会出现闪光点,就会取得比较 好的成绩。 , How to achieve sentence variety 在写作时要注意句式的变化, 使文章读起来富有生气。同样一个意思可以通过多种句式来表 达。在考虑英语句式的变化时, 我们一般从以下几个方面入手: (1) 既可以用主动式也可以用被动式, 例如: 例1. Today, we use machines not only in industry but in other sectors of national economy as well. Today machines are widely used not only in industry but in other sectors of national economy as well. 例2. Some people believe that it will be possible for automatic machines to replace completely in the future. It is believed by some people that it will be possible for automatic machines to replace completely in the future. (2) 既可以用肯定式也可以用否定式, 例如: 例1. I shall go there unless it rains. I shall go there if it doesn't rain. 例2. Their daily lives don't provide them with the exercise needed to keep them healthy. Their daily lives fail to provide them with the exercise needed to keep them healthy. (3) 某些修饰成分既可以用在句子前面也可以用在后面, 还可以用在中间, 例如: 例1. With a car, people can get around freely. People can get around freely with a car. 例2. In fact, speaking is one of the most important means of communication. Speaking is in fact one of the most important means of communication. (4) 既可以是正常语序也可以是倒装语序, 例如: 例1. A new type of TV sets was among the products on display. Among the products on display was a new type of TV sets. 例2. We did not realize the problem of energy crisis until the end of last century. Not until the end of last century did we realize the problem of energy crisis. (5) 既可以用简单句也可以用并列句或者主从复合句, 例如: 例1. Nowadays a lot of people work in office, spending most of their time indoors. Nowadays a lot people work in office and they spend most of their time indoors. 例2. There are many means of getting information and they enable us to keep up with what is going on in the world. 用请下载www.docin.com/yongqing There are many means of getting information which enable us to keep up with what is going on in the world. III. Writing Step by Step Directions: You are allowed 30 minutes to write a composition on the topic „We Need to Broaden Our knowledge‟. You should write no less than 120 words and you should base your composition on the outline below: 1. 科学技术是社会发展所不可缺少的 2. 社会科学和自然科学相互渗透 3. 现代大学生需要广博的知识 Step I: Write topic sentence Review: In exposition and argumentation we can find topic sentences are frequently used in paragraph writing. Sample 1 The world is full of disappointed people. Some of them probably never had much ambition to start with; they sat back and waited for something good and felt cheated because it never happened. Some of them had very set, specific ambitions and, for one reason or another, never got what they wanted. Others got what they wanted but found it wasn‘t exactly what they‘d expected it to be. Disappointed ambition provides fodder (素材 )for both drama and melodrama (情节剧 ): aspiring athletes ( who could have been contenders), aspiring dancers ( all they ever needed was the music and the mirror ). Sample 2 Is marriage, as is so often supposed, more strongly associated with men‘s happiness than women‘s? In both European and North American national surveys, the happiness gap between the married and never-married is similar for women and men. The results of nearly a hundred such studies confirm this: Although a bad marriage may be more depressing to a woman than to a man, the myth that single women report greater happiness than married women can be ignored. Sample 3 As regards health, I have nothing useful to say since I have little experience of illness. I eat and drink whatever I like, and sleep when I cannot keep awake. I never do anything whatever on the ground that is good for health, though in actual fact the things I like doing are mostly wholesome. Sample 4 Neat people place neatness above everything, even economics. They are incredibly wasteful. Neat people throw away several toys every time they walk through the den. I knew a neat person once who threw away a perfectly good dish drainer because it had mold on it. The drainer was too much trouble to wash. And neat people sell their furniture when they move. Please write topic sentence now! Step II Write supporting sentences The function of supporting sentences: As the topic sentences are decided supporting sentences are used to explain, develop, and support the meaning of the topic sentences. After we finish the topic sentence we should try to see it from different angles, think of it with every aspects, choose the proper ones, and write them down one by one. 用请下载www.docin.com/yongqing Sample 1 Topic sentence: Some zoologists and psychologists compare modern man to a caged lion. Supporting sentences: The first level: Living conditions in crowded cities, they say, are similar to those of animals in a zoo and make the inhabitants unusually aggressive. If the human population had not increased so rapidly, people would have had more space and freedom. The second level: In prehistoric times a group of about 60 people had many kilometers of empty land to n wander and search for food in. If conditions had remained thus, man might have been no more aggressive than his fellow creatures. The third level: As it is, it is possible for as many as 30,000 people to be working in a single office-building. It is not surprising if in these conditions people behave aggressively towards each other. In fact, it is almost impossible for them to behave otherwise. thThe 4 level: Man must have become more aggressive over the years as the world population had increased. ( This is the last sentence and also a concluding sentence.) Sample 2 Topic sentence: How do youths spend their incomes? Supporting sentences: , Both female and male teenagers spend most of their money on clothes, CDs, stereo equipment, entertainment, and travel. , Young women spend most on cosmetics, followed by clothes and jewelry. , Young men spend the most on sporting goods, cameras, CDs, stereo equipment, bicycles, shoes, jeans, musical instruments, and electronic games. Advice The variety of your sentences will make your article vivid, attractive, appealing and interesting. Teenagers often spend hours shopping, especially on weekends. The fact that they are doing more shopping may result in their spending more money in stores they go to. In addition, youths often have a great deal of authority in store-selection decisions, which means that stores must attract them with an effective appeal. Although the popular belief is that young people buy products impulsively and are less rational than the market as a whole, surveys indicate that most respondents aged 14 to 25 compare prices and brands before buying. Research on adolescent shopping behavior has produced the following tentative conclusions: 1)….2)….3)…. How to write supporting sentences? 1. Try to use examples: Examples are always used to present a story, a fact, a piece of information, or the author‘s own experience etc. to support the author‘s view-point. Sample 1 Do hackers regard themselves as criminals? Probably not. But they do not make a distinction between ―good‖ and ―bad‖ hacking. Take, for example, the case of Ian, a 16-year-old hacker. He admits quite proudly that he enjoys doing just for the fun of it. Nevertheless, he prefers breaking into software programs rather than systems because there is less chance of being prosecuted. Ian never hacks for profit although he knows some who do work for money and even just for the fun of causing someone else headaches. Like many members of the hacker community, he does not feel morally obliged to turn ―bad‖ hackers over to the authorities. 用请下载www.docin.com/yongqing Sample 2 In some cases, a patient‘s ―right to die‖ has subtly become a ―duty to die‖. Some people are pressured toward euthanasia by exhausted and impatient relatives. A story is told of a woman whose relatives gathered in Amsterdam for her planned euthanasia. One relative came from overseas. When the patient had last-minute doubts, the family said, ―You can‘t have her come all this way for nothing.‖ Instead of ensuring that the patient‘s true wishes were observed, the doctor carried out the euthanasia. Sample 3 …at least 30 percent of adult women were using the same brands they first chose as teenager. Translated into total market figures, the findings would mean, for instance, that 6,760,000 women still are using the same brand of cosmetics and 8,900,000 still are eating the same kind of packaged cheese that they first bought. Sample 4 …. Like exercise, good nutrition and proper medical care, methods such as relaxation therapies are only one part of the recipe for good health. Still, they are an important ingredient. 2. Proverbs, old sayings, experts‟ theories, facts, information etc. can also be used as supporting sentences. Sample 1 ….Nobody wants to be caught napping or found asleep at work. To quote a proverb: ―Some sleep five hours, nature requires seven, laziness nine and wickedness eleven.‖ Sample 2 ― There is senseless notion that children grow up and leave home when they are 18, and the truth is far from that,‖ says sociologist Larry Bumpass of the University of Wisconsin. Today, unexpected numbers of young adults are living with their parents. ― There is a major shift in the middle class,‖ declares sociologist Allan Schnaiberg of Northwestern University, whose son, 19, moved back in after an absence of eight months. Sample 3 The case for college has been accepted without question for more than a generation. All high school graduates ought to go, says conventional wisdom and statistical evidence, because college will help them earn more money, become ‗better‘ people, and learn to be more responsible citizens than those who don‘t go. Sample 4 As the Bible tells us, worrying about money----or anything else for that matter----won‘t do us any good. ― Who of you by worrying can add a single hour to his life?‖ Jesus asked. ―And why do you worry about clothes? See how the lilies of the field grow? They do not labor or spin.‖ Sample 5 …. Will you have a stronger immune response and greater health if you are happier, less stressed, and more optimistic? Experts believe that the answer is yes. There are studies showing that by employing certain mind-body techniques that help reduce stress and improve outlook, cancer patients can live longer. …. English Proverbs , All roads lead to Rome. , Art is long, but life is short. , Beauty lies in lover‘s eyes. , Diligence is the mother of good luck. , Don‘t put off till tomorrow what should be done today. 用请下载www.docin.com/yongqing , Doubt is the key of knowledge. , Easier said than done. , Easy come, easy go. , Facts speak louder than words. , Failure is the mother of success. , Four eyes see more than two. , A friend in need is a friend indeed. , God helps those who helps themselves. , Health is not valued till sickness comes. , He laughs best who laughs last. , Hope for the best and prepare for the worst. , It is the last straw that breaks the camel‘s back. , It‘s never too late to learn. , Knowledge is power. , Love is blind. , Many hands make quick work. , Money makes the mare go. , Never judge by appearances. , No news is good news. , There are two sides to every question. , Time cures all things. , Time is money. , Time and tide wait for no man. , Two heads are better than one. , Well begun is half done. , Where there is a will, there is a way. Quotations from Benjamin Franklin , Early to bed, early to rise makes a man healthy, wealthy, and wise. , God helps them that helps themselves. , Diligence is the Mother of good luck. , Energy and persistence conquer all things. , Genius without education is like silver in the mine. , Having been poor is no shame, but being ashamed of it, is. Quotations from John Dewey , Failure is instructive. The person who really thinks learns quite as much from his failures as from his successes. , Education is not a preparation for life; education is life itself. , In brief, the function of knowledge is to make one experiences freely available to other experiences. Quotations from Eric Hoffer Nature attains perfection, but man never does. There is a perfect ant, a perfect bee but man is perpetually unfinished. He is both an unfinished animal and an unfinished man. It is this incurable unfinishedness which sets man apart from other living things. For, in the attempt to finish himself, man becomes a creator. Moreover, the incurable unfinishedness keeps man perpetually immature, perpetually capable of learning and growing. Lessons from Jefferson , Go and see. , You can learn from everyone. , Judge for yourself. , Do what you believe is right. , Trust the future; trust the young. , Neither believe nor reject anything, because any other person has rejected or believed it. Heaven has given you a mind for judging truth and error. Use it. 用请下载www.docin.com/yongqing , There are two sides to every question. If you take one side with decision and act on it with effect, those who take the other side will of course resent your actions. Quotations from Ralph Waldo Emerson , To be great is to be misunderstood. , Good is a good doctor, but Bad is sometimes a better. , What lies behind us and what lies before us are small matters compared to what lies within us. , The reward of a thing well done, is to have done it. , All live is an experiment. The more experiments you make the better. Please write supporting sentences now! Paragraphs , Introductory paragraph: It is the first paragraph and it comes to the subject. Sample 1 Exercise. Eat right. Don‘t smoke. These are some of the most common words of advice to people who wish to stay healthy. But a growing amount of scientific research shows that there is another, equally important, aspect to staying well, peace of mind. peace of mind. Sample 2 Marketers are interested in understanding what products will sell well in the youth market. It is also important to appreciate the influence that young people have on the purchases of others, such as parents. In fact, sometimes marketers are more interesting in young people‘s influence on other buyers than in their role as the main purchasers of certain items. Summary: We find from the two paragraphs that the function of the introductory paragraph is to raise the problem and then deal with or discuss the problem in the following paragraphs. , Concluding Paragraph Sample 1 So remember, if music be the food of love, do it slowly, especially if you‘re driving, then we can all ―play‖ on. Sample 2 Each one of us is a unique, valuable individual. We are interesting in our own personal ways. The better we understand ourselves, the easier it becomes to live up to our full potential. Let‟s not allow shyness to block our chances for a rich and fulfilling life. Summary: We learned from the concluding paragraphs that in the last paragraph the author will come to a conclusion for the argument or statement in the previous paragraphs. Please remember, as human beings, our own thought is the most important thing , which will make our article bright. The articles in our textbooks are good models for us to learn writing skills so please enjoy them. The Sample One We Need to Broaden Our Knowledge We all know that knowledge, especially scientific and technological knowledge, is necessary to develop our modern society. Without it, we may still stay in primitive society, neither conquering the nature, nor knowing the essence of the earth. Once we acquire the knowledge about nature, we come to realize that it is still impossible to build an ideal society by relying only on scientific and technological knowledge. 用请下载www.docin.com/yongqing Therefore social knowledge becomes essential, too. Once we know about it, we obey the rule of social life, and learn the social culture such as history, politics, and music. However, the most important point is that social science interacts with natural science. In the past years, we just focused on our own development but neglected the protection of natural resources. Having got lessons from our past activities, therefore, we have already noticed the importance of interaction and have been trying to get benefits from it with the minimal loss. As for college students, we should acquire comprehensive knowledge, which is not only the need of our society, but also the need of our new century. 用请下载www.docin.com/yongqing
/
本文档为【英语四级培训教师用书】,请使用软件OFFICE或WPS软件打开。作品中的文字与图均可以修改和编辑, 图片更改请在作品中右键图片并更换,文字修改请直接点击文字进行修改,也可以新增和删除文档中的内容。
[版权声明] 本站所有资料为用户分享产生,若发现您的权利被侵害,请联系客服邮件isharekefu@iask.cn,我们尽快处理。 本作品所展示的图片、画像、字体、音乐的版权可能需版权方额外授权,请谨慎使用。 网站提供的党政主题相关内容(国旗、国徽、党徽..)目的在于配合国家政策宣传,仅限个人学习分享使用,禁止用于任何广告和商用目的。

历史搜索

    清空历史搜索